2ちゃんねる スマホ用 ■掲示板に戻る■ 全部 1- 最新50    

■ このスレッドは過去ログ倉庫に格納されています

■ちょっとした物理の質問はここに書いてね184■

1 :ご冗談でしょう?名無しさん:2014/12/24(水) 22:24:38.38 ID:kJi775RD.net
前スレ
■ちょっとした物理の質問はここに書いてね183■ [転載禁止](c)2ch.net
http://wc2014.2ch.net/test/read.cgi/sci/1415802393/

★荒らし厳禁、煽りは黙殺
★書き込む前に   >>2   の注意事項を読んでね
★数式の書き方(参考)はこちら   >>3-5   (予備リンク:   >>2-10    )
===質問者へ===
重要 【 丸 投 げ 禁 止 】

・質問する前に
1. 教科書や参考書をよく読む
2.
http://www.google.com/
  などの検索サイトを利用し、各自で調べる
3. 学生は自分の学年、物理科目の履修具合を書く
4. 宿題を聞くときは、どこまでやってみてどこが分からないのかを書く
5. 投稿する前に、ちゃんと質問が意味の通る日本語か推敲する、曖昧な質問文には曖昧な回答しか返せない
・「力」「エネルギー」「仕事」のような単語は物理では意味がはっきり定義された言葉です、むやみに使うと混乱の元
・質問に対する回答には返答してね、感謝だけでなく「分からん」とかダメOK
・質問するときはage&ID表示推奨
・高度すぎる質問には住人は回答できないかもしれないけれど、了承の上での質問なら大歓迎

===回答者へ===
・丸投げは専用スレに誘導
・不快な質問は無視、構った方が負け
・質問者の理解度に応じた適切な回答をよろしく
・単発質問スレを発見したらこのスレッドへの誘導をよろしくね
・逆に議論が深まりそうなら新スレ立てて移動するのもあり
・板違いの質問は適切な板に誘導を
・不適切な回答は適宜訂正、名回答は素直に賞賛

2 :ご冗談でしょう?名無しさん:2014/12/24(水) 22:25:06.09 ID:???.net
数式の書き方の例 ※適切にスペースを入れると読みやすくなります
●括弧: (), [], {}を適切に入れ子にして分かりやすく書く
●スカラー: a,b,...,z, A,...,Z, α,β,...,ω, Α,Β,...,Ω,...(「ぎりしゃ」「あるふぁ〜おめが」で変換)
●ベクトル: V=(v1,v2,...), |V>,V↑, (混乱しないならスカラーの記号でいい。通常は縦ベクトル)
●テンソル: T^[i,j,k...]_[p,q,r,...], T[i,j,k,...; p,q,r,...]  (上下付き1成分表示)
●行列: M[i,j], I[i,j]=δ_[i,j] M = [[M[1,1],M[2,1],...], [M[1,2],M[2,2],...],...], I = [[1,0,0,...],[0,1,0,...],...]
(右は全成分表示。行または列ごとに表示する。例:M=[[1,-1],[3,2]])
●対角行列: diag(a,b) = [[a,0],[0,b]]
●転置行列・随伴行列:M^T, M†("†"は「だがー」で変換可) ●行列式・トレース:|A|=det(A), tr(A)
●複号: a±b("±"は「きごう」で変換可)
●内積・外積: a・b, a×b
●関数・汎関数・数列: f(x), F[x(t)] {a_n}
●平方根: √(a+b) = (a+b)^(1/2) = sqrt(a+b) ("√"は「るーと」で変換可)
●指数関数・対数関数: exp(x+y)=e^(x+y) ln(x)=log_e(x) (底を省略して単にlogと書いたとき多くは自然対数)
 括弧を省略しても意味が容易に分かるときは省略可: sin(x) = sin x
●三角関数、逆三角関数、双曲線関数: sin(a), cos(x+y), tan(x/2), asin(x)=sin^[-1](x), cosh(x)=[e^x+e^(-x)]/2
●絶対値:|x| ●ノルム:||x|| ●共役複素数:z^* = conj(z)
●階乗:n!=n*(n-1)*(n-2)*...*2*1, n!!=n*(n-2)*(n-4)*...

3 :ご冗談でしょう?名無しさん:2014/12/24(水) 22:25:37.76 ID:???.net
質問・回答に標準的に用いられる変数の例

a:加速度、昇降演算子 A:振幅、ベクトルポテンシャル B:磁束密度 c:光速 C:定数、熱・電気容量
d:次元、深さ D:領域、電束密度 e:自然対数の底、素電荷 E:エネルギー、電場
f:周波数 f,F:力 F:Helmholtzエネルギー g:重力加速度、伝導度
G:万有引力定数、Gibbsエネルギー、重心 h:高さ、Planck定数 H:エンタルピー、Hamiltonian、磁場
i:虚数単位 i,j,k,l,m:整数のインデックス I:電流、慣性モーメント j:電流密度・流束密度
J:グランドポテンシャル、一般の角運動量 k:バネ定数、波数、Boltzmann定数 K:運動エネルギー
l,L:長さ L:Lagrangian、角運動量、インダクタンス m,M:質量 n:物質量 N:個数、トルク
M:磁化 O:原点 p:双極子モーメント p,P:運動量、圧力 P:分極、仕事率、確率 q:波数
q,Q:一般化座標、電荷 Q:熱 r:距離 R:抵抗、気体定数 s:スピン S:エントロピー、面積 t,T:時間 T:温度
U:ポテンシャル、内部エネルギー v:速度 V:体積、ポテンシャル、電位
W:仕事、状態数 x,y,z:変数、位置 z:複素変数 Z:分配関数

β:逆温度 γ:抵抗係数 Γ:ガンマ関数 δ:微小変化 Δ:変化 ε:微小量、誘電率 θ:角度 κ:熱伝導率
λ:波長、固有値 μ:換算質量、化学ポテンシャル、透磁率 ν:周波数 Ξ:大分配関数 π:円周率 ρ:(電荷)密度、抵抗率
σ:スピン τ:固有時 φ:角度、ポテンシャル、波動関数 ψ:波動関数 ω:角振動数 Ω:状態密度

4 :ご冗談でしょう?名無しさん:2014/12/24(水) 22:27:09.45 ID:???.net
>>3
このテンプレ不要だと思う

5 :アリストテレス:2014/12/24(水) 22:55:17.43 ID:???.net
>>1
死ね。

6 :ご冗談でしょう?名無しさん:2014/12/24(水) 23:18:36.49 ID:???.net
おまえもな

死んで欲しい糞コテ
http://anago.2ch.net/test/read.cgi/tubo/1343379513/162,163

7 :ご冗談でしょう?名無しさん:2014/12/25(木) 09:03:15.73 ID:???.net
切断の事で質問があります

原子レベルの刃先を持つ刃物があれば大抵のものは切断できますか?
もしも上記で切断できないものがある場合、教えてもらいたいです
できれば理由も教えてもらえると助かります
よろしくお願いします

結合エネルギーが邪魔になるのか、刃先と同等または硬いものは切断できないのか
そこの所も詳しく教えていただけると嬉しいです

8 :ご冗談でしょう?名無しさん:2014/12/25(木) 10:30:35.93 ID:???.net
プログレッシブ・ナイフのことかな?

9 :ご冗談でしょう?名無しさん:2014/12/25(木) 10:42:25.80 ID:???.net
原子レベルの刃で大根を切るのは大変だろう

10 :ご冗談でしょう?名無しさん:2014/12/25(木) 12:27:04.57 ID:AQKAe47h.net
どこで聞けばいいのかわからないのでここに
灯油を燃やした時に出る紫外線はどのくらいのものですか?(いろいろ検索したが出てこなかった)
火災報知器で紫外線に反応するものがあるので炎から紫外線は出ていると思うのですが
どのくらいの量なのでしょうか?
ストーブで日焼けしてしまうでしょうか?

11 :ご冗談でしょう?名無しさん:2014/12/25(木) 12:50:39.09 ID:???.net
永遠の無になりたいのですが、自殺しても永遠の無にはなれませんか?

12 :ご冗談でしょう?名無しさん:2014/12/25(木) 13:25:17.01 ID:???.net
>>10
出るわけねー

13 :ご冗談でしょう?名無しさん:2014/12/25(木) 14:10:07.78 ID:???.net
目子レベルの筋で男根を切るのは大変だろう

14 :ヤコブ:2014/12/25(木) 14:49:30.07 ID:???.net
旅客機のパイロットと建築家はどっちの方が凄いのでしょうか?

15 :ご冗談でしょう?名無しさん:2014/12/25(木) 16:13:08.88 ID:???.net
荒らしは去れ

16 :ご冗談でしょう?名無しさん:2014/12/25(木) 16:47:54.86 ID:???.net
>>10
有機物のスペクトルは化学の人が専門なんじゃない?
赤外域の話だけど、吸収スペクトルから分子構造を調べたりするらしいし

17 :ご冗談でしょう?名無しさん:2014/12/25(木) 18:06:07.74 ID:???.net
280nm以下がなんたらかんたら
https://www.jstshingi.jp/abst/p/08/829/iwate3.pdf
http://www.amazon.co.jp/dp/4274087131

18 :ご冗談でしょう?名無しさん:2014/12/25(木) 23:12:41.12 ID:???.net
なるほど、火災以外の料理やストーブでは反応しないように紫外線を使うわけか
だったら、ますますストーブから出るわけが無い

19 :ご冗談でしょう?名無しさん:2014/12/26(金) 00:47:16.10 ID:???.net
>>10
確か、量子力学の成立に関係した理論で、Rayleigh Jeans の法則絡みで
波長とエネルギーレベルの関係式があるはず。

20 :19:2014/12/26(金) 00:51:45.74 ID:???.net
レイリー・ジーンズの法則やウィーンの法則は実験とずれることが
判明し、結局:

http://ja.wikipedia.org/wiki/%E3%83%97%E3%83%A9%E3%83%B3%E3%82%AF%E3%81%AE%E6%B3%95%E5%89%87

↑のプランクの法則が正しい式だということが分かったという話。

灯油を燃えている時の温度が分かれば、波長と輝度の関係が分かる。

21 :ご冗談でしょう?名無しさん:2014/12/26(金) 01:28:34.80 ID:???.net
この重りは何グラム以上か計算できますか?
https://www.youtube.com/watch?v=Axvm89_odX4

22 :ご冗談でしょう?名無しさん:2014/12/26(金) 11:49:21.03 ID:???.net
先に前スレ埋めようよ

23 :ご冗談でしょう?名無しさん:2014/12/26(金) 18:21:43.58 ID:???.net
自分で埋めろよ

24 :ご冗談でしょう?名無しさん:2014/12/26(金) 19:21:05.36 ID:???.net
狸を埋めろ

25 :ご冗談でしょう?名無しさん:2014/12/26(金) 21:12:51.00 ID:???.net
なんでこれずっと回ってるの?
https://www.youtube.com/watch?v=287qd4uI7-E

26 :ご冗談でしょう?名無しさん:2014/12/26(金) 23:05:04.23 ID:???.net
十分に摩擦を低減させてやれば限られた時間内では止まらない様に
動かしてやることはできるだろうね。

27 :ご冗談でしょう?名無しさん:2014/12/26(金) 23:14:02.31 ID:???.net
一番最初のSelf-flowing flaskから怪しいんだけど。
細いパイプの出口が、大きな容器の液面より高い位置にあるから
流れ続けるはずがない。

28 :ご冗談でしょう?名無しさん:2014/12/26(金) 23:16:32.14 ID:???.net
ナノモーター使ってるじゃ

29 :ご冗談でしょう?名無しさん:2014/12/26(金) 23:22:08.69 ID:???.net
右の支持部に到達してから急に吸い上げられてるから、多分そこにポンプ機構が隠れているんだろう。
左下にポンプを埋め込むと、左の容器の液面が最初から不自然に下がるから、液体の体積が十分溜まった後で右のポンプが働くように工夫されている。
他のも巧妙に動作機構が隠されている。

30 :ご冗談でしょう?名無しさん:2014/12/27(土) 12:26:37.72 ID:???.net
日本をGDP世界一にするにはどうすれば良いのでしょうか?

31 :ご冗談でしょう?名無しさん:2014/12/27(土) 14:53:47.91 ID:???.net
実在哲学の管理人はすごいんですか?

32 :ご冗談でしょう?名無しさん:2014/12/27(土) 15:03:57.23 ID:???.net
完全な熱力学関数として
U(S,V,N),F(T,V,N),G(T,P,N),H(S,P,N),J(T,V,μ)
はよく聞くけど、変数の組み合わせとして残った
(S,V,μ),(S,P,μ)
からなる熱力学関数には名前付いてるんでしょうか?

33 :ご冗談でしょう?名無しさん:2014/12/27(土) 15:59:25.19 ID:???.net
いままでは、仕事は無茶きつくても、社員持ち株のストックオプションで
一攫千金、20代で巨額の金を稼いで、30代で引退し、あとは悠々自適
の生活を、というような感じのゴールドラッシュだったのだが,
共産圏が崩れたのと通信の進歩に依り、仕事が貧乏人の中国やインドに
どんどん流れていって、賃金が下がり、首切りもどんどん増えている状況では、
人気が無くなるのも宜なるかな。
あとは、大学レベルの計算科学って、所詮は数学パズルとほとんど変わらない
ような状況で,面白いと感じるのはパズルマニアぐらい。
それと,アルゴリズム特許などが台頭してくると,プログラムを書くのにも
いちいち過去の事例だのを調べなけりゃならんし、特許を取れと上からつつか
れると、先行事例や論文をサーベイしてからじゃないと出来ない。これは
負担として著しく多い。昔は過去の人がどうしたこうしたというのは知識と
して知っていれば、楽が出来たり、無駄な努力を避けられるという点では
知識は力なりだったのだが、今やプログラムを自分の思うままに自由気ままに
書いて製品にするということは難しくなった。プログラミング環境も、
やたらとぶ厚い電話帳のようなライブラリの説明書を座右に置いて、そう
いったお仕着せのAPIで書かないと、いけないとされ、ようするに行儀作法が
無闇に要求されるようになった。共同作業ではなおさらそうだ。
そういった、お仕着せのルールだの,事例調査うんぬんのような御役所
仕事みたいな世界は、アメリカンドリーム的ではもはやない。
マイクロソフトによる事実上の市場支配も、それ以外の会社に勤める
人間にとっては、陰鬱な現実である。

34 :ご冗談でしょう?名無しさん:2014/12/27(土) 16:00:11.05 ID:???.net
コピペ

35 :ご冗談でしょう?名無しさん:2014/12/27(土) 16:25:56.45 ID:???.net
真っ当な意見と見せかけ、実は詭弁で論点をはぐらかす輩が多々おります。
皆様も以下の「詭弁の特徴」を覚え、そういう輩を排除しましょう。。
例:「犬ははたして哺乳類か」という議論をしている場合
   あなたが「犬は哺乳類としての条件を満たしている」と言ったのに対して
   否定論者が…
 1:事実に対して仮定を持ち出す
     「犬は子供を産むが、もし卵を生む犬がいたらどうだろうか?」
 2:ごくまれな反例をとりあげる
     「だが、尻尾が2本ある犬が生まれることもある」
 3:自分に有利な将来像を予想する
     「何年か後、犬に羽が生えないという保証は誰にもできない」
 4:主観で決め付ける
     「犬自身が哺乳類であることを望むわけがない」
 5:資料を示さず自論が支持されていると思わせる
     「世界では、犬は哺乳類ではないという見方が一般的だ」
 6:一見関係ありそうで関係ない話を始める
     「ところで、カモノハシは卵を産むのを知っているか?」
 7:陰謀であると力説する
     「それは、犬を哺乳類と認めると都合の良いアメリカが画策した陰謀だ」
8:相手のイデオロギーの裏をかき悪用する
      「左翼の間では犬は哺乳類であるというのは常識になっているけどね。」
 9:知能障害を起こす
     「何、犬ごときにマジになってやんの、バーカバーカ」
 10:自分の見解を述べずに人格批判をする
     「犬が哺乳類なんて言う奴は、社会に出てない証拠。現実をみてみろよ」
 11:ありえない解決策を図る
     「犬が卵を産めるようになれば良いって事でしょ」

話題そらしの定型のです。皆さん注意しましょう!

36 :ご冗談でしょう?名無しさん:2014/12/27(土) 16:26:51.33 ID:???.net
詭弁のガイドラインはそれ自体が詭弁っていう高度なネタなんだけどな

37 :ご冗談でしょう?名無しさん:2014/12/27(土) 16:35:37.85 ID:???.net
コピペコピペ

38 :ご冗談でしょう?名無しさん:2014/12/27(土) 18:26:23.77 ID:???.net
計算理論といえば、ここ30年もずっとP≠NP問題が主題で、
未だに解決の糸口が見当たらないらしい。
この壁を突破しないと、本格的な「数理科学」とは言えない。
この間の素数判定⊆Pの快挙も、無名のインド人だった
本場アメリカは何やってんだという感じではある。

39 :ご冗談でしょう?名無しさん:2014/12/27(土) 18:29:42.17 ID:???.net
>>38
え、素数判定ってP問題だったの?
arXiveのURLとかある?
まあショアのアルゴリズムで解けるから違和感はないが

40 :ご冗談でしょう?名無しさん:2014/12/27(土) 18:33:04.86 ID:???.net
コンピュータ科学の論点は、
数学的な公理系を擬似的にエミュレートする点にある。

一方、コンピュータ工学の発展には
既存の社会科学的理論を仮想的なデバイスの利用で、
応用することが出来る。

発展の分野はまだ多くあるし、多くの工学と数理が並列的である分野はいままでになかったので
「情報科学」なる分野が生まれている。これらは社会科学や人文科学の理論体系の試金石となる。

パソコンやOAについてしか捉えていない議論は無駄だとしか思えない。

41 :ご冗談でしょう?名無しさん:2014/12/27(土) 18:34:33.59 ID:???.net
>>32
定義は?

42 :ご冗談でしょう?名無しさん:2014/12/27(土) 18:37:32.48 ID:???.net
楽しいよコンピュータ
いろいろできるからねぇ

知識欲のある人間にとっては広い興味を持てる計算機は面白い
それ故危険な部分もあるんだけど...

43 :ご冗談でしょう?名無しさん:2014/12/27(土) 18:39:00.11 ID:???.net
ところでこのスレは何のスレでしたっけ?

44 :ご冗談でしょう?名無しさん:2014/12/27(土) 18:40:44.16 ID:???.net
>>41
所与の引数を持つ完全な熱力学関数という定義ではご不満なの?

45 :ご冗談でしょう?名無しさん:2014/12/27(土) 18:44:47.69 ID:???.net
大学での情報教育が
ワード、エクセル、パワーポイント、OUTLOOK
でのおゆうぎの時間でしかない。

こんな「情報教育」なんて、百害あって一利無し。
コンピュータメーカーの市場として産業の育成としての
売れ残りパソコンの捌け口にしか思っていない国や役所と、
下らない教育もどきをやってる大学が多すぎる。

46 :ご冗談でしょう?名無しさん:2014/12/27(土) 18:49:18.45 ID:???.net
コンピュータなんて所詮は文房具だから、割り切って計算力をいかに使うか、
考えたほうがよいね。物理、数学、化学、分子生物学、すべて計算力があって
サイエンスが成り立つ。MITやUCBのコンピュータサイエンスは、もともとUNIXや
INTERNETを作ったところだから、基礎がしっかりしている。日本も長期的に
考えて、東大、京大、東工大などが、新しいUNIXや通信システムを作れる
ようになるにはどうしたらよいか考えよう。

47 :ご冗談でしょう?名無しさん:2014/12/27(土) 19:06:39.59 ID:???.net
まあ物理もコンピュータがなければ、えらく困るには違いない

モデルを作るにしても作り方によっては、計算して答えが出てくるまでに果てしなく時間がかかる

そういう意味では研究の根幹にも関わりかねないが

48 :ご冗談でしょう?名無しさん:2014/12/27(土) 19:10:33.38 ID:???.net
2chで「学問・理系」の「情報システム」というスレッドを観てみれば、
いかに「情報」関係が学問じゃない具合に認識されているかが良く判る。
情報を看板にしている会社への就職事情だの、社内の人事の噂などが
どこが「学問・理系」だぁ? そんなやからが、情報を語り、情報系の企業に
就職しているのが、日本の実状かと思えば、馬鹿らしくって。

49 :ご冗談でしょう?名無しさん:2014/12/27(土) 19:13:47.63 ID:???.net
コンピュータ科学はまだ終ってなくても
コンピュータ屋は終ってるからな・・・
現実問題として勉強したことが「パラダイムシフト」で無駄になる、
覚えてもどうせ無駄なら勉強しないほうが賢い選択だと言える。
それよりもコンピュータ屋が身につけておく一生ものの知識技術は沢山ある。
デジタル土方として使い捨てにされたあとは
・食べられる野草・木の実の知識
・毒草の見分け方
・魚釣りの技術
・イノシシ狩等の技術
・火を起こす技術
・木の葉でテントを張る技術
・水の確保の仕方
なんてのが役に立つからな。

50 :ご冗談でしょう?名無しさん:2014/12/27(土) 19:19:06.91 ID:???.net
どんどんと、プログラミング言語とかOSとか、システムコールとか
クラスライブラリなどをクルクルと、ちょこまか勝手に変更し、
メーカー毎に少しずつ方言を作り、ソースコードを、末端で勝手に
手直しし、ロクなドキュメントを書かないでコードをどんどん書き飛ばす
のが花形プログラマーで、彼/彼女に頼らなければ何も出来ないから
文句も言わず、さくさくとコードを書いて貰って、納品し、後は野となれ
山となれ。後日問題が起きたら、再びコーディングし直して、行きあたり
ばったりの設計と、土方仕事のコードとデバッグで、やっつけ仕事を
すれば、人月百万の仕事。そういったただれた土建屋同様以下の業界が
ソフトウェア業界。

51 :ご冗談でしょう?名無しさん:2014/12/27(土) 19:22:46.03 ID:???.net
いずれ、アニメーター同様の労働環境になることは間違いない。
あまり知られていないが、昭和30年代後半〜40年代前半までは、
日本のアニメーターは花形職業で、仕事はきつかったが賃金も良かった。
ところが、外国に仕事を外注するようになり始めてから、賃金は据置き化
上がらなくなり、国内での仕事量が減ると、受注合戦になって単価は
下がるばかりで儲からず、会社も直接雇用せずに、個人業者との
請負契約制度へ実態を偽装するようになった。最低賃金制はもはや関係ない。
プログラマーも、おそらく企画、設計の上層部は普通のサラリーマン
並の給金を得ることが将来も可能だろうが、いずれそれ以外の労働者は、
海外ネットワークの労働者と同一レベルの賃金しか取れなくなることを
呑まされるようになる。そうして年収200万円の個人事業主になることを
目指して、ネットの向こうの中国人やインド人を相手に日夜競争し努力する
ことになる。

52 :ご冗談でしょう?名無しさん:2014/12/27(土) 19:26:08.39 ID:???.net
新しく考え出される物理モデルや系の計算量のオーダーに対して、
コンピュータの進化のスピードが遅すぎるのが問題かな。というか、そもそも
ノイマン型で全てをやろうって時点である程度の限界が見えてるのが面白くない要素のひとつなのかも。
(=処理能力の限界が見えている、完全に数値化されている)
量子コンピュータや生体コンピュータが出てきたら少しは面白くなってくると思うよ。
まぁSi半導体での限界も近づきだしたので次のフェーズに入るのは時間の問題だと思う。

53 :ご冗談でしょう?名無しさん:2014/12/27(土) 19:36:37.80 ID:???.net
日本でも情報工学/情報科学出ても、大手のメーカーにいけば装置制御の
ためのプログラムとか、せいぜいが携帯のアプリ書きで、多くはCOBOLとか
通信会社の制御ソフト、あるいはWINDOWS系なら英語のソフトの日本語化
というような詰まらん仕事があるだけで、社会を変革させるようなソフトや
基本ソフト、などの仕事は滅多無い。典型的なのは、過去に開発された
膨大なソフトのデバッグや保守ばかりとなっている。つまり、全然楽しく
ないんだよね。ソフト作るぞっていって就職しても、実際は出向してSE
という名目の土方作業ばかりだったりとか。せいぜいが既存の装置とソフトを
組み合わせてシステムとして納品するための設計でしかなかったりするんだ。
客からクレームがあれば、電話一本でトンでいって、OSにパッチを当てたり
昼も夜もなかったりする。社会の底辺で支える3K的なものでしかない。
それなのに、自分はXXをやっているということを公言することは許されない
職業だ。

54 :ご冗談でしょう?名無しさん:2014/12/27(土) 19:45:41.94 ID:???.net
就職してもその多くは、うわっつら撫でるような人海戦術に投入されるだけ。
日本だと例えば、JAVAの開発ツールを使って、ユーザーインタフェースを
書いたり、WEBページを毎日作りなおしたりさせられるのがオチ。
もうすこし基本的なネットワーク関係だと、日夜マシンルームでサーバーの
管理と障害の回復、モニターリング、ユーザーサポートが職務だったりする。
もっとよくてもSEのうちシステムプラニングで、お客の要望に応じて
各社の製品をどう組み合わせて予算を浮かせて利益を出すかと言うことに
悪戦苦闘する。しかも、ハードもソフトも陳腐化が激しくて、つぎからつぎ
へと出る製品の仕様やツールのマニュアルをどんどんよみつづけなければ
ならず、数年前に得た知識はたちまち無用になっていくので、経験の蓄積
がされにくい、蓄積しても利点にならず陳腐化するので、使用者側も
年齢の多い人員に高い給料を払うメリットが無いので、30とか35歳になったら
世程出来る奴とかハイレベルの設計が出来る人間を除いて、肩を叩いて
辞めさせるか、あるいは従量制労働/成果主義で賃下げをするようになる。

55 :ご冗談でしょう?名無しさん:2014/12/27(土) 20:04:17.44 ID:???.net
まーた新しいキチガイかよ・・・

56 :ご冗談でしょう?名無しさん:2014/12/27(土) 20:52:56.71 ID:???.net
コンピュータサイエンスとは, 人間の領域と神の領域とに線引きをする学問で
数学基礎論の一分野である. 人間ができることとは何だろうか, 神にしかでき
ないことは何だろうか, そのようなことを考えるのがコンピュータサイエンス
である.

57 :ご冗談でしょう?名無しさん:2014/12/27(土) 20:58:47.22 ID:???.net
金属と金属を接合しても空乏層はできると思うのですが、
いろんな本を探してみてもそのことについて書いてある本がありません
なぜですか?

58 :ご冗談でしょう?名無しさん:2014/12/27(土) 21:02:09.10 ID:???.net
その昔鉄道愛好会がハカー発祥の地となったMITだが、計算機科学の学生なんて
20年くらい前にはすでに大多数が只働きの過重労働コーダーになってたって聞いたな。
Xはそんな奴隷労働で作ったんだろうな。
不満が鬱積してはワームやトロイを作るやつが後を絶たなかったって。
それでも起業できるチャンスがあるうちは良かったが、今はそれも無いしねえ。。。

59 :ご冗談でしょう?名無しさん:2014/12/27(土) 21:04:02.45 ID:???.net
基本的に「科学」そのものは
メシの種にはなりにくいからね
こいつは理系だろうが文系だろうが
変わらないと思うよ

つーかこの国の教育関係者って
基本的にその手の「基礎的分野」てのが
大嫌いだと思うよ
「役に立つか立たないか」が
その学問の価値を決める大前提らしいし

総合大学に「工学部門」をつくったのも
この国が最初みたい

60 :ご冗談でしょう?名無しさん:2014/12/27(土) 21:23:35.16 ID:???.net
>>57
出来ないよ

61 :ご冗談でしょう?名無しさん:2014/12/27(土) 22:14:42.01 ID:???.net
山口人生が解いた

62 :ご冗談でしょう?名無しさん:2014/12/27(土) 22:17:31.03 ID:???.net
ああああああああああああああああああああああああああああああああああああああああああああああああああああ

63 :ご冗談でしょう?名無しさん:2014/12/27(土) 22:18:53.96 ID:???.net
かまってちゃん

64 :ご冗談でしょう?名無しさん:2014/12/27(土) 22:28:21.92 ID:???.net
大英帝国と道明寺財閥はどっちの方が偉大ですか?

65 :ご冗談でしょう?名無しさん:2014/12/28(日) 08:47:34.61 ID:HN1wS4Xc.net
地球にいる双子のAとBが互いに反対側に飛び立って同時に帰ってきたときは
双子のパラドックスはどうなるの?一方が地球に留まっているという普通の
場合は分かるのだが。

66 :ご冗談でしょう?名無しさん:2014/12/28(日) 08:48:48.21 ID:???.net
スピードが同じならずっと同い年だろうに。

67 :ご冗談でしょう?名無しさん:2014/12/28(日) 10:15:23.78 ID:ZQ+70bIN.net
化学結合と科学結合が同士となるから、空気も反応してしまうから。

68 :ご冗談でしょう?名無しさん:2014/12/28(日) 10:16:21.17 ID:ZQ+70bIN.net
化学結合と化学結合が同士となるから、空気も反応してしまうから。

69 :ご冗談でしょう?名無しさん:2014/12/28(日) 10:52:34.35 ID:???.net
ゴミを書くな

70 :ご冗談でしょう?名無しさん:2014/12/28(日) 13:35:27.82 ID:HN1wS4Xc.net
>>66
相対論によれば、この場合は全く対照的なので、双方が相手より若いという
本当のパラドックスになると思うが?
片方が地球に留まっている場合は、確かに非対称だが。

71 :ご冗談でしょう?名無しさん:2014/12/28(日) 13:38:34.75 ID:???.net
何かが無いと波って伝わらないと思うけど光は何の波なの?

72 :ご冗談でしょう?名無しさん:2014/12/28(日) 15:17:34.76 ID:???.net
高橋康だっけに興味深い記述があってだな

「光は何もない真空中を伝わるという考え方に我々が慣れてしまっただけで
 何故何も媒質がないのに光が走れるのかはまるでわかっていない」

とかなんとか

73 :ご冗談でしょう?名無しさん:2014/12/28(日) 15:42:00.10 ID:???.net
空気中を伝わる音だって分子間は真空だろ
だいたい真空が何もないわけじゃないし

74 :ご冗談でしょう?名無しさん:2014/12/28(日) 15:48:01.47 ID:???.net
やっぱり何かを伝わってる訳だね。光って無くなるもんね
照らした光がどこに消えたのか不思議だった。

75 :ご冗談でしょう?名無しさん:2014/12/28(日) 16:09:55.27 ID:???.net
数理モデルに実在の媒質は必要ないでしょ
あっても問題ないけど

76 :ご冗談でしょう?名無しさん:2014/12/28(日) 19:05:20.94 ID:???.net
大英帝国とアインシュタインはどっちの方が偉大ですか?

77 :ご冗談でしょう?名無しさん:2014/12/28(日) 19:38:51.73 ID:???.net
なんでカセットテープからDATに通してUSB音響機器でPCに取り込もうとしても、
PCのSPDIF入力のレベルが0になって取り込めないんですか?
ブルーレイの光出力をDATに通したものは取り込めるのに。

78 :ご冗談でしょう?名無しさん:2014/12/28(日) 20:23:25.21 ID:???.net
建築学と計算機科学はどっちの方が難しいのでしょうか?

79 :ご冗談でしょう?名無しさん:2014/12/28(日) 20:55:17.95 ID:???.net
>>71
電磁場の波であると考えられている。
電磁場は空間そのものに付随するものと考えられている。

80 :ご冗談でしょう?名無しさん:2014/12/28(日) 22:20:59.71 ID:???.net
光の波動性についてはむしろ量子論に答えを求めた方が正解な気がする。

81 :ご冗談でしょう?名無しさん:2014/12/28(日) 22:48:19.77 ID:HN1wS4Xc.net
>>78
その難しさはどう測る? ていうか、どういう目的での比較?
対象が見えにくいという意味では計算機科学かなと思うが

82 :ご冗談でしょう?名無しさん:2014/12/28(日) 23:34:33.53 ID:???.net
>>81
世界中の人間に、建築学と計算機科学を学ばせて、どっちの方が難しかったかを聞くってのはどうでしょうか?

83 :ご冗談でしょう?名無しさん:2014/12/29(月) 01:05:04.80 ID:???.net
真空ってのは最低エネルギー状態だから電磁場に限らず全ての場があって零点振動してるんだぜ

84 :ご冗談でしょう?名無しさん:2014/12/29(月) 01:48:40.28 ID:???.net
>>75
>>79
>>80
>>83
ありがとう
電磁場がないと光が通らない訳ね
電磁場がないってことはなさそう。ずっと遠い銀河も見えてるし。
電磁波で満ちてるけど壁もすり抜けてるけど、光はすり抜けない。

85 :ご冗談でしょう?名無しさん:2014/12/29(月) 02:05:35.94 ID:???.net
>>84
電磁波と光は同じもの。
電磁波は電磁場の振動。
物理では電磁波のことを単に光と呼ぶことが多い。

86 :ご冗談でしょう?名無しさん:2014/12/29(月) 09:06:14.37 ID:ooUbSv2R.net
>>82
そのままだね。話が広がらんね。
そうやって答えが出て、さて何が分かるの?

87 :ご冗談でしょう?名無しさん:2014/12/29(月) 11:40:55.85 ID:???.net
荒らしに触るなよ

88 :ご冗談でしょう?名無しさん:2014/12/29(月) 14:15:06.00 ID:???.net
空と空間の違いを教えてください。

89 :ご冗談でしょう?名無しさん:2014/12/29(月) 14:15:30.90 ID:???.net
荒らしは去れ

90 :ご冗談でしょう?名無しさん:2014/12/29(月) 14:29:11.16 ID:???.net
いや、荒らしじゃないです。
真面目に、空と空間の違いが知りたいんです。

91 :ご冗談でしょう?名無しさん:2014/12/29(月) 14:32:41.45 ID:???.net
高校で物理を選択してなかったのですが、大学から物理をやりたくなりました
大学から物理をはじめてついていけるものなのでしょうか?
それと(高校物理がぬけてても)理解できるような何かいい教科書ってありますか?
高校の教科書読み返してもいいですけど、せっかくなら微積分使った教科書から
はじめた方がいいと思っています

92 :ご冗談でしょう?名無しさん:2014/12/29(月) 14:44:36.12 ID:???.net
相対論的な正準形式(ハミルトン形式)ってありますか?
相対論だと時空が対等になる一方、正準形式は時間を特別扱いしてますよね?

93 :ご冗談でしょう?名無しさん:2014/12/29(月) 15:31:26.82 ID:???.net
>>85
やっぱそうなのね
でもなんか波そのものってのがしっくり来なくて
海が無いところに懐中電灯付けたら海ができるわけで
海が電磁場か

94 :ご冗談でしょう?名無しさん:2014/12/29(月) 15:58:03.99 ID:???.net
誰か空と空間の違いを教えてください。お願いします。

95 :ご冗談でしょう?名無しさん:2014/12/29(月) 16:06:37.15 ID:???.net
桜卍婆に聞いてみてはどうでしょうか

96 :ご冗談でしょう?名無しさん:2014/12/29(月) 17:58:02.10 ID:???.net
>>91
おれいまだに駿台の物理入門本棚にあるよ

97 :ご冗談でしょう?名無しさん:2014/12/29(月) 20:02:49.59 ID:???.net
http://i.imgur.com/ywdOrrk.jpg
http://i.imgur.com/tL6n1nI.jpg
名古屋市科学館の展示について質問です
ボールを同時に転がした時、ゴールに1番先に着いたのは(3)でビリは(1)でした
(3)は1番距離が大きいのになぜこのような結果になったのでしょう?
また、スタートからゴールまで最速でボールを転がすためにはどのようなコースになりますか?

98 :ご冗談でしょう?名無しさん:2014/12/29(月) 20:06:50.37 ID:???.net
それは最速降下線問題として有名です。
変分という数学の道具を使うと、最速のコースがサイクロイドであることが証明できます。

99 :遅獄先生−主対性卓上理論より実戦:2014/12/29(月) 20:08:41.11 ID:pMuVFJI4.net
空間は障子襖絵みたいなもんで、空は、sky's the limit翼をくださいみたいな曲だな。
絵の具で色を混ぜて、ハモればいいだけ。

100 :ご冗談でしょう?名無しさん:2014/12/29(月) 20:24:38.49 ID:???.net
ゴミを書くな

101 :ご冗談でしょう?名無しさん:2014/12/29(月) 22:01:26.80 ID:JsK4azjq.net
原子の体積が分かったんだが
さらにそこからその原子の大きさ(直径)を求めろという問題が出やがった

102 :ご冗談でしょう?名無しさん:2014/12/29(月) 22:08:50.00 ID:???.net
>>99
真面目に教えてくれ。

103 :ご冗談でしょう?名無しさん:2014/12/29(月) 22:20:09.55 ID:???.net
>>101
原子は球対称だから楽勝じゃん

104 :ご冗談でしょう?名無しさん:2014/12/29(月) 22:35:32.49 ID:???.net
空と真空と空間の違いが分からん。
誰か教えてくれ。

105 :ご冗談でしょう?名無しさん:2014/12/29(月) 22:47:14.11 ID:???.net
>>103
それは実験で検証されているのでしょうか?

106 :ご冗談でしょう?名無しさん:2014/12/29(月) 23:09:12.46 ID:???.net
>>105
理論的に球対称にしかなりえないし
実験的にも球対称でない原子なんか観測されてない

107 :ご冗談でしょう?名無しさん:2014/12/29(月) 23:11:06.44 ID:???.net
>>106
実験のソースは?

108 :ご冗談でしょう?名無しさん:2014/12/29(月) 23:11:45.34 ID:???.net
量子力学的に見たら確率の波でしかない原子の実体って結局何なんだと思う

109 :ご冗談でしょう?名無しさん:2014/12/29(月) 23:37:29.68 ID:???.net
自転車のタイヤが大きいほうが小さい物より一回転するまでに進む距離が長いから速いって聞いたんですけど
タイヤに加える力は変わらないから結局一回転させるまでに掛かる時間が異なりますよね?
なら速度変わらなくね?って思ったんですけど実際のところどうですか?

110 :ご冗談でしょう?名無しさん:2014/12/29(月) 23:51:13.95 ID:???.net
>>108
光子と電磁波の関係と同じ
と分かったふりをしてみる

111 :ご冗談でしょう?名無しさん:2014/12/29(月) 23:52:38.19 ID:???.net
>>108
人間は巨視的な知覚しかできない、電子顕微鏡等の画像も同様、
確率・波動現象(波)とは量子力学現象の巨視的効果のことだろ。

112 :ご冗談でしょう?名無しさん:2014/12/29(月) 23:54:58.95 ID:???.net
いみふ

113 :ご冗談でしょう?名無しさん:2014/12/30(火) 00:04:02.42 ID:???.net
人間が知覚不能の原子現象を確率と波とかマクロ現象に置き換えて理解してるのだよ。

114 :ご冗談でしょう?名無しさん:2014/12/30(火) 00:06:54.72 ID:???.net
いみふ

115 :ご冗談でしょう?名無しさん:2014/12/30(火) 00:10:41.26 ID:???.net
>>109
基本的には君の考えで正しい。
ただトライアルバイクの様なギア比の低いバイクだと、ケイデンス(クランク回転数)が速度の制約となるので、
タイヤ径が大きいほうが速度的には有利になる。
適切なギア比のロードバイクとかに乗ってる場合は、転がり抵抗は径が大きいほうが少し有利だが、空気抵抗的にはやや不利、
700Cと650Cとかなら大した差ではない。

116 :ご冗談でしょう?名無しさん:2014/12/30(火) 00:20:59.78 ID:???.net
>>107
http://io9.com/the-first-image-ever-of-a-hydrogen-atoms-orbital-struc-509684901
まあこれは若干異方性が見えてるけどな

117 :ご冗談でしょう?名無しさん:2014/12/30(火) 00:41:12.59 ID:???.net
>>116
へリーム、ストロンチュームは?

118 :ご冗談でしょう?名無しさん:2014/12/30(火) 00:47:09.59 ID:???.net
めんどくさい自分で探せ

119 :ご冗談でしょう?名無しさん:2014/12/30(火) 00:52:50.96 ID:???.net
水素原子の波動関数の二次元への投影が円に近い

120 :ご冗談でしょう?名無しさん:2014/12/30(火) 01:08:16.54 ID:???.net
>>108
確率の波と考えるよりは測定を大量に行った時
その場所に物理的実体が観測されるという統計的予測を与えるものと考えた方がいいと思う
確率の波というあやふやなものよりははっきりとする

121 :ご冗談でしょう?名無しさん:2014/12/30(火) 01:08:17.65 ID:???.net
英国国教会と皇室が権威で戦ったらどっちが勝つの?

122 :ご冗談でしょう?名無しさん:2014/12/30(火) 01:11:12.25 ID:???.net
日本の首相と英国の首相はどっちの方が偉いのでしょうか?

123 :ご冗談でしょう?名無しさん:2014/12/30(火) 01:12:59.53 ID:???.net
しかし第二量子化はディラック場の量子化なわけでその理屈では電磁波も確率の波なのか?

124 :ご冗談でしょう?名無しさん:2014/12/30(火) 01:13:47.07 ID:???.net
英国の国王と米国の大統領はどっちの方が偉いのでしょうか?

125 :ご冗談でしょう?名無しさん:2014/12/30(火) 01:15:54.26 ID:???.net
英国王室とロスチャイルド家はどっちの方が偉大ですか?

126 :ご冗談でしょう?名無しさん:2014/12/30(火) 01:17:58.24 ID:???.net
ロスチャイルド家とハプスブルク家はどっちの方が偉大ですか?

127 :ご冗談でしょう?名無しさん:2014/12/30(火) 01:22:32.90 ID:???.net
イギリス連邦とアメリカ合衆国が戦争をしたらどっちが勝つの?

128 :ご冗談でしょう?名無しさん:2014/12/30(火) 01:24:05.84 ID:???.net
大英帝国とアッラーはどっちの方が偉大ですか?

129 :ご冗談でしょう?名無しさん:2014/12/30(火) 01:25:33.40 ID:???.net
アイザック・ニュートンとイエス・キリストはどっちの方が偉大ですか?

130 :ご冗談でしょう?名無しさん:2014/12/30(火) 01:52:54.66 ID:???.net
荒らしは去れ

131 :ご冗談でしょう?名無しさん:2014/12/30(火) 02:46:23.62 ID:???.net
無理やり絡んでくるやつというのは相手に振りほどかれる時にこそ快感を覚えるらしい

132 :ご冗談でしょう?名無しさん:2014/12/30(火) 12:27:10.45 ID:???.net
>>123
静電場の確率解釈って出来たっけ?

133 :ご冗談でしょう?名無しさん:2014/12/30(火) 15:01:09.75 ID:???.net
英国王室もドイツ系、ロックフェラー家もドイツ系、ロスチャイルドもドイツ人、ハプスブルク家もドイツ系の家系・・・

ドイツ人凄すぎだろ・・・。

134 :ご冗談でしょう?名無しさん:2014/12/30(火) 15:03:57.95 ID:???.net
イギリス人とドイツ人はどっちの方が優秀ですか?

135 :ご冗談でしょう?名無しさん:2014/12/30(火) 15:11:49.76 ID:???.net
大英帝国と月の表面積はどっちの方が広いですか?

136 :ご冗談でしょう?名無しさん:2014/12/30(火) 16:40:53.15 ID:???.net
小出物理学のこの演習問題について教えてください。
ttp://detail.chiebukuro.yahoo.co.jp/qa/question_detail/q1476673514


知恵袋の回答で
---
(運動エネルギー)=(位置エネルギー)−(摩擦エネルギー)
1/2mv^2 = mgh - Lf
---
とありますが、いきなりLfが出てくるのは何故なんでしょうか。x軸に平行な仕事しかしてないってことなんでしょうか?
傾斜があるならそれはあり得ないんでは…?


自分なりに、「fは摩擦力の大きさ(の平均値)」の意味がわからないのでfを用いずに
μを摩擦係数、φを傾斜として解いた結果

1/2mv^2 = mg(sinφ - μcosφ)*(h^2+L^2)^(1/2)
=mg(h - μL)

v=(2gh-2μgL)^(1/2)

となりました。これではf=μmgということになりますが、
普通摩擦力fは-μmgcosφなんでは…?と、色々混乱しております。助けてください。

137 :ご冗談でしょう?名無しさん:2014/12/30(火) 17:06:54.83 ID:???.net
聞く事を絞らん奴ってなんか疑わしい

138 :ご冗談でしょう?名無しさん:2014/12/30(火) 17:30:40.52 ID:???.net
大英帝国とソ連が戦争をしたらどっちが勝ちますか?

139 :ご冗談でしょう?名無しさん:2014/12/30(火) 18:39:57.77 ID:???.net
荒らしは去れ

140 :ご冗談でしょう?名無しさん:2014/12/30(火) 20:09:27.38 ID:???.net
疑わしいワロタ

141 :ご冗談でしょう?名無しさん:2014/12/30(火) 20:28:17.03 ID:???.net
世界最大の財閥を作りたいのですが、どうすれば良いのでしょうか?

142 :ご冗談でしょう?名無しさん:2014/12/30(火) 20:39:33.04 ID:???.net
疑問に思ったんだが、この世とあの世ってどっちの方が広いの?

143 :ご冗談でしょう?名無しさん:2014/12/30(火) 20:40:16.80 ID:???.net
>>136です。自己解決しました。
Lは坂の横成分でなく斜面自体の長さでした。勘違いでお騒がせして申し訳ありません

144 :ご冗談でしょう?名無しさん:2014/12/31(水) 03:51:22.67 ID:???.net
「微分形式による解析力学 改訂増補版」(木村利栄、菅野礼司 著)
の101ページ、§5-6 シンプレクティク変換で、「φ^*がφの逆写像である」って書かれています。
逆写像なら自明のことになるので、これは間違いだと思うのですが、どうなのでしょう?
引き戻しと逆写像って異なりますよね?

145 :ご冗談でしょう?名無しさん:2014/12/31(水) 12:31:44.40 ID:???.net
孤立系の内部エネルギーは何で常に0では無いのですか?

146 :ご冗談でしょう?名無しさん:2014/12/31(水) 13:01:46.12 ID:???.net
常にゼロである必然性がないからです。

147 :ご冗談でしょう?名無しさん:2014/12/31(水) 13:17:57.02 ID:???.net
必然性…?もう少しわかりやすく教えてくれますか?

148 :ご冗談でしょう?名無しさん:2014/12/31(水) 13:42:04.56 ID:???.net
なんで0だと思うの?

149 :遅獄先生−主対性卓上理論より実戦:2014/12/31(水) 14:33:54.98 ID:XffmOMtN.net
だから、筋肉を使っても、スピードがパワーが落ちるから、
脂肪を使って生きる。と決め込む。男社会
というか男といふもの。
脂肪ロスアンジェネリックゼロ社会の意味だよな。
子育て支援女性に脂肪、時に筋肉、骨スープとか。

150 :ご冗談でしょう?名無しさん:2014/12/31(水) 16:20:49.72 ID:???.net
http://www.int2.info/news1.htm
この人は天才なんですか?

151 :ご冗談でしょう?名無しさん:2014/12/31(水) 16:43:19.96 ID:???.net
物理化学大義に孤立系の内部エネルギーは一定だと書いてありました…

152 :ご冗談でしょう?名無しさん:2014/12/31(水) 16:51:21.99 ID:???.net
一定だと0ですか。

まぁ元々エネルギー相対値にしか意味ないけどね。

153 :ご冗談でしょう?名無しさん:2014/12/31(水) 17:19:59.67 ID:???.net
仮に孤立系のエネルギーが0だったら、ある孤立系にエネルギーを与えても0のまま変わらないとかいう変なことになるだろう。
エネルギーを与える時間以外は孤立系としてね。
あと、熱力学ではエネルギー保存則は、力学と違って原理(実験や観測結果からの経験則)だから。

154 :ご冗談でしょう?名無しさん:2014/12/31(水) 17:54:28.71 ID:???.net
熱電対スレが無かったのでここで
ガスコンロの掃除してて、立ち消え防止装置のセンサーを掃除したら、
点火した時に手を離すと火が消えてしまうのが直った

センサーは熱電対で、その起電力でノーマリークローズの電磁弁を開けて
ガスを流す仕組みになっている

質問はここからで、熱電対ってちょっと汚れたくらいでそんなに感度落ちるものなの?

155 :ご冗談でしょう?名無しさん:2014/12/31(水) 19:11:14.66 ID:???.net
ちょっと起電力が落ちただけでも電磁弁が動かんくなるんじゃないか?

156 :ご冗談でしょう?名無しさん:2014/12/31(水) 22:44:53.89 ID:???.net
>>153
お前の「エネルギーを与える」の定義がわからん。

157 :ご冗談でしょう?名無しさん:2014/12/31(水) 22:51:06.60 ID:???.net
>>156
二つの孤立系があって、片方に熱を加えるとか仕事をすることで、もう片方になるとして、その両方のエネルギーが同じで0だったらおかしいでしょ?

158 :ご冗談でしょう?名無しさん:2014/12/31(水) 23:08:19.22 ID:???.net
>>157
だったら最初からそう書け。
エネルギーを与えてもエネルギーが変わらないって形容矛盾だろ。

んで、仕事できたり熱が与えられたりしたらそれはそもそも「孤立系」なのか?

159 :ご冗談でしょう?名無しさん:2015/01/01(木) 00:30:48.64 ID:???.net
無限の過去から無限の未来までずっと孤立した系でなければ孤立系とは言わないってこと?
ある時間外界とエネルギーをやりとりしなければ、その時間においては孤立系というと思うが。

160 :ご冗談でしょう?名無しさん:2015/01/01(木) 00:35:43.39 ID:???.net
熱電対の起電力なんて温度だけで決まってるだろうに、
表面のちょっとした汚れで温度がそもそも変わるのかと

ガスが燃えてるか消えてるかのon/offだから、
閾値なんか相当粗く取っても大丈夫だろうに、何故汚れたくらいで誤動作する

161 :ご冗談でしょう?名無しさん:2015/01/01(木) 00:59:02.61 ID:???.net
宇宙の外側の空間は無限ですか?

162 :ご冗談でしょう?名無しさん:2015/01/01(木) 01:13:06.87 ID:???.net
>>160
君が最初に思ったであろう通り、温度が変わってるからさ。
表面のちょっとした汚れで温度は段違いに変わる。
代表的な物質の熱抵抗率を調べてみるといい。
180℃に熱した油の入った鍋が上においてあっても
誤動作しないためのマージンは必要なんだから
ちょっとの汚れでもわりと致命的なのさ。

163 :ご冗談でしょう?名無しさん:2015/01/01(木) 01:59:17.39 ID:???.net
物理学者と数学者と哲学者

頭がいい順を教えてください。

164 :ご冗談でしょう?名無しさん:2015/01/01(木) 03:09:35.74 ID:???.net
物理学とは、実験と理論を両輪に、我々を取りかこむ森羅万象に潜む法則を一つ一つ解き明かしていく学問です。
虹はなぜ7色か、雪の結晶はなぜあのように美しい形となるかなどの身近な疑問から、
宇宙はどのようにしてでき、どうなってゆくのか。それを認識する我々という生命はどうやってできたのかなど、
誰でも一度は疑問に思ったことのあるような根源的な問題まで、科学は実験や観測を通して一つ一つ解き明かし、
人類の知を蓄積してきました。その中でも物理学は、自然現象を極限まで単純化することで、
問いと答えを正しさが検証できるところまで引きつけ、数学と観測を手段として自然に問いかけることで、理解を深めてきました。
中でも、物質の成り立ちを求めて、物質を極限まで分けて理解しようとする要素還元論的な手法は大きな成功を収めてきました。
物質のミクロな姿を究極まで突きつめることで、拓かれてきた素粒子の研究は、
質量の起源にも到達し、ダークマターなど未知の粒子にも迫ろうとしています。
そうやって追いつめてきたミクロの極限である素粒子の世界が、宇宙の始まりの高エネルギー状態の理解につながることもまた大きな驚きです。
まさに、一周して自分自身の尻尾を飲み込むウロボロスの蛇の図を思い起こさせる、
ミクロとマクロがつながる構造がそこにあります。
一方、物理の世界はそれ以外の方向にも広がっていることも忘れてはなりません。
要素還元的な物理の方法を、自然を掘り下げて理解する物理学の縦糸とすると、
異なる現象の関係をつなぐ横糸のような物理もまた重要です。例えば、熱力学や統計力学は、
物質やスケールの違いを超えて様々の系に適用できる物理学の強力な方法です。

165 :ご冗談でしょう?名無しさん:2015/01/01(木) 03:10:14.45 ID:???.net
運動、熱、波動、対称性などの概念は、対象を”もの”ではなく”こと”として捉える概念であり、物理学に不可欠な横糸とも言うことができます。
物性物理学は、物質が凝縮して集団となったとき、個々の原子の性質をはるかに超えた新たな性質を持ちうることを明らかにしてきました。
そして、自然には存在しない物質さえ作り出し、現代社会をその根底から支えています。また、非平衡物理や生物物理、量子情報などの比較的新しい分野では、
横糸としての役割を発揮して、従来の物理学が対象としてきた範囲をさらに拡張して、生命現象や情報の世界にも迫ろうとしています。
このように、物理学のロジックは縦横に網目のように絡み合いながら自然の理解を少しずつ広げ、
また社会にも影響を及ぼしています。その意味で現在は、物理学自体も進歩を遂げ、新しい物理学の姿が見え始めてきている時代と言っても過言ではないでしょう。
物理学は、その深さと広がりにおいて、真に学びがいのある、そして学んだことが確実に皆さんの将来の力となる学問です。

166 :ご冗談でしょう?名無しさん:2015/01/01(木) 03:20:22.20 ID:???.net
汚れと熱電対が化学反応起こして電池みたいになってんじゃね?

167 :ご冗談でしょう?名無しさん:2015/01/01(木) 08:48:34.99 ID:???.net
そもそも熱電対の起電力(mVオーダー)で稼働する電磁弁って…。

168 :ご冗談でしょう?名無しさん:2015/01/01(木) 09:08:08.48 ID:???.net
電圧よりは電流だと思う
そんな弱々しい弁で本当にガスの圧力に勝てるのかと
まあ、実際に機能してるんだけど

169 :ご冗談でしょう?名無しさん:2015/01/01(木) 10:13:02.36 ID:???.net
mVで動く電磁弁なんてノイズで誤作動しまくりだよ。

ソレノイドの規格はACで100Vか200V(モノによっては220V)だし、
DCだって12Vか24Vですよ。

170 :ご冗談でしょう?名無しさん:2015/01/01(木) 15:42:54.83 ID:???.net
質問です。
1kw/h=3.6MJ
灯油1Lの熱量=3.67MJ
ということなのですが

実際に600Wの電気ファンヒーターと、0.66kw/h(0.64ml/h)の石油ファンヒーターで
部屋を保温すると石油ファンヒーターの方が温かいです。
24時間保温による室温安定後、2時間づつ使用した結果です。
外の状態ですが、外気温は石油ファンヒーターのときの方が3度ほど低く、風の状態は
たぶん似たようなものだと思います。

湿度の違いによるものではなく、温度計で見ても600Wの方は約0.5度下がりました。
ほとんど同じ熱量を排出しているのになぜ体感できるほどの差が生まれるのでしょうか?
素人が適当にやったから勘違いや間違いによる結果でしょうか?

171 :ご冗談でしょう?名無しさん:2015/01/01(木) 15:57:36.29 ID:???.net
1行目から間違ってる

172 :ご冗談でしょう?名無しさん:2015/01/01(木) 16:02:38.42 ID:???.net
>>170
kw/hって何?

173 :ご冗談でしょう?名無しさん:2015/01/01(木) 16:52:02.79 ID:???.net
>>170
石油ファンヒーターは最弱のとき値ですが、まさか途中で強くなったりはしてないでしょうね

174 :ご冗談でしょう?名無しさん:2015/01/01(木) 16:54:54.34 ID:???.net
>>173
火力固定できる機種ですのでそれはないです

175 :ご冗談でしょう?名無しさん:2015/01/01(木) 17:25:35.22 ID:???.net
外気と室温との温度差は発生する熱量に比例するはずです。
600Wと660Wが正しければ、温度差も10%異なることになるのでは。

176 :170:2015/01/01(木) 17:37:46.86 ID:???.net
ありがとうございます。
素人実験の誤差と考えるべきかもしれませんね。

177 :ご冗談でしょう?名無しさん:2015/01/01(木) 18:16:17.86 ID:???.net
統計力学の最初の部分なんですが
M個の量子状態をN個の粒子に分配する場合の数が
W_N(M)=(M+N-1)!/((N-1!M!)であるのはいいのですが,
M-x個の量子状態をN-1個の粒子に分配する場合の数のxの0からMの総和
\sum^M_{i=0} W_(N-1)(M-x)=W_N(M)
になることが証明できません. どのようにしたらよいのでしょうか
とりあえず数式をずらっと書き出してみたのですが, 和の部分をどう処理すればいいのかで
手づまりです

178 :ご冗談でしょう?名無しさん:2015/01/01(木) 19:03:32.88 ID:???.net
東京大学理学部物理学科に進学する場合、どの程度のパソコンのスキルが必要になってきますか?

179 :ご冗談でしょう?名無しさん:2015/01/01(木) 19:12:00.15 ID:???.net
>>177
2項係数だから
W_N(M)=W_N(M-1)+W_{N-1}(M)
W_N(M-1)=W_N(M-2)+W_{N-1}(M-1)
    :
以上の最右項を足す

180 :ご冗談でしょう?名無しさん:2015/01/01(木) 19:29:09.03 ID:???.net
>>179
ありがとうございます.
言われてみればそもそも高校数学の基礎でした・・・
わざわざ書き出していた自分が恥ずかしい・・・
ありがとうございました

181 :ご冗談でしょう?名無しさん:2015/01/02(金) 22:05:44.26 ID:???.net
1月4日に地球の重力が軽くなる噂があるそうです
そこで実験をしてみようと思うんですが
はかりに1kgの物を載せてwebカメラで撮影した場合
地球の重力に変化があったら
はかりの表示する重さに変化はあるんでしょうか?

182 :ご冗談でしょう?名無しさん:2015/01/02(金) 22:35:28.79 ID:???.net
ttp://www.police.pref.hyogo.jp/sonota/zattou.htm
「将棋倒し」のニュースを見て思ったんですが、
個々人が出す大したこと無くて、むしろPassiveだと思うんですが、
人が大勢死ぬほどのダメージが出ることがあるのはどういう力学なんでしょう?


報道や資料では群集心理的な要因ばかりがクローズアップされますが、
建築設計では人が集まりやすい場所でのリスクなど重要だと思います。
物理の方でトライボロジや粉粒体(渋滞学とか?)の知見から、
研究事例等ありますでしょうか?

183 :ご冗談でしょう?名無しさん:2015/01/02(金) 22:42:15.85 ID:???.net
>>181
オカルトっぽい話だが、答えはyes、ただし天秤ばかりだとダメだ。

184 :ご冗談でしょう?名無しさん:2015/01/02(金) 22:47:30.36 ID:???.net
1月4日は地球が太陽から最も遠くなる日だが、重力とは関係ないな。
「実験」ではなく「観測」だろうね。重力は「大きい」か「小さい」。
重力の変化があれば、0.1%くらいならばバネばかりで見ることはできるだろう。
ただ、温度による指示値の変化や、気圧による浮力の変化もある。

185 :ご冗談でしょう?名無しさん:2015/01/02(金) 22:59:38.28 ID:???.net
>>182
西成活裕の「渋滞学」の「第3章 人の渋滞」などどうでしょうか

186 :ご冗談でしょう?名無しさん:2015/01/02(金) 23:51:39.46 ID:???.net
>>181
地球が太陽から受ける重力が最も小さくなる日を勘違いして
地球が人間等々に及ぼす重力が小さくなる日だと思ったんじゃないの?
ちなみに太陽の重力の変動を測ろうと思ったら誤差0.0001%程度の観測を
毎回正午か真夜中に少なくとも三ヶ月程度は継続して行わないと観測できないよ。

187 :ご冗談でしょう?名無しさん:2015/01/03(土) 00:27:41.28 ID:???.net
0.0001%はどっから出てきた

188 :ご冗談でしょう?名無しさん:2015/01/03(土) 00:34:03.81 ID:???.net
>>187
地球の重力に対して太陽の重力は大体0.06%、
地球の離心率は0.017なので
この変動を検知するには0.0001%程度の精度がいるよね?
ざっくりだけど。

189 :ご冗談でしょう?名無しさん:2015/01/03(土) 00:43:06.94 ID:???.net
0.06%はどっから出てきた

190 :ご冗談でしょう?名無しさん:2015/01/03(土) 00:53:56.49 ID:???.net
>>189
Google先生に教えてもらいました。
ttp://www.google.com/search?q=%E5%A4%AA%E9%99%BD%E3%81%AE%E8%B3%AA%E9%87%8F/%E5%9C%B0%E7%90%83%E3%81%AE%E8%B3%AA%E9%87%8F/%E5%A4%A9%E6%96%87%E5%8D%98%E4%BD%8D%5E2*%E5%9C%B0%E7%90%83%E3%81%AE%E5%8D%8A%E5%BE%84%5E2*100

191 :ご冗談でしょう?名無しさん:2015/01/03(土) 00:56:49.60 ID:???.net
やっぱり何も考えてなかったか
太陽から受ける万有引力から公転による慣性力を差し引けよ

192 :ご冗談でしょう?名無しさん:2015/01/03(土) 01:09:04.37 ID:???.net
>>191
公転による向心加速度は
ttp://www.google.com/search?q=2pi*%E5%A4%A9%E6%96%87%E5%8D%98%E4%BD%8D/1%E5%B9%B4%5E2/%E9%87%8D%E5%8A%9B%E5%8A%A0%E9%80%9F%E5%BA%A6*100
で0.01%程度なので無視するにはでかいですね。
思ったよりでかかった。
ご指摘どうもでした。

193 :ご冗談でしょう?名無しさん:2015/01/03(土) 01:12:26.95 ID:???.net
そうじゃねーよ
太陽に因る重力は潮汐力としてしか働かないと言ってるんだ

194 :ご冗談でしょう?名無しさん:2015/01/03(土) 01:13:25.12 ID:???.net
あ、しまった、
ttp://www.google.co.jp/search?q=4pi^2*%E5%A4%A9%E6%96%87%E5%8D%98%E4%BD%8D/1%E5%B9%B4%5E2/%E9%87%8D%E5%8A%9B%E5%8A%A0%E9%80%9F%E5%BA%A6*100
で0.06%程度ですね。
思ったより自分はバカだった。
ご指摘どうもでした。

195 :ご冗談でしょう?名無しさん:2015/01/03(土) 01:14:46.50 ID:???.net
いやまあそういうことでもあるか

196 :ご冗談でしょう?名無しさん:2015/01/03(土) 01:28:23.03 ID:???.net
>>186
指摘を踏まえて考え直すと、
重力はR^-2で効いて、向心加速度というか遠心力はR^1で効くので、
さっぴいて0.06%*(-2+1)*0.017=-0.00102%
となり、有効数字を一桁確保して0.0001%程度の精度が必要。
という思考経路がバカで結果だけあってるやつだったわけですね。
工学頭で物理板に突っ込んじゃダメだな。

197 :ご冗談でしょう?名無しさん:2015/01/03(土) 01:33:01.63 ID:???.net
計算に使われた定数のせいでそうなっていないだけで
地球の重心に働く万有引力とその静止系で働く遠心力とは厳密にキャンセルする

198 :ご冗談でしょう?名無しさん:2015/01/03(土) 01:46:50.43 ID:???.net
>>197
地球中心で厳密にキャンセルするのは真円軌道の時のみでは?
離心率があることを前提にしているのだからその近似は使えないと思うのですが

199 :ご冗談でしょう?名無しさん:2015/01/03(土) 02:15:36.35 ID:???.net
太陽の静止系で円軌道でなかったとして
動径方向の加速度(向心加速度)しか考えないなら近似だが
楕円軌道だろうと円軌道だろうと双曲線軌道だろうと自由落下しているなら厳密にゼロ

200 :ご冗談でしょう?名無しさん:2015/01/03(土) 03:02:22.53 ID:???.net
>>199
ああ、遠心力という言葉から、動径方向のみを考えてしまっていましたが、
慣性力を全部考えるならその通りですね。

201 :182:2015/01/03(土) 03:17:59.28 ID:???.net
>>185
ありがとうございます
早速チェックしてみます

202 :ご冗談でしょう?名無しさん:2015/01/03(土) 13:09:15.04 ID:???.net
月や太陽の重力なら大潮・小潮として常に見れるだろ

203 :ご冗談でしょう?名無しさん:2015/01/03(土) 18:23:28.78 ID:???.net
>>20
黒体放射じゃなく灯油の炎の話でしょうに。

204 :ご冗談でしょう?名無しさん:2015/01/03(土) 18:28:23.86 ID:???.net
青い炎は特定スペクトルだが
赤い炎はスス粒子の黒体放射

205 :ご冗談でしょう?名無しさん:2015/01/03(土) 18:38:11.50 ID:???.net
灯油の青い炎の話じゃないの

206 :ご冗談でしょう?名無しさん:2015/01/03(土) 23:33:48.09 ID:???.net
電磁気学の鏡像法に関する質問です。

qa,qb,qcという正の点電荷が等間隔(間隔a)にあり、
aとbの中点に導体の平板がある状況で
qaに働くクーロン力を求めよ、という問題があります。

この問題を解くとき
導体があるので、鏡像の点電荷として-qa,-qb,-qcの電荷を考え、
qaに働くクーロン力は
qaと-qaの間に働くクーロン力
qaとqb     〃
qaとqc 〃
qaと-qc    〃
これらの和として考えれば良いのでしょうか?

質問が長くなり、分かりにくくなりましたがよろしくお願いします。

207 :ご冗談でしょう?名無しさん:2015/01/04(日) 00:05:45.49 ID:???.net
>>206
-qbが抜けてるがそれを直せば大丈夫

208 :ご冗談でしょう?名無しさん:2015/01/04(日) 00:10:36.66 ID:???.net
>>206
あ、aとb,cを隔てるように導体板があるのか。
なら-qaだけです。

209 :ご冗談でしょう?名無しさん:2015/01/04(日) 01:49:09.90 ID:???.net
qa側の領域の電場求めるなら鏡像電荷をqa側に置いたらイカんよ

210 :ご冗談でしょう?名無しさん:2015/01/04(日) 22:23:45.89 ID:???.net
>>208 >>209
ありがとうございます!
aとb,cが隔ててあるのでqaと-qaでクーロン力を求めるのは分かりましたが、
もしもqbのクーロン力を求めるならば
bとcが同じ領域にあるので、
-qb,qc,-qc
この3つのクーロン力の和として考えれば良いんですかね?

211 :ご冗談でしょう?名無しさん:2015/01/05(月) 01:49:27.55 ID:???.net
>>210
細かいとこつっこむとqaとqaは相互作用しないけどそういうことです。

212 :ご冗談でしょう?名無しさん:2015/01/05(月) 07:57:29.97 ID:???.net
無になりたいのですが、自殺しても無にはなれませんか?

213 :ご冗談でしょう?名無しさん:2015/01/05(月) 11:34:49.25 ID:???.net
荒らしは去れ

214 :ご冗談でしょう?名無しさん:2015/01/05(月) 13:51:43.65 ID:???.net
絶縁体と酸化物は似ているものだと講義で言ってたのですが、
どういう意味ですか。

215 :ご冗談でしょう?名無しさん:2015/01/05(月) 14:22:05.27 ID:???.net
どこの講義だ、バラエティ番組か?

216 :ご冗談でしょう?名無しさん:2015/01/05(月) 14:40:39.41 ID:???.net
mos構造mis構造云々の部分だったと思います

217 :ご冗談でしょう?名無しさん:2015/01/05(月) 16:44:15.03 ID:???.net
その講義で言ってた意味でしかないな
聞いてない人には関係ない

218 :ご冗談でしょう?名無しさん:2015/01/05(月) 19:24:55.34 ID:???.net
風船が超高速(音速以上)でぶつかってもダメージにはならないんでしょうか?
またその時風船は割れますか?

219 :ご冗談でしょう?名無しさん:2015/01/05(月) 19:49:45.10 ID:???.net
ダメージを物理的に定義してから来い

220 :ご冗談でしょう?名無しさん:2015/01/05(月) 20:48:07.38 ID:???.net
いかなる考え方をしても揺るぎないものは何ですか?

221 :ご冗談でしょう?名無しさん:2015/01/06(火) 00:55:04.38 ID:???.net
日本とイギリスってどっちの方が自然科学&形式科学の水準が高いですか?

222 :ご冗談でしょう?名無しさん:2015/01/06(火) 02:04:53.08 ID:???.net
荒らしは去れ

223 :ご冗談でしょう?名無しさん:2015/01/06(火) 06:20:39.43 ID:???.net
理論物理学者と数学者の数学の平均レベルについて教えてください。

数学の理論(級数の収束や関数の連続性みたいなやつ)はもちろん数学者の方が出来ると思います。
しかし、計算力となればどちらが出来るのでしょうか?
数学者に負けないぐらいの難しい計算は理論物理学者もしてると思うのです。

224 :ご冗談でしょう?名無しさん:2015/01/06(火) 12:55:36.52 ID:???.net
オマエには無意味

225 :ご冗談でしょう?名無しさん:2015/01/06(火) 16:17:25.39 ID:???.net
光速で放屁したらどうなりますか?

226 :ご冗談でしょう?名無しさん:2015/01/06(火) 17:30:30.83 ID:???.net
>>225

宇宙がヤバイ(例のコピペ

227 :ご冗談でしょう?名無しさん:2015/01/06(火) 21:37:55.29 ID:???.net
サイトによってラプラス変換の定義式の積分範囲が-∞→∞の時と0→∞の時があるんですけど、計算結果変わりますよね?
-∞→∞は間違ってますよね?

228 :ご冗談でしょう?名無しさん:2015/01/06(火) 23:38:45.55 ID:???.net
>>227
どちらも間違ってはいない。
両側ラプラス変換と片側ラプラス変換という別のものを
どちらもラプラス変換と省略して呼んでるだけ。

229 :ご冗談でしょう?名無しさん:2015/01/07(水) 15:48:09.00 ID:???.net
大体の三角関数の積分は留数定理で解けますよね?

230 :ご冗談でしょう?名無しさん:2015/01/07(水) 16:04:12.37 ID:???.net
邪王真眼と同じく留数定理は最強。
ただし使いこなすには特異点と不可視境界線を見つける必要がある。
一般人には困難。

231 :ご冗談でしょう?名無しさん:2015/01/07(水) 17:12:58.78 ID:???.net
留数定理で解ける問題しか出さないなら

232 :ご冗談でしょう?名無しさん:2015/01/07(水) 17:33:15.04 ID:???.net
>>229
マルチ

分からない問題はここに書いてね397 [転載禁止](c)2ch.net
http://wc2014.2ch.net/test/read.cgi/math/1418822341/618

233 :ご冗談でしょう?名無しさん:2015/01/07(水) 18:21:16.24 ID:???.net
>>232
そのスレで回答が二択出てきたのでこちらでも質問させていただきました

234 :ご冗談でしょう?名無しさん:2015/01/07(水) 19:20:41.28 ID:???.net
バカ質問はそうなる

235 :ご冗談でしょう?名無しさん:2015/01/07(水) 22:25:21.04 ID:???.net
イギリスとフランスは世界的に見てどっちの方が有名ですか?

236 :ご冗談でしょう?名無しさん:2015/01/07(水) 22:28:45.00 ID:???.net
マルチポストしていい理由なんてありゃせん。

237 :ご冗談でしょう?名無しさん:2015/01/08(木) 23:50:33.07 ID:???.net
留数定理が最強ってのは納得できる
f(z)=1/(e^z+1)
って関数(フェルミ分布関数)はz=iπ(2n+1)で特異点を持つ(nは整数)
これを逆に利用して、nに関する離散的な和を積分に直すことが出来る。
有限温度における場の量子論ではこういうテクニックをよく使う。
つまり、小鳥遊六花たそ(^ω^)ペロペロ

238 :ご冗談でしょう?名無しさん:2015/01/08(木) 23:54:42.87 ID:???.net
なにを今さら

239 :ご冗談でしょう?名無しさん:2015/01/09(金) 00:34:16.14 ID:???.net
回転は、平面(2次元)と回転軸(1次元)+時間ですが、
4次元空間+時間になると、
立体(3次元)と回転軸(1次元)+時間の他に、
平面(2次元)と回転面(2次元)+時間のような回転も
ありうるのでしょうか?

考えてもわかりませんでした

240 :ご冗談でしょう?名無しさん:2015/01/09(金) 00:43:44.14 ID:???.net
>>239
4次元空間+時間、は3次元空間+時間=4次元時空の間違いだよね。
それなら相対性理論でいうローレンツ変換は「回転」そのものではないけれど
空間と時間を回転と似たルールの下で混ぜる変換だよ。

241 :ご冗談でしょう?名無しさん:2015/01/09(金) 03:36:43.04 ID:???.net
よくテレパスできるな
何言ってるのか分からんかった

242 :ご冗談でしょう?名無しさん:2015/01/09(金) 04:17:43.68 ID:???.net
天才デイトレーダーと天才物理学者はどっちの方がIQが高いですか?

243 :ご冗談でしょう?名無しさん:2015/01/09(金) 07:46:10.58 ID:???.net
Hの定理ってなんですか?
Hな定理なんですか?

244 :ご冗談でしょう?名無しさん:2015/01/09(金) 09:40:14.97 ID:???.net
本当に4次元の回転というか、4次元以上の回転の話なら、回転軸という考え方は使えない。3次元空間が特殊で、回転軸という概念が存在する。
例えばxy平面で回転させるとして、3次元ならそれに直交するのはz軸方向だけなので、それを回転軸という考えられる。
しかし、4次元以上なら、z軸以外にもxy平面にもz軸にも直交する軸があるから、回転軸というのは定義できず、回転面でしか考えられない。

こういう3次元空間+時間の特殊性を考えると、次元の数というのは理論的に決まるんじゃないかと思えるよね。これ以外にも5次元以上の時空なら理論は簡単になるのに、4次元だと途端に難しくなるということがあるし。

245 :ご冗談でしょう?名無しさん:2015/01/09(金) 09:54:49.81 ID:???.net
何を言ってるんだ

246 :ご冗談でしょう?名無しさん:2015/01/09(金) 12:46:34.57 ID:???.net
>>244
一般の4次元回転で固定される方向は無いから、回転軸に相当する回転面などない
固定される方向があって回転軸があるのは奇数次元

247 :ご冗談でしょう?名無しさん:2015/01/09(金) 13:02:25.11 ID:???.net
>>246
2次元以上なら何次元だろうとxy平面での原点を中心とした回転があるでしょ?
まず2次元空間では軸なんてないし、4次元以上ならその回転軸なんて奇数次元でも定義できないと思うけど

248 :ご冗談でしょう?名無しさん:2015/01/09(金) 13:07:35.17 ID:???.net
例えば直交座標で(x,y,1,2,3,4,5,6)という座標があったとして、
(x',y',1,2,3,4,5,6)=(xcosA - ysinA, xsinA + ycosA,1,2,3,4,5,6)
という変換は回転変換でしょ?

249 :ご冗談でしょう?名無しさん:2015/01/09(金) 14:35:37.80 ID:???.net
最初に書いてある「一般の」が見えんかったのか?
特殊例なら何でもありに決まってる
回転角ゼロなら全部あるぞ

250 :ご冗談でしょう?名無しさん:2015/01/09(金) 16:16:07.06 ID:???.net
回転はdet=1の実直交行列(実特殊直交行列)で表わされる
回転は何度繰り返しても発散も退化もしない事から固有値の絶対値は1
固有多項式が実多項式であるから固有値は絶対値1の共役複素数のペアか実数±1となるが
det=1であるから実固有値があれば+1である
奇数次元なら共役複素数ペアから外れる実固有値があるので固有値1が存在する
固有値1に対応する固有ベクトルは不変なので回転軸となる

251 :ご冗談でしょう?名無しさん:2015/01/09(金) 16:20:04.75 ID:???.net
おっと、実固有値−1が偶数個ある場合もあるか
奇数次元なら+1が残るから結論は同じだが

252 :ご冗談でしょう?名無しさん:2015/01/09(金) 19:23:04.05 ID:???.net
俺はso(n)のリー代数で与えられる変換を回転と考えてたんだけれど、そういうのもあるのか
基本的にd次元のポアンカレ群の部分群としてしか回転群を勉強したことしかないから初めて見たわ

253 :ご冗談でしょう?名無しさん:2015/01/10(土) 09:41:47.84 ID:???.net
単純に複数の平面で回転した時、それを一つの平面の回転と見なすことができる場合は特殊で、
偶数次元なら全てのベクトルが変換することもあるし、奇数次元ならあるベクトルだけは変換されないことがあるということか。
5次元以上で「回転軸」というのは3次元のそれから拡張された概念で、3次元でのイメージとはだいぶ違うね。
3次元だと1バラメータでの連続変換だけど、5次元以上だと複数パラメータによる連続変換だし。

254 :ご冗談でしょう?名無しさん:2015/01/10(土) 11:34:02.11 ID:???.net
ハーバード大学とケンブリッジ大学はどっちの方がレベルが高いですか?

255 :ご冗談でしょう?名無しさん:2015/01/10(土) 11:37:33.04 ID:???.net
ソフス・リーさんとロスチャイルドさんはどっちの方が天才であると言えますか?

256 :ご冗談でしょう?名無しさん:2015/01/10(土) 23:53:15.55 ID:???.net
2つの原子核と2電子のハミルトニアンに対するシュレディンガー方程式の解を
Slater行列式で表して、Hartree Fock法(をローターン法)で解いた場合、

f(x1,x2)=(1/√2)det|φ1(x1) φ2(x2)|
というエネルギー固有状態があったとします。

これに直交している別のエネルギー固有状態:
g(x1,x2)=(1/√2)det|φa(x1) φb(x2)|
についてですが、

厳密に φa=φ2 となるような(f とは直交する)固有状態 g は一般的に存在して
いますか?

2つの原子核が十分離れている場合、近似的にφa≒φ2 となる固有状態が存在
しているとは思うのですが、一般的にはどうでしょうか?
2つの原子核が近い場合には存在していないのが普通ですか?

257 :ご冗談でしょう?名無しさん:2015/01/11(日) 00:34:01.13 ID:???.net
>>256
スピンをどう扱うつもりさ

258 :ご冗談でしょう?名無しさん:2015/01/11(日) 01:16:39.15 ID:???.net
>>256
スピン自由度のことなんかまったく考えてなかったんだろうが、
RHF法で4電子系を解くものとするとそんなものは存在しない。

259 :256:2015/01/11(日) 01:52:03.42 ID:???.net
元々、1つの原子核に1つの電子に対応するハミルトニアンの固有状態を
{φi}とした場合、2電子系でも相互作用が弱い場合、

ψ12(x1,x2)=(1/√2)det|φ1(x1) φ2(x2)|
ψ23(x1,x2)=(1/√2)det|φ2(x1) φ3(x2)|
ψ34(x1,x2)=(1/√2)det|φ3(x1) φ4(x2)|
ψ45(x1,x2)=(1/√2)det|φ4(x1) φ5(x2)|
・・・

が近似的には、全て2電子の場合の固有状態にとても近いということだと
思ったんです。

相互作用が弱くない場合は、そういうことは言えないのですね?

260 :ご冗談でしょう?名無しさん:2015/01/11(日) 02:44:27.53 ID:???.net
>>259
仏教学と数学はどっちの方が難しいのでしょうか?

261 :ご冗談でしょう?名無しさん:2015/01/11(日) 03:36:16.23 ID:???.net
ガウスの法則が球面で成り立つのはわかるのですが任意の閉曲面で成り立つのは何故なんですか?

262 :ご冗談でしょう?名無しさん:2015/01/11(日) 04:12:45.38 ID:???.net
入れ物の形を変えても中身は変わらない。

263 :ご冗談でしょう?名無しさん:2015/01/11(日) 04:26:18.26 ID:???.net
そもそもなぜ「形」が存在するのか?
そもそもなぜ「大きさ」が存在するのか?
そもそもなぜ「色」が存在するのか?
そもぞもなぜ「音」が存在するのか?
      ・
      ・
      ・

科学は「なぜ」には答えられないのでしょうか?

264 :ご冗談でしょう?名無しさん:2015/01/11(日) 04:30:23.01 ID:???.net
>>261
たとえ、原点から外へ貫いた磁束がもう一度中に入って来るように変な閉曲面をとっても、最終的には外へ出てゆく。
同じように、内側へ貫いた磁束がもう一度外へ出て行っても、最終的には閉曲面の内側に入ってくる。
だから閉曲面がぐにゃぐにゃに曲がってても、貫く磁束は変わらない。

265 :ご冗談でしょう?名無しさん:2015/01/11(日) 07:31:14.39 ID:???.net
宇宙飛行士とお坊さんはどっちの方が凄いですか?

266 :ご冗談でしょう?名無しさん:2015/01/11(日) 08:08:08.33 ID:???.net
ニュートンとムハンマドはどっちの方が偉大ですか?

267 :ご冗談でしょう?名無しさん:2015/01/11(日) 09:27:39.77 ID:???.net
宇宙は膨張してると言われていますが、とすると仮に膨張の限界があるとして
今度は収縮したら時間は逆行して宇宙が最小の状態になり、再びビッグバンが
発生して宇宙ができるのでしょうか?

268 :ご冗談でしょう?名無しさん:2015/01/11(日) 10:13:56.75 ID:???.net
空(くう)と空間は同じですか?

269 :ご冗談でしょう?名無しさん:2015/01/11(日) 10:26:06.24 ID:???.net
>>267
ビッグクランチの考え方だね。
重力場方程式の宇宙定数の値によってそのまま膨張し続ける解とどこかで収縮に転じる解がある。
でも観測によると、前者が正しいみたいで、宇宙膨張の膨張は加速している。
引力で引き合って膨張は減速していると考えていた天文学者はこの観測結果にビビったらしい。

このままいけば100兆年後には水素が枯渇して恒星がなくなり、ブラックホールばかりになる。
そのブラックホールもずっと遠い未来に蒸発して、宇宙にはほんとに何も存在しなくなる。

270 :ご冗談でしょう?名無しさん:2015/01/11(日) 10:44:19.32 ID:???.net
空(くう)と空間の違いが分かりません。
誰か教えてください。

271 :ご冗談でしょう?名無しさん:2015/01/11(日) 11:39:31.99 ID:???.net
荒らしは去れ

272 :ご冗談でしょう?名無しさん:2015/01/11(日) 12:09:04.96 ID:???.net
曲率と変化率って同じですよね
何故名前が違うのですか

273 :ご冗談でしょう?名無しさん:2015/01/11(日) 12:13:19.51 ID:???.net
>>272
同じじゃないから。

274 :ご冗談でしょう?名無しさん:2015/01/11(日) 13:16:55.41 ID:???.net
荒らしじゃなくて真面目な質問です。

空(くう)と空間の違いを教えてください。

275 :ご冗談でしょう?名無しさん:2015/01/11(日) 13:46:56.67 ID:???.net
>>274
死ねば解るんじゃない?

276 :ご冗談でしょう?名無しさん:2015/01/11(日) 19:21:12.97 ID:???.net
量子力学を応用した、未来のマーケティングシステム「Scanamind」
http://wired.jp/2014/12/15/scanamind/

なんで量子力学がマーケティングに役立つのかさっぱりわかりません。どういうことでしょうか?

277 :ご冗談でしょう?名無しさん:2015/01/11(日) 19:30:03.74 ID:???.net
科学用語でこけおどしって良くある奴だな

278 :ご冗談でしょう?名無しさん:2015/01/11(日) 20:10:26.35 ID:???.net
量子力学を使って因果律を崩壊させる……

279 :ご冗談でしょう?名無しさん:2015/01/11(日) 21:55:18.56 ID:???.net
単に線型代数の問題を解いてるようにしか読み取れない
シュレディンガー方程式とか使ってないだろ

280 :ご冗談でしょう?名無しさん:2015/01/11(日) 22:01:02.22 ID:???.net
電報道に頼めよ

281 :ご冗談でしょう?名無しさん:2015/01/12(月) 00:03:54.31 ID:???.net
典型的な「知の欺瞞」

282 :ご冗談でしょう?名無しさん:2015/01/12(月) 00:31:20.86 ID:???.net
divB=0という式は自然観測的に導かれたもので数学的な証明とかはないんですか?
あったら教えてくださいお願いいたします

283 :ご冗談でしょう?名無しさん:2015/01/12(月) 01:03:48.24 ID:???.net
「マクスウェル方程式より上位の原理はありますか?」という質問だと思いますが、「ありません」

284 :ご冗談でしょう?名無しさん:2015/01/12(月) 01:05:06.68 ID:???.net
その意味ならあるだろ

285 :ご冗談でしょう?名無しさん:2015/01/12(月) 01:51:20.69 ID:???.net
作用原理や量子論を除けばないでしょ

286 :ご冗談でしょう?名無しさん:2015/01/12(月) 01:55:10.46 ID:???.net
>>282
その先になるとゲージ理論になってくる。
マックスウェル方程式は、U(1)ゲージ変換に対して物理が不変であることを要請すると出てくる。
しかし、それがなぜかはわからん。

287 :ご冗談でしょう?名無しさん:2015/01/12(月) 03:03:41.82 ID:???.net
>>285
なぜ除く

288 :ご冗談でしょう?名無しさん:2015/01/12(月) 03:07:39.85 ID:???.net
今の量子論て古典物理を量子化するモデルで不自然」

289 :ご冗談でしょう?名無しさん:2015/01/12(月) 03:28:51.71 ID:???.net
モノポールって未だに発見されてないですけど
あるものを観測するんじゃなくて人為的に作ることって出来ないんですかね

290 :ご冗談でしょう?名無しさん:2015/01/12(月) 04:01:41.38 ID:???.net
基本的に自然界に存在しないものを人間が作るのは無理。

>>288
空から基本方程式が降ってくるわけないんだから、それ以上の自然さを求められてもしょうがない。
相対性原理とか等価原理みたいな思想が根底にあるなら別だけど。
むしろ古典系を量子化してうまくいくことに感心すべきじゃないの。

291 :ご冗談でしょう?名無しさん:2015/01/12(月) 05:47:49.14 ID:???.net
ニュートン運動方程式もマクスウェル方程式もアインシュタイン方程式も測地線方程式も方程式なのに

292 :ご冗談でしょう?名無しさん:2015/01/12(月) 13:24:10.93 ID:???.net
>>289
必要なエネルギーがでかすぎる

293 :ご冗談でしょう?名無しさん:2015/01/12(月) 13:55:53.65 ID:???.net
大英帝国とモンゴル帝国はどっちの方が偉大ですか?

294 :ご冗談でしょう?名無しさん:2015/01/12(月) 15:29:02.74 ID:???.net
アラブ人とユダヤ人はどっちの方が優秀ですか?

295 :ご冗談でしょう?名無しさん:2015/01/12(月) 16:31:56.02 ID:???.net
セルオートマトンのスレって無いんですか

296 :ご冗談でしょう?名無しさん:2015/01/12(月) 16:43:39.21 ID:???.net
>>291
だからどうした?

297 :ご冗談でしょう?名無しさん:2015/01/12(月) 18:34:27.07 ID:???.net
わからないなら黙ってればいいのに

298 :ご冗談でしょう?名無しさん:2015/01/12(月) 19:10:50.39 ID:???.net
煽っても煽ってもレスしませんからー

299 :ご冗談でしょう?名無しさん:2015/01/12(月) 19:34:29.47 ID:???.net
>>291
ニュートン運動方程式も
マクスウェル方程式も
アインシュタイン方程式も
測地線方程式も
天から降ってきたと言いたいのか?

300 :ご冗談でしょう?名無しさん:2015/01/12(月) 20:04:40.15 ID:???.net
バカは黙ってればいいのに

301 :ご冗談でしょう?名無しさん:2015/01/12(月) 21:43:19.25 ID:???.net
変分原理みたいなのは、他にもいろいろな極値がありえるのに、なんで特定の極値を考える特定の積分がこの世界に関連するのか?という問いについては答えは無い。

302 :ご冗談でしょう?名無しさん:2015/01/12(月) 21:47:54.29 ID:???.net
変分原理なんてただの言い換えじゃん

303 :ご冗談でしょう?名無しさん:2015/01/13(火) 09:42:18.81 ID:???.net
テスト。

304 :ご冗談でしょう?名無しさん:2015/01/13(火) 11:55:27.52 ID:???.net
このサイトに書いてあること説明してください
http://members3.jcom.home.ne.jp/m_hidaka/04_kensyou/04_02_00.html

305 :256:2015/01/13(火) 12:59:28.76 ID:???.net
どなたかご親切な方、「>>259」にお答えくだされば幸いです。

306 :ご冗談でしょう?名無しさん:2015/01/13(火) 13:04:21.74 ID:???.net
>>304
デタラメ
以上。

307 :ご冗談でしょう?名無しさん:2015/01/13(火) 13:07:59.65 ID:???.net
相間の言う事を詮索しても無駄だよな

308 :ご冗談でしょう?名無しさん:2015/01/13(火) 14:31:55.10 ID:???.net
>>304
論文を読めばほとんどの人がそんな勘違いするわけないと思うが、最後に再定義したのはエネルギーであって、M'-MをMに再定義なんかしてない

309 :ご冗談でしょう?名無しさん:2015/01/13(火) 14:37:35.26 ID:???.net
>>301
経路積分で考えても、答えはないのか?

310 :ご冗談でしょう?名無しさん:2015/01/13(火) 15:27:38.40 ID:???.net
>>308
というか明らかに静止エネルギーでない文脈でE=mc^2を使っているので
そもそも最初から根本的に間違ってる

311 :ご冗談でしょう?名無しさん:2015/01/13(火) 16:08:04.29 ID:???.net
>>304のアインシュタインの証明で、

>しかし、vがcに比べて小さければ、Kに関する運動量も2次の微小量(1に対してv2/c2)を無視すれば同一である。
>運動量のz成分は(E/2c)sinα あるいは十分な正確さで(高次の量を無視して)(E/2c)α=(E/2c)(v/c)である。

ってところは、
エネルギーの増加をΔEとして、運動量のz成分の厳密な値は
{(E+ΔE)/2c}sinα
で、ΔEは一次の微小量だから、一次近似で消えるってことなら理解できるんだけど、
二次の微小量を無視すれば同一というのはよくわからん。

あと、運動量=Mvを満たす量として質量を定義したなら、E=Mc^2は厳密に成り立つはずなのに(定数の不定性は除いて)、この導出だと
E=Mc^2+o(v)
になるはずだよね?(o(v)はvの一次の量)

つまり、この導出だと、精々適用範囲はニュートン力学が成り立つ範囲ってことでいいのか?

312 :ご冗談でしょう?名無しさん:2015/01/13(火) 17:01:00.37 ID:???.net
とんでも批判も科学的思考できないやつがやると害悪でしかないな

313 :ご冗談でしょう?名無しさん:2015/01/13(火) 23:12:43.29 ID:???.net
金属と金属を接合したときに界面に空乏層がどのように
形成されるのかを考えようとした時に、前までは空乏層幅をdとした時

d=2*(誘電率)*(真空の誘電率)* (電位) / (素電荷)*(キャリア濃度)

であらわされるので金属の場合は分子に対してキャリア濃度が多くてdが無視できるをほど
狭くなり実質的に考えなくてよくなるものだと思っていたのですが
よく見ると金属の場合、誘電率をどうやって見積もったらいいのかわからないので
結局計算できなくて困っております。

どなたか金属-金属接合での界面の電荷の空間分布についてわかる方がいれば教えてください

314 :ご冗談でしょう?名無しさん:2015/01/14(水) 00:52:10.64 ID:???.net
本を読むと、アインシュタインの方程式をホーキング博士がとくとこれこれこういう結論になるというような文章をちょくちょくみるんだけど、
アインシュタインさんは、方程式の答えを書いていないの?

315 :ご冗談でしょう?名無しさん:2015/01/14(水) 00:57:21.52 ID:???.net
奴は線形近似で議論はしたが
特殊解は見つけていない

316 :ご冗談でしょう?名無しさん:2015/01/14(水) 02:33:09.34 ID:???.net
一番深い学問は何ですか?

317 :ご冗談でしょう?名無しさん:2015/01/14(水) 02:56:45.66 ID:???.net
できるだけ浅くて薄っぺらい質問でこのスレッドをいっぱいにしてください。お願いします。

318 :イスラム金融指導者遅獄先生主対性理論より実戦コーヒー豆SHO-GUN:2015/01/14(水) 11:05:59.72 ID:Pp6NLXQW8
科学って学の中でも最下ランクぐらいさ。

319 :ご冗談でしょう?名無しさん:2015/01/14(水) 13:10:20.09 ID:???.net
ああああああ

320 :ご冗談でしょう?名無しさん:2015/01/14(水) 13:16:59.46 ID:???.net
イギリス王室とNASAはどっちの方が偉大ですか?

321 :ご冗談でしょう?名無しさん:2015/01/14(水) 13:26:47.45 ID:???.net
パレスチナに住みたいのですが、どうすれば良いのでしょうか?

322 :ご冗談でしょう?名無しさん:2015/01/14(水) 13:34:22.06 ID:???.net
イギリス人は世界一の勝ち組で間違いないでしょうか?

・英語が世界共通語。
・一番多く植民地を持ち、一時は世界最強国となり覇権を握った。

323 :ご冗談でしょう?名無しさん:2015/01/14(水) 14:08:57.39 ID:???.net
>>313
空乏層という用語をどう言う意味で使っている?
一般的な意味の空乏層は金属-金属界面には生じないぞ。

324 :ご冗談でしょう?名無しさん:2015/01/14(水) 14:34:50.36 ID:???.net
ニュートンの運動の法則の第1法則(慣性の法則)について質問です。
Wikipediaによると「すべての物体は、外部から力を加えられない限り、静止している物体は静止状態を続け、運動している物体は等速直線運動を続ける」とあります。
これは他の物理学者によって「外部から力が働かない座標系を定義できる」という風に言い換えられ、第1法則と第2法則は違うんだよ的な事をよく耳にします。
でもそうだとしたら、ニュートンは何故そんな回りくどい言い方をしたのでしょうか?
もし彼が現代風に第1法則を解釈していたら、科学者らしく明確に第2法則と分けて書くと思うのです。
ニュートンは何度も同じことを言うアホだったのですか?
それとも、そんな回りくどい言い方をする理由があったのでしょうか?

325 :ご冗談でしょう?名無しさん:2015/01/14(水) 15:12:05.47 ID:???.net
イギリス王室の開祖とマイクロソフト社の創業者はどっちの方が偉大ですか?

326 :ご冗談でしょう?名無しさん:2015/01/14(水) 16:52:23.67 ID:???.net
>>324
当時は慣性の法則は常識ではなく、等速であろうと物体が動いているのは力が働いているからだというのが常識だったのです。
その常識をぶち壊す意味が込められています。

ニュートンに習って、例え他の原理で内包されているとしても、一般相対論の原理に「光速度は不変ではない」というのを別個に書いててもいいと思います。

327 :ご冗談でしょう?名無しさん:2015/01/14(水) 17:32:18.11 ID:???.net
>>326
つまり「大事なことなので二回言いました」って事ですね。

328 :ご冗談でしょう?名無しさん:2015/01/14(水) 17:32:53.18 ID:???.net
ハッタリはもう何年もこの一芸でレスし続けてんだから言うだけ無駄

329 :ご冗談でしょう?名無しさん:2015/01/15(木) 00:40:51.94 ID:???.net
>>326
ニュートン自身は>>324のような理解に達していない
等速度運動しようとする「慣性の力」なるものが働くと言っている

330 :ご冗談でしょう?名無しさん:2015/01/15(木) 00:58:04.91 ID:???.net
>「外部から力が働かない座標系を定義できる」という風に言い換えられ

これって屁理屈だよな
第2法則と分ける理由にならない

331 :ご冗談でしょう?名無しさん:2015/01/15(木) 02:27:29.97 ID:???.net
第二法則で議論する空間を事前に作らなければいけない以上第一法則は必須だ

332 :ご冗談でしょう?名無しさん:2015/01/15(木) 02:39:25.31 ID:???.net
>>324
標準的には、
運動量の定義 : 運動量は物体の(慣性)質量と速度の積
第一法則 :
 (a) 慣性系で力を受けない物体は等速直線運動する
 (b) 慣性系がある
第二法則 :
 慣性系において運動量の時間微分は物体の受ける力に等しい
 (ニュートン曰く:慣性系において物体の運動量の変化は物体の受ける力に比例する)
という感じになっていて、非慣性系の運動は論じられていない。
非慣性系の運動については、フォーマルにはダランベールの原理を通じて、
実用上は「近似的に慣性系と見なせる非慣性系」の存在を示すことで理論に取り込んでる。
あと、力の定義は静力学に由来していて、実質的には無定義語。
第二法則を力の定義にするのは解析力学の立場で、
ニュートンの力学は力を基本として構築されてるので若干抽象的というかあやふやになる場面が多い。

>>326
アリストテレス的な運動論はニュートン以前から否定されていて、一般的にはその功績はガリレイに帰せられる。
ただ、天体の運動や重力についての理解はニュートン以前には得られていなくて、
たとえばガリレイは水平面に沿った運動については慣性を見出したけど、鉛直方向については慣性を認めていない。
だから慣性の法則の完全な形での発見はニュートンによるものだと言っていいと思うけど、
>>326が思っているほどにはその発見は劇的じゃない。

333 :ご冗談でしょう?名無しさん:2015/01/15(木) 03:02:32.48 ID:???.net
>>331
別に第2法則の解釈がそうだというだけで
わざわざ分ける理由がない

なぜ力や速度の定義は一つの式で表すのに空間の定義だけ分ける必要がある?

334 :ご冗談でしょう?名無しさん:2015/01/15(木) 03:37:36.41 ID:???.net
第二法則はma=Fが成り立つとしか言っていない
これが成り立つ特定の空間=慣性系の存在を第一法則で保証している
慣性系の存在を第二法則に含めたいのなら第二法則を修正するしかない
もっとも第二法則に慣性系の存在を含めるにはandで接続するのだがそれじゃあ第三法則もandで接続して一つにまとめればいい
慣性系の存在とその上での関係式なんて別のものを一まとめにするんならそれもありだ

335 :ご冗談でしょう?名無しさん:2015/01/15(木) 04:10:13.27 ID:???.net
ma=Fが成り立てばm!=0でF=0でv'=0
これを「わざわざ」わける意味は?

336 :ご冗談でしょう?名無しさん:2015/01/15(木) 04:28:20.87 ID:???.net
「わざわざ」でもなくないかな
確かに一緒に書いてもいいとは思うが、別段分けて書くことは不自然でもないと思う。

337 :ご冗談でしょう?名無しさん:2015/01/15(木) 04:33:50.04 ID:???.net
位置も速度も力も説明しなきゃ分からない方程式なのに
なぜか慣性系の存在だけをわける特段の理由がない

338 :ご冗談でしょう?名無しさん:2015/01/15(木) 04:40:54.67 ID:???.net
ウィリアム1世(イングランド王)とビル・ゲイツ(マイクロソフト創業者)はどっちの方が偉大ですか?

339 :ご冗談でしょう?名無しさん:2015/01/15(木) 05:20:31.74 ID:???.net
一般の空間においてxやvが存在する以上議論する空間=慣性系を定める必要がある
ma=Fが成り立つというのはその議論する空間上での話であって一般には成り立たない
ma=Fが成り立つと述べるだけの第二法則では不足する

340 :ご冗談でしょう?名無しさん:2015/01/15(木) 05:26:21.62 ID:???.net
いやそういうことじゃなくて
mもaもFも説明が必要なのになんで慣性系だけわけんのよって話

341 :ご冗談でしょう?名無しさん:2015/01/15(木) 05:53:28.93 ID:???.net
慣性系の定義にはmは必要ないのでは

342 :ご冗談でしょう?名無しさん:2015/01/15(木) 05:56:09.65 ID:???.net
xもvも既に説明されているから
既に存在しているそれらを特定の空間上で議論することからNewton力学は始まる
議論する空間を指定する第一法則はNewton力学において必須だ

343 :ご冗談でしょう?名無しさん:2015/01/15(木) 05:58:13.06 ID:???.net
東京大学とケンブリッジ大学ってどっちの方が物理のレベル高い?

344 :ご冗談でしょう?名無しさん:2015/01/15(木) 06:04:56.69 ID:???.net
第二法則は運動量と力の関係で、力と速度や加速度、質量との関係は運動量を定義して初めて成立する。運動量は慣性の大きさを定量化する量で、慣性系において意味を持つ。

345 :ご冗談でしょう?名無しさん:2015/01/15(木) 06:08:08.53 ID:???.net
第三法則と同じだけ分ける意味があるのかってことでしょ
誰も慣性系の議論が必要ないなんて言ってるひといないでしょ

346 :ご冗談でしょう?名無しさん:2015/01/15(木) 07:27:01.05 ID:???.net
空間をも飲み込む生物が居たとして、その生物が空間を飲み込んだらどうなりますか?

347 :ご冗談でしょう?名無しさん:2015/01/15(木) 08:02:09.12 ID:???.net
測度論でも測度0は一般の測度の前に別に扱ったりするし
ある量が0の場合を基礎として一般論を展開するのはよくあることだろ

348 :ご冗談でしょう?名無しさん:2015/01/15(木) 08:08:23.66 ID:???.net
空間をも飲み込む生物が居たとして、その生物が空間を飲み込んだらどうなりますか?

349 :ご冗談でしょう?名無しさん:2015/01/15(木) 08:47:45.54 ID:VQ9QhjJt.net
>>323
返信遅れてすいません
一般に金属-金属界面に空乏層ができないとあるのですが
なぜそうなるのか教えていただけないでしょうか?

350 :ご冗談でしょう?名無しさん:2015/01/15(木) 10:29:38.39 ID:???.net
ニュートン力学の公理に慣性の法則は必要か [転載禁止]c2ch.net
4 :ご冗談でしょう?名無しさん:2014/11/14(金) 11:42:29.57 ID:???
ニュートンの認識では力の概念がまだ整理されておらず、
物体に内在する「慣性の力」も、外力(現代的視点における力)と
同列に置かれていた。なので「慣性の力」に対する第一法則と
外力に関する第二法則が同列に扱われているのはそれなりに
必然的なことだった

この辺の事情は山本義隆著「古典力学の形成」に詳しい

なおma=Fが力学問題に対する一般的で明示的な方程式として
提唱されたのはニュートンよりずっと後のオイラー。

351 :ご冗談でしょう?名無しさん:2015/01/15(木) 11:02:03.45 ID:???.net
>>350
山本のに詳しいっていうかそのまま写しただけだな

352 :ご冗談でしょう?名無しさん:2015/01/15(木) 15:26:37.02 ID:Q28WZ2SP.net
>>349
キャリアが多い

353 :ご冗談でしょう?名無しさん:2015/01/15(木) 16:46:56.37 ID:???.net
こんな誤解を招く第1法則をつくったニュートンはアホってことです

354 :ご冗談でしょう?名無しさん:2015/01/15(木) 16:54:49.74 ID:???.net
ていうかこのスレにプリンキピア読んだことある奴どれくらいいんだろ。
俺は読んだことないけど。
読んだことなくて批判してる奴なんていないだろから一杯いるんだろうな。

355 :ご冗談でしょう?名無しさん:2015/01/15(木) 17:36:28.86 ID:???.net
プリンキピアなんて読まずとも、運動の法則は高校の教科書にも書いてあるじゃないか

356 :ご冗談でしょう?名無しさん:2015/01/15(木) 18:00:12.14 ID:???.net
第一法則の内容は必要不可欠である以上誤解するのはそいつが阿呆なだけ
第二法則に組み込むかどうかだけの話
ただ第二法則にはそれが記されていない以上組み込むには第二法則も書き換えなければならない
現状の第二法則で完結してるなら第一法則なんて廃れてるわ

357 :ご冗談でしょう?名無しさん:2015/01/15(木) 18:10:55.00 ID:???.net
wikipedia
運動の第1法則
「すべての物体は、外部から力を加えられない限り、静止している物体は静止状態を続け、運動している物体は等速直線運動を続ける」

運動の第2法則
時刻を t、物体に作用する力を F として、運動の状態を表す量として運動量 p を導入すればdp/dt=f(F)と表すことが出来る。

上で慣性系導入できるのにdp/dt=0では慣性系導入できないのは屁理屈

358 :ご冗談でしょう?名無しさん:2015/01/15(木) 19:17:47.25 ID:uwIW4Gxe.net
ひなかわいいの〜

359 :ご冗談でしょう?名無しさん:2015/01/15(木) 20:21:15.23 ID:???.net
>>356
だったらなんであんな周りくどい一文で慣性系という座標系を定義をする?
科学者なら誰だって誤解を与えないようにもっと明確に書く
後付で第1法則に意味を見出して、思考停止した奴がそれを真に受けてるだけとしか思えん

360 :ご冗談でしょう?名無しさん:2015/01/15(木) 20:51:33.82 ID:5kX/wCXs.net
>>357
その表現なら下は位置を定義してないから第1法則必要かな
でも第一法則はp=mvでいいと思う

361 :ご冗談でしょう?名無しさん:2015/01/15(木) 21:02:55.35 ID:???.net
ニュートンはそもそもF=dp/dtと書いてないからな
第1法則と第2法則を別のものと考えててもそんなに不自然じゃない

でも現代において第2法則にF=dp/dtと書いておいて
あいまいな表現の第1法則を慣性系の定義が必要だから強調するのは不自然

362 :ご冗談でしょう?名無しさん:2015/01/15(木) 23:14:05.30 ID:???.net
慣性系の定義にはF=maは必要ない
というか、相対論的には第二法則は近似式だが、第一法則は相対論でも正しい

363 :ご冗談でしょう?名無しさん:2015/01/15(木) 23:17:24.59 ID:???.net
>>359
>後付で第1法則に意味を見出して、思考停止した奴がそれを真に受けてるだけとしか思えん

運動方程式が慣性系で定義される以上、慣性系の存在を仮定する文言を先に書くでしょ
舞台設定して初めて運動方程式は意味を持つ

364 :ご冗談でしょう?名無しさん:2015/01/15(木) 23:25:05.74 ID:???.net
別に特殊相対論でも正しいから分けてるわけじゃないだろ

365 :ご冗談でしょう?名無しさん:2015/01/15(木) 23:36:46.91 ID:???.net
ニュートンの運動方程式はやたら強調されるのにガリレイ変換は強調されない
特殊相対論的運動方程式って下手すら知らない奴いるしローレンツ変換ばかり

物理の説明って場当たり的だよな

366 :ご冗談でしょう?名無しさん:2015/01/15(木) 23:40:49.00 ID:nb4f/Ue+.net
相対対象のベクトル、ベクトル解析のはどこまで引けますかといっているのかな。ちょっとした疑問がが残っている。
∞ー=∞ーが成り立つ方程式の範囲を超えると、物理は崩壊するかと疑問が残っている。

367 :ご冗談でしょう?名無しさん:2015/01/15(木) 23:44:59.64 ID:nb4f/Ue+.net
相対対象のベクトル、ベクトル解析のはどこまで引けますかといっているのかな。ちょっとした疑問がが残っている。
∞ー=∞ーが成り立つ方程式の範囲を超えると、物理の法則の崩壊はするかと疑問が残っている。

368 :ご冗談でしょう?名無しさん:2015/01/16(金) 00:13:48.62 ID:ijcFneGm.net
すべてに神がいる功徳を積んで、世界を広げて、世界の物理法則を広げるしかないのか。

369 :ご冗談でしょう?名無しさん:2015/01/16(金) 00:18:00.48 ID:???.net
>>365
> ガリレイ変換は強調されない
そんなバカな。初等教育で速度の合成やってるはずだし、
物理勉強したなら、ほとんど即座に時間反転対称性とガリレイ変換対称性を確かめたと思うよ。
「相対性原理」の方が意識されないというのであれば確かにその通りだと思うけど。

物理の思想的に、場当たり的で済まない説明は避ける傾向にある。

370 :ご冗談でしょう?名無しさん:2015/01/16(金) 00:20:07.10 ID:???.net
現に三法則に入ってません

371 :ご冗談でしょう?名無しさん:2015/01/16(金) 00:26:28.64 ID:???.net
とりあえずプリンキピア貼っときますね。
http://en.wikisource.org/wiki/The_Mathematical_Principles_of_Natural_Philosophy_(1846)

372 :ご冗談でしょう?名無しさん:2015/01/16(金) 00:28:42.33 ID:???.net
せめてここ貼って
http://en.wikisource.org/wiki/The_Mathematical_Principles_of_Natural_Philosophy_%281846%29/Axioms,_or_Laws_of_Motion

373 :ご冗談でしょう?名無しさん:2015/01/16(金) 00:48:45.80 ID:jHfnyAFEm
このプリンキピアを貼るのは一瞬だが、
読むのは百年かかるであろう。

374 :ご冗談でしょう?名無しさん:2015/01/16(金) 09:30:14.86 ID:???.net
究極の天才は誰ですか?

375 :ご冗談でしょう?名無しさん:2015/01/16(金) 09:51:27.74 ID:???.net
>>374
お前はただのバカだから安心しろ

376 :ご冗談でしょう?名無しさん:2015/01/16(金) 11:27:57.97 ID:???.net
wiki貼る奴は馬鹿

377 :ご冗談でしょう?名無しさん:2015/01/16(金) 11:32:02.09 ID:???.net
中身も理解できない奴はバカ

378 :ご冗談でしょう?名無しさん:2015/01/16(金) 11:47:16.92 ID:???.net
>>357
dp/dt=F
この文だけでF=0ならv一定より等速度運動をすることが示せるから第一法則を明示する必要はないと言いたいのか
だが空間の指定がない以上任意の空間に対してdp/dt=Fが成り立つわけだ
これからF=0ならv一定という結論を導くためにはFは現在の力とは別物でなければならない
Fという物理量自体を見直すのというのなら第二法則を起点に置くのもいいだろう

379 :ご冗談でしょう?名無しさん:2015/01/16(金) 12:29:58.14 ID:???.net
頭悪すぎ

380 :ご冗談でしょう?名無しさん:2015/01/16(金) 12:44:52.50 ID:???.net
>>378
第1法則はそんな表現してない
むしろdp/dt=0と言っている
厳密にはmdv/dt=0だが

381 :ご冗談でしょう?名無しさん:2015/01/16(金) 14:09:29.76 ID:???.net
>>375
お前はただのバカでゴミクズの池沼だからさっさと首吊って自殺しろ

382 :ご冗談でしょう?名無しさん:2015/01/16(金) 14:16:21.70 ID:???.net
>>381
安心しろよ、お前が騒いだって誰も自殺しないし誰も傷つかない。
自分がバカだという自覚を胸に堂々と生きていけばいいさ

383 :ご冗談でしょう?名無しさん:2015/01/16(金) 14:20:11.42 ID:???.net
>>382
現実から目を逸らさないでちゃんと自分がバカだってことを自覚しながら生きていけよな。

384 :ご冗談でしょう?名無しさん:2015/01/16(金) 14:22:00.86 ID:???.net
言われなくてもさ。
そもそもこんなスレにバカ以外が張り付いてると思ってるのか?

385 :ご冗談でしょう?名無しさん:2015/01/16(金) 14:25:38.44 ID:???.net
と、バカが申しております。

386 :ご冗談でしょう?名無しさん:2015/01/16(金) 14:30:25.84 ID:???.net
何でそんなに必死なの?
自分だけはバカじゃないと思ってたの?

387 :ご冗談でしょう?名無しさん:2015/01/16(金) 15:02:24.59 ID:???.net
まあまあ落ち着け、争いは同レベルで起きる

388 :ご冗談でしょう?名無しさん:2015/01/16(金) 16:53:32.73 ID:???.net
第一法則は相対論でも成り立って、第二法則は相対論では成り立たないと言ったのは、第一法則は第二法則とは独立な概念ということを示すためだったんだが。

389 :ご冗談でしょう?名無しさん:2015/01/16(金) 17:34:33.88 ID:???.net
>>380
第一法則はF=0ならばdp/dt=0となるような空間=慣性系が存在すると言ってるんだよ
で第二法則はそのような空間において成り立つ関係式dp/dt=Fを示しているんだよ

390 :ご冗談でしょう?名無しさん:2015/01/16(金) 18:11:13.00 ID:???.net
>>389
そう表現せずに>>380と表現されてるから言ってるんだが

391 :ご冗談でしょう?名無しさん:2015/01/16(金) 18:20:39.37 ID:???.net
ここの人たちは何故ニュートンが三つの法則に分けたのかを議論してるの?
それとも何故いまだに三つの法則に分けた形で学校教育がなされているかを議論してるの?

392 :ご冗談でしょう?名無しさん:2015/01/16(金) 18:22:14.95 ID:???.net
http://i.imgur.com/Ju0tRqF.jpg
ここの1と2がわかりません。ガウス積分は実数ならわかるのですが、複素数だとどうやって計算するんでしようか

393 :ご冗談でしょう?名無しさん:2015/01/16(金) 18:23:00.81 ID:???.net
独立した概念だから法則わけなきゃいけないのなら力や質量や運動量はなぜわけないの?

394 :ご冗談でしょう?名無しさん:2015/01/16(金) 18:25:01.63 ID:???.net
>>393
どうやって分けるの?

395 :珈琲豆SHO=GUN:2015/01/16(金) 18:45:45.90 ID:uJFr+Zyq4
理系や物理だけのボキャブラリーじゃ理解。仮にそんなものがいるとして
それすらできん。(論文、試験)なんて無理。

396 :ご冗談でしょう?名無しさん:2015/01/16(金) 18:43:50.74 ID:???.net
第1法則がdp/dt=0と言っているにもかかわらず分けるんだったら第2法則を

・運動は運動量という値によってその性質が表される
・運動量を変化させる力というものが存在する
・運動量と力はdp/dt=Fという関係である

みたいに面倒くさく分けることはできる
なぜ、運動量と力の定義とその関係は一つの法則で表すのに
その結果の一つであるdp/dt=0は
第1法則の定量的表現であると考えられるにもかかわらず分けて説明するの?
dp/dt=0を内包するんだからここで慣性系も導入されているとしてもいいんじゃないの?

・そもそものニュートンの3法則に近い形で残したい
という理由なら一応は納得する

ただ分けなければ正しくないというのならそれは根拠が弱いと言っている

397 :ご冗談でしょう?名無しさん:2015/01/16(金) 18:49:36.35 ID:???.net
さらに言えば第2法則になにも仮定を加えずに第3法則は導けないと思うから
第3法則は独立性が高い
それに対して内包して説明して考えても不自然じゃない第1法則と第2法則は並べるのは不自然に感じる

398 :ご冗談でしょう?名無しさん:2015/01/16(金) 18:51:41.78 ID:???.net
じゃあもうそれでいいよ。はいこの話はもう終わり。

399 :ご冗談でしょう?名無しさん:2015/01/16(金) 18:56:15.76 ID:???.net
まあ物理学なんてこんなもんでしょ

400 :ご冗談でしょう?名無しさん:2015/01/16(金) 19:07:11.63 ID:???.net
不可逆過程と可逆過程を組み合わせたサイクルではクラウジウスの関係式より塔ツQ/T<0となると教科書に書かれていますが
エントロピーは状態量だからサイクルA→B→Aと一周して同じ状態にもどったら塔ツQ/T=0とならないんですか?

401 :ご冗談でしょう?名無しさん:2015/01/16(金) 19:09:02.89 ID:???.net
>>392
exp(-az^2)と複素積分を考える
z^2=(r^2)exp(i2θ)
として、θをうまく固定すればaの位相を打ち消せる。つまり、指数の肩が全て実数になる。そのθを固定してrだけ動かす積分は、元々のx軸の積分経路からθ回転させた直線に沿った積分になる。
コーシーの積分定理を使って、その二つの積分が等しいことを示せばいい。
すると答えはaの絶対値を使ったガウス積分と同じということになる。

402 :ご冗談でしょう?名無しさん:2015/01/16(金) 19:24:01.28 ID:???.net
>>397
第二法則は、ある大きさの「力」が存在するのなら物体の運動がどうなるかを述べたもので
どんな「力」が存在するかについては何も言っていない。
特に第二法則からは「力」が0になることがあることは導けない。
なので第一法則で「力」が0になることが確かにあるのだと述べているわけで、
これは第二法則とは独立だ。
それに加え、論理的に意味があるだけじゃなく、この世界には重力が普遍的に存在することを考えると
心理的にも特筆する価値があるな。

403 :ご冗談でしょう?名無しさん:2015/01/16(金) 19:27:16.60 ID:???.net
>>392
1の(ii)は積分経路の変更を二回しないといけない。
まず、平方完成する。
そして今度は長方形の積分経路を考える。
長方形を無限に横に伸ばせば、無限遠の辺の積分は0になり、コーシーの積分定理より上辺と下辺の積分の値が一致する。
上辺と下辺のいずれかがx軸になるわけだが、仮に下辺がx軸として、上辺のx軸に平行な直線は
z=u-b/2a
とすればいい。uは実数。
すると、exp(-az^2-bz)は実数になる。
すると1(i)に帰着する。
で、1(i)を解くのに積分経路の変更を一度したから、合計で2回積分経路を変更することにはなるのだが、まあすでに解いた(i)の結果を使えばいいので、そういう意味では積分経路を一回変更すれば解けるとも言える。

404 :ご冗談でしょう?名無しさん:2015/01/16(金) 19:30:58.11 ID:???.net
>特に第二法則からは「力」が0になることがあることは導けない。

もうね

405 :ご冗談でしょう?名無しさん:2015/01/16(金) 19:37:20.24 ID:???.net
数学で定義域を1つの公理で書けってかw

406 :ご冗談でしょう?名無しさん:2015/01/16(金) 19:40:58.25 ID:???.net
>>392
簡単そうだったから俺も考えてたけど、以外に解けなくてムカついた。
>>401>>403
GJ
案外面倒くさいことしないといけないんだね。

407 :ご冗談でしょう?名無しさん:2015/01/16(金) 19:58:51.22 ID:???.net
反射律は推移律から導けそうだけど実際には出来ず、独立した公理として必要である
みたいなことだと言えばわかるかな

408 :ご冗談でしょう?名無しさん:2015/01/16(金) 20:02:07.99 ID:???.net
んでなぜわざわざそれだけ独立させて運動量や力は一つの法則ですべて説明するわけ?

409 :ご冗談でしょう?名無しさん:2015/01/16(金) 20:05:27.04 ID:???.net
>>374が死ねばみんな丸く収まる

410 :ご冗談でしょう?名無しさん:2015/01/16(金) 20:17:58.51 ID:???.net
>>376
wikipediaのことをwikiと呼ぶのは情弱

411 :ご冗談でしょう?名無しさん:2015/01/16(金) 20:21:56.95 ID:???.net
>>408


412 :ご冗談でしょう?名無しさん:2015/01/16(金) 20:39:20.73 ID:???.net
力の大きさが運動量の変化に比例するって法則で0になるかは導けないってそもそもおかしいだろ

413 :ご冗談でしょう?名無しさん:2015/01/16(金) 21:08:21.68 ID:???.net
>>390
第一法則は慣性系の存在を示している
この上で第二法則があることにより力Fというものが定まる
質量の定義は第一法則に依存するが第二法則には依存しない
mdv/dt=FでF=0とすれば第一法則と言うがこれで慣性系を指定するというのならm,Fの定義はどうするのだ?
第一法則では力Fというものは陽に出てこないが第二法則では出てきてしまう

414 :ご冗談でしょう?名無しさん:2015/01/16(金) 21:12:39.46 ID:???.net
そもそも第一法則は力が定義されず使われてて循環論法ですが>>357

415 :ご冗談でしょう?名無しさん:2015/01/16(金) 21:17:08.53 ID:???.net
>>401>>403は略解で、無限遠で被積分関数が0になることは証明しないといけないし、
(i)の複素平面の積分経路は扇型(もしくは直角三角形)二つを頂点で合わせた形になるから、
コーシーの積分定理はまず片方の扇型(もしくは直角三角形)に適用して、そんでもう片方も同様という感じで書けばいい。
まあ別にわざわざ念を押すことでもないかもしれんけど。

416 :ご冗談でしょう?名無しさん:2015/01/16(金) 21:29:12.84 ID:???.net
Wikipediaでは力という言葉が使われているだけだが
まさかWikipediaを鵜呑みにする馬鹿ではあるまい

417 :ご冗談でしょう?名無しさん:2015/01/16(金) 21:37:47.64 ID:???.net
>>413>>416はさすがに恥ずかしいぞ
>>372

418 :ご冗談でしょう?名無しさん:2015/01/16(金) 21:40:58.31 ID:???.net
>>415
@x軸上での積分(非積分関数が複素数)、A円弧(aの偏角を打ち消すθまで)での積分、B動径軸上での積分(非積分関数が実数)に分割
コーシーの積分定理より@+A+B=0
Aはガウス型の関数だから、無限遠であらゆる冪よりも早く0に収束して消える
Bは実数だからいつものガウス積分
つまり、@もいつものガウス積分でおkってことですね

419 :ご冗談でしょう?名無しさん:2015/01/16(金) 21:43:42.08 ID:???.net
諦めたら

420 :ご冗談でしょう?名無しさん:2015/01/16(金) 22:05:33.11 ID:???.net
>>417
恥ずかしいのはお前だ
プリンシピアに書かれている内容そのままについての議論をしているのか
プリンシピアの内容は間違いだらけなんてのは常識でそれを改めて指摘する人はごく一部の人だけだろうよ
普通の人は正しく整理されたものに基づいて議論している
そして整理された運動の法則に関してさらに第一法則と第二法則をまとめられるのではないかという議論になっている
で論理に煩い現在では定義されていない力Fを陽には使わないで第一法則を記述する

421 :ご冗談でしょう?名無しさん:2015/01/16(金) 22:07:53.63 ID:???.net
別にF=0なら力を陽に使わないのと同じだしm=constでdv/dt=0
まったく問題ない

422 :ご冗談でしょう?名無しさん:2015/01/16(金) 22:08:59.20 ID:???.net
wikipediaの第1法則がお気に召さないならもっと主流な表現出してね

423 :ご冗談でしょう?名無しさん:2015/01/16(金) 22:09:37.14 ID:???.net
で整理された現在での第二法則はよくmdv/dt=Fで表されるがこの場合m,Fが陽に現れているのでその説明は必要だろうよ
どちらかをその式でもって定義してやってもいいがな

424 :ご冗談でしょう?名無しさん:2015/01/16(金) 22:09:48.70 ID:???.net
ドラゴンボールでニュートン生き返らせて聞けよ

425 :ご冗談でしょう?名無しさん:2015/01/16(金) 22:12:53.72 ID:???.net
つか第一法則が一番ニュートンの表現が残ってる法則だし

426 :ご冗談でしょう?名無しさん:2015/01/16(金) 22:14:06.30 ID:???.net
>>423
別に>>421で第一法則と同等になるんだからなんら問題ないだろ

427 :ご冗談でしょう?名無しさん:2015/01/16(金) 22:15:47.51 ID:???.net
mdv/dt=FのもとでF=0とするのにFの説明がいらないとは正気か

428 :ご冗談でしょう?名無しさん:2015/01/16(金) 22:16:17.86 ID:???.net
>>418
多分、R→∞で0になることを示すのは簡単じゃないと思う。
簡単じゃないというか、多少テクニカルなことをしないといけないと思う。

429 :ご冗談でしょう?名無しさん:2015/01/16(金) 22:18:21.52 ID:???.net
第一法則で「外部から力を加えられない」のと「力Fが0である」で
なにか表現力に差がありますか?

430 :ご冗談でしょう?名無しさん:2015/01/16(金) 22:21:30.77 ID:???.net
>>428
不等式で非積分関数の上限を抑える感じかな?

431 :ご冗談でしょう?名無しさん:2015/01/16(金) 22:21:52.87 ID:???.net
外部から力を加えられないというのは力という言葉を陽にしなくても表現できる
力が働かない状況を力という言葉無しで作ればいいのだから
mdv/dt=Fという第二法則は当然力という言葉無しで表記できない

432 :ご冗談でしょう?名無しさん:2015/01/16(金) 22:28:09.02 ID:???.net
プリンキピアで循環論法と言うと質量が密度×体積で密度が質量÷体積ってのも有名。

433 :ご冗談でしょう?名無しさん:2015/01/16(金) 22:46:41.40 ID:???.net
>>430
俺のやり方がおかしいのかもしれないが、aの偏角(位相)がπ/2より大きいか小さいかで場合分けがいる気がする。
複素共役を取ればいいから、aの偏角は0からπまでを考えればいいとして。

434 :ご冗談でしょう?名無しさん:2015/01/16(金) 22:47:22.65 ID:???.net
あのさあ別に第1法則があろうがなかろうが力の説明必要でしょ
別に第1法則があったからといって力が説明できるわけじゃない
というか第1法則はいきなり「力」を使って定義してる。
陽にしてないというと聞こえはいいが未定義ってことでしょ。
その状況でよければ力F=0でも同じだよといいたいの


第2法則で具体的にdp/dt=Fとして物質属性である質量mの存在と力Fの存在と
その関係が比例関係であるといっている

第1法則は慣性系の存在が必要だということを主張してるといいたいというのはわかる

しかし第2法則のその比例関係を認めるということは
mdv/dt=0はv=constで第一法則が主張したい慣性系そのものの存在を
認めることも含んでいると考えることができる
「わざわざ」第1法則を分けなくても第2法則を受け入れれば
第一法則を認めるのと同じ意味があるってこと

だから「わざわざ」第1法則が必要なの?って言ってるの

435 :ご冗談でしょう?名無しさん:2015/01/16(金) 22:50:19.92 ID:???.net
あ、反比例関係だった

436 :ご冗談でしょう?名無しさん:2015/01/16(金) 23:28:00.26 ID:???.net
力が働かないような状況を考えるのに力という言葉がいると思ってるのか
力という言葉が未定義でも後に導入する力が0となる状況は考えられる
質量の定義は第二法則に依らないので力はmdv/dt=Fで初めて陽になる
F=0と言うのは力Fが定義されていないと意味を持たないが

437 :ご冗談でしょう?名無しさん:2015/01/16(金) 23:32:36.50 ID:???.net
っていうかお前らいつまで続けるんだよw

438 :ご冗談でしょう?名無しさん:2015/01/16(金) 23:33:19.23 ID:???.net
dp/dt=0
dp/dt=FのときF=0でdp/dt=0
上に力Fは定義されている必要はないが下は定義されていなければならない
下の式だけで議論するのなら力Fを定義しないと

439 :ご冗談でしょう?名無しさん:2015/01/16(金) 23:33:59.52 ID:???.net
>力が働かないような状況を考えるのに力という言葉がいると思ってるのか
>力という言葉が未定義でも後に導入する力が0となる状況は考えられる
なにが言いたいのか分からない
>質量の定義は第二法則に依らないので力はmdv/dt=Fで初めて陽になる
なにが言いたいのか分からない
F=0と言うのは力Fが定義されていないと意味を持たないが
なにが言いたいのか分からない

で君の力の定義は?

440 :ご冗談でしょう?名無しさん:2015/01/16(金) 23:40:59.63 ID:???.net
後に導入する力がどういうものなのかは既に知っているので力が働かないような状況は力が未定義でも考えられる
物理現象が言葉の定義で変わるわけではないからそのような状況では未定義状態でも(後に導入する)力が働かない
第一法則のもとで第二法則によって陽にされたFを力と言えば良い

441 :ご冗談でしょう?名無しさん:2015/01/16(金) 23:44:12.85 ID:???.net
うんそのこだわりおれはいらんわ
じゃあね

442 :ご冗談でしょう?名無しさん:2015/01/16(金) 23:46:30.99 ID:???.net
むしろ君の力の定義を聞きたいな
いきなりmdv/dt=Fを持ち出すがm,Fはどう定義されるのか
そしてその定義は既存のものに一致するのか

443 :超音波テロの被害者:2015/01/16(金) 23:46:42.53 ID:qq7YLFgV.net
超音波テロの被害にあっています。
卑劣極まりない被害にあっています。

何が起こったかわからないときから、
わかってみれば、
まだ世の中に知られていない超音波テロ。

世の中のどれだけの音の振動源・発信源が
使用されているのかわからないが、
多数の振動源・発信源がシステム化され、
ネットワークを通して、
超音波・音波を集中させて
対象を攻撃するらしい。

人や社会が襲われ、罪もない人が超音波で襲われ、
卑劣な被害にあっています。

444 :超音波テロの被害者:2015/01/16(金) 23:47:26.40 ID:qq7YLFgV.net
聞こえる声、音。超音波テロの加害者の声。
「もらいました」という声とともに、
形のあるもの、ないもの、奪っていき、壊していく
超音波テロの加害者の声。

超音波による物理的な力で、
ものが飛び、ものが壊れる。
それが人間の体に対してまで。

身体の表面を突き抜け、内臓を攻撃される。
頭蓋骨を突き抜け、意識を失わされる。
聞こえる声、認識できない声で、精神的なダメージ。
人間の体を壊そうとする超音波テロ。

「見続けるのがいやだから、殺して終わる」、
「証拠隠滅だ」という超音波テロの加害者の声とともに
強烈な超音波の攻撃。
叫ばされ、いたぶられ、
超音波テロの卑劣な被害にあっています。
心の底から被害を訴え、祈っています。

天に神に届きますように。

445 :ご冗談でしょう?名無しさん:2015/01/16(金) 23:48:26.88 ID:qq7YLFgV.net
I'm suffering from dirty strong supersonic attacks!! Supersonic terrorisms!!
Help me!!

446 :ご冗談でしょう?名無しさん:2015/01/16(金) 23:50:29.72 ID:???.net
第一法則が必要不可欠だからここまで残ってるわけだし勝負の結果は初めから見えていた

447 :ご冗談でしょう?名無しさん:2015/01/17(土) 00:25:59.36 ID:???.net
第一法則は「慣性系が存在する」みたいに書きなおすべき、って主張もあるよね。

448 :ご冗談でしょう?名無しさん:2015/01/17(土) 00:38:51.87 ID:???.net
第0法則として「力はベクトル和で重ね合わせできる」を加えるべきという主張もあった。

449 :ご冗談でしょう?名無しさん:2015/01/17(土) 00:42:01.95 ID:xNt408xL.net
ベクトル空間の定義から始めよう

450 :ご冗談でしょう?名無しさん:2015/01/17(土) 00:57:06.82 ID:???.net
しかし容易に分かるように、慣性の法則は決して特別の法則ではなく、すでに、すべての運動規定的状況(力)は加速度を定める、というガリレイの直観の中に、同時に含まれている。
[…]偉大な研究者たちにさえとりついた創始者の偏見が、同一の事実を二つの異なる事実として考え、同じことを二重に形式化させただけなのである。(マッハ2006a,pp.230-1)

451 :ご冗談でしょう?名無しさん:2015/01/17(土) 01:50:51.12 ID:???.net
>>448
それがあったら中学時代の俺は躓かなかったなあ
斜面を滑る物体に働く力を何で斜面に垂直な方向と水平な方向に分解していいのかのか疑問だった
まあ結局それは公理な訳だが

452 :ご冗談でしょう?名無しさん:2015/01/17(土) 02:06:15.28 ID:???.net
第二法則で力が数学的な式で定義される
それはベクトルで表されているから力はベクトルでありその性質を持つ
改めて力のベクトル性を公理として言う必要はない

453 :ご冗談でしょう?名無しさん:2015/01/17(土) 03:39:56.33 ID:???.net
現にこれだけ揉めてるわけだし、ニュートンの運動の法則はイケてないと思う

454 :ご冗談でしょう?名無しさん:2015/01/17(土) 04:28:49.43 ID:???.net
数値計算でX座標をN個の離散的な点で近似する場合で、
f(i) (i=0,….N-1) が与えられたとします。
数値微分でf'(i)を求める場合、三点近似公式を使うと、そのままだと
f'(0) と f'(N-1) の2点は求められません。
5点近似公式の場合は、同様に 4 点が求められません。

数値微分には「端点処理」なるものがあるそうで、それについて詳しく書かれた
ウェブ・ページなどを紹介してもらえませんか?

455 :ご冗談でしょう?名無しさん:2015/01/17(土) 05:56:22.12 ID:???.net
>>433
類似の問題がyahoo知恵袋にあった
http://detail.chiebukuro.yahoo.co.jp/qa/question_detail/q14109949924
この場合は|a|=1に対応するけど。

思ってたよりも難しかった。

456 :ご冗談でしょう?名無しさん:2015/01/17(土) 09:44:57.54 ID:???.net
>>455
やっべ。
z=re^iθ
として、θ固定したら
dz=e^iθdr
になるから、答えに偏角表れるわ。
恥ずかしい・・・
質問した人またこれ見てくれればいいが。

457 :ご冗談でしょう?名無しさん:2015/01/17(土) 10:06:08.40 ID:???.net
ニュートンの運動の法則がイケてないってより
指導原理から始めたり基礎方程式から始めたり
熱力学みたいに第0法則や第3法則を入れたり入れなかったりで
流儀がありすぎてかつあいまいなのが問題だと思う

物理は自然科学のなかで一番厳密にしやすい学問なんだから
もっと統一感の共有考えてもいいと思う

458 :ご冗談でしょう?名無しさん:2015/01/17(土) 13:00:29.99 ID:pBwnEQXSY
物理が分かってるなら趣味さ

459 :ご冗談でしょう?名無しさん:2015/01/17(土) 14:53:48.03 ID:???.net
どっちでもいいよもう

460 :ご冗談でしょう?名無しさん:2015/01/17(土) 15:48:46.39 ID:???.net
ひなかわいいの〜

461 :ご冗談でしょう?名無しさん:2015/01/17(土) 19:25:07.88 ID:???.net
>>452
第二法則では物体に働く「全力」と加速度の関係が表されているが、
複数の力が働いた時にその力から「全力」を計算するルールは与えられていない。
よってその計算方法を与える法則が必要というのが第ゼロ法則派の主張。

そもそも力がベクトル和で足し算できるというのは静力学の範囲で既に経験的に
見つけられてた法則だからね。

462 :ご冗談でしょう?名無しさん:2015/01/17(土) 19:55:44.69 ID:y/n5AKO1.net
もう物理法則はプログラム言語で離散化して
その極限をもって法則としてくれ

めんどくさい

463 :ご冗談でしょう?名無しさん:2015/01/17(土) 20:23:25.48 ID:???.net
>>454
ノイマン境界条件とか何らかの境界条件を課す必要がある。
一番簡単なのは、0, N-1 上の差分を使わないことだけど。

464 :ご冗談でしょう?名無しさん:2015/01/17(土) 20:41:44.12 ID:???.net
第零法則なんて誰が主張してるの

465 :ご冗談でしょう?名無しさん:2015/01/17(土) 20:56:36.30 ID:jWq8w7PP.net
そもそも力の加法則なんかいらんだろ
合力さえ定まっているなら運動は決まるんだから
計算の利便性で力の分割合成は有用だがそれを公理とするのはダサい
現実では複数の力F1,F,2が働いていると考えてそれの和をとる
しかし初めからF1+F2という力が働いていたと認識すれば良い

466 :ご冗談でしょう?名無しさん:2015/01/17(土) 21:00:07.02 ID:???.net
>>465
力の加法性を認めなければ、物体にかかる重力を測定することが不可能になる。
よって、万有引力が慣性質量に比例するか否かの実験が不可能になる。

467 :ご冗談でしょう?名無しさん:2015/01/17(土) 21:01:44.62 ID:???.net
物理としては、理屈と実験結果との一致だけが唯一の判定基準だから
ダサいと感じるかとか綺麗と感じるかとかはどうでもいい。
ディラックは数学的な美しさだけを追求してたまたま美しい方程式を発見したけれど、
その後は何も「発見」できなかった事実を鑑みると、
理屈の綺麗さだけに頼ると物理の進展には寄与できないということを教えてくれる。

468 :ご冗談でしょう?名無しさん:2015/01/17(土) 21:07:44.40 ID:???.net
>>467
実験事実と一致してればどんなに汚い方程式でもいいって言うならそれはそれでひどい発言だなw

469 :ご冗談でしょう?名無しさん:2015/01/17(土) 21:10:16.55 ID:???.net
加速度ベクトルだということと運動方程式が成立することから
力もベクトルだと言えてるんじゃない?

470 :ご冗談でしょう?名無しさん:2015/01/17(土) 21:11:00.56 ID:???.net
実験事実と一致する汚い方程式と、実験事実と一致しない綺麗な方程式があれば
物理的に価値があるのは前者だ。

471 :ご冗談でしょう?名無しさん:2015/01/17(土) 21:11:05.44 ID:???.net
加速度「が」ベクトル

472 :ご冗談でしょう?名無しさん:2015/01/17(土) 21:12:49.91 ID:???.net
>>470
相対論前夜は実験と一致する汚い方程式がたくさんあったよなw
結局生き残ったのはアインシュタイン(やポアンカレ)の美しい式だけだったけど

473 :ご冗談でしょう?名無しさん:2015/01/17(土) 21:20:58.28 ID:74J6fZGY.net
修正ニュートン力学は価値があるかな

474 :ご冗談でしょう?名無しさん:2015/01/17(土) 21:23:18.21 ID:???.net
そんなことはない
既に最終的な姿は知っているのだから

475 :ご冗談でしょう?名無しさん:2015/01/17(土) 21:25:06.07 ID:vthAdo1f.net
ニュートン力学なんて、量子力学のマクロ近似版でしょ?

476 :ご冗談でしょう?名無しさん:2015/01/17(土) 21:28:05.81 ID:vthAdo1f.net
期待値版といったほうがよかったか

477 :ご冗談でしょう?名無しさん:2015/01/17(土) 21:30:14.08 ID:???.net
量子力学は古典力学の関係式の量子化という手続きとってる理論だけど

478 :ご冗談でしょう?名無しさん:2015/01/17(土) 21:32:41.55 ID:???.net
もう新しいスレ立ててそこでやってくれ

479 :ご冗談でしょう?名無しさん:2015/01/17(土) 21:49:23.80 ID:???.net
地球から4光年離れた星があったとして、
もしも仮に、光と同じ速度で進める宇宙船(実際には重さが増えて無理だけど、それは無視して…)でそこに向かったとしたら、
地球から見た場合は、4年かかって向こうにたどり着いたかのようにみえるが、
宇宙船に乗ってる宇宙飛行士からすれば、
4年もかからずに、もっと短い時間で向こうに到着するという理解でOKなのでしょうか。

480 :ご冗談でしょう?名無しさん:2015/01/17(土) 21:56:00.08 ID:???.net
>>479
光に近い速さで動く場合しか考えられません。少なくとも特殊相対論の枠組みでは。
その場合、速度次第で、宇宙船に乗ってる人からすると旅が1秒で終わるということもありえます。

481 :ご冗談でしょう?名無しさん:2015/01/18(日) 02:52:59.05 ID:aARBisit9
「実験事実と一致する」は「未来の実験事実と一致する(予言能力)」を含む
過去の実験事実としか一致しない汚い方程式には予言能力がない

482 :ご冗談でしょう?名無しさん:2015/01/18(日) 04:15:57.51 ID:???.net
>>472
お前さん歳いくつだ?

483 :ご冗談でしょう?名無しさん:2015/01/18(日) 05:25:02.10 ID:???.net
もしや>>472氏はQfwfq氏なんでは?

484 :ご冗談でしょう?名無しさん:2015/01/18(日) 09:48:39.07 ID:???.net
光が電磁場の波であって電磁場の最大速度が光速だとして、加速し続けられるロケットがあったら光速超えるんじゃない?ロケットは電磁場を媒体にしてなくて電磁場の制限を受けないわけだから。他の制限があるの?

485 :ご冗談でしょう?名無しさん:2015/01/18(日) 10:00:08.67 ID:???.net
光速度不変原理は電磁場の制約ではなく時空の性質

486 :ご冗談でしょう?名無しさん:2015/01/18(日) 11:14:04.35 ID:???.net
>>485
そうなのね

487 :ご冗談でしょう?名無しさん:2015/01/18(日) 15:30:23.37 ID:3fWtU8MQ.net
>>479
地球の立場から考えた場合:
ロケットの時間の進み方が地球の時計に比べて遅いので
ロケットの人は自分の時計をみると地球の時計より速く到着したことになる
ロケットの立場から考えた場合:
4光年という距離は地球の人が測った距離でロケットの物差しで測った距離はもっと短かくなる
そのためロケットの人にしてみれば割と速く到着する

488 :ご冗談でしょう?名無しさん:2015/01/18(日) 16:09:37.40 ID:???.net
物理のエッセンス力学・波動4訂版p14問題番号11

滑らかな水平面上の点Aで、角θの方向に初速V0で投げ出した。水平面との最初の衝突点をB,2度目の衝突点をCとする。
BC間の距離を求めよ。反発係数をeとする。



対称性を使ってBに衝突するときの速度がV0、Bから鉛直方向に速度eV0sinθというのは分かります。

解説では
BC間の鉛直方向について
0=(eV0sinθ)t−1/2gt^2
となっているのですが、この0は何ですか?
等加速直線運動の式ですが、なぜ変位を0として計算しているのでしょうか?

489 :ご冗談でしょう?名無しさん:2015/01/18(日) 16:13:35.24 ID:???.net
そりゃ鉛直方向に変位0だからだろうよ
BCは水平面にあるんだから

490 :ご冗談でしょう?名無しさん:2015/01/18(日) 16:15:51.93 ID:???.net
>>488
高校物理質問スレがこの板に別にあるよー

ってのはともかく、まず何をどの条件で解こうとしてるのか混乱してるだろ。
ストーリーとしては跳ね返った後の鉛直方向の速度を使って、次に衝突するまでの時間(t)を出す
そのtを使ってBC間の飛距離ev_0tcosθを計算する

っていう感じだけどここまではOK?

んで0が何ですかっていう質問だけど、教科書読め…じゃなくて鉛直方向に飛んでいって再び高さ0になる時=飛んでる時間っていえば通じるかね?
他の問題でやらなかった?

491 :ご冗談でしょう?名無しさん:2015/01/18(日) 16:42:56.65 ID:???.net
>>489
>>490


何か混乱してたみたいです
BもCも高さは同じ、だからB→Cの変位は0ってことですよね
変位についての式を立てたのに変位について考えてませんでした
素早い回答ありがとうございました。

教えていただいたスレも探そうと思います

492 :ご冗談でしょう?名無しさん:2015/01/19(月) 04:21:42.31 ID:x/trXARS.net
二重スリット実験についてだけど、
あれって観察してると波の性質が消えるっていうけど、具体的にはどうやって観察してるの?

よく分からないから、ビデオで撮影してるようなもんだと仮定するけど、そうすると、ビデオの電源が入って無ければ、波の性質は残るの?

もしビデオの電源が入って無ければ波の性質が残るなら、ビデオの電源が入ってるけど、回路が壊れかけてて、録画できたりできなかったりする場合は、録画出来てる時だけ波の性質が消えるの?

あと、録画はしてても、撮影後にビデオを破壊したら、観察者は実質的には居なくなるけど、その場合はどうなるの?

そもそも観察者ってなにをもって定義されてるの?無機物でも光子の振る舞いを記録さえしてれば、その後誰もその記録を見れなくても、光子からしたら観察されたことになるのかな?

493 :ご冗談でしょう?名無しさん:2015/01/19(月) 06:51:29.59 ID:5OZJWUP/.net
文系の大学1年生です。選択必修の物理学からの質問です。
質問が3つあります、親切な方、答えにいたるまでの解法を教えてください。

(1)水のそこにある石などの物体が、実際より浅いところにあるように見えるのはなぜか、説明せよ

光の屈折によるというのは分かりましたが、これがなぜなのか上手く説明できません。

(2)空気中での音速は温度15度で340メートル秒である。また水中では1500メートル秒である。
音が空気中から水中へ入射したときの屈折の様子を光の場合と比較せよ。

光の場合と逆で入射角<屈折角だからという答えで正しいのでしょうか?

(3)亜鉛の仕事関数は3.6eVである。亜鉛に光(紫外線)を当てたとき、電子が飛び出してくる(光電効果)のための光の条件は何か

仕事関数と光電効果の意味は何かは分かりました。調べたところ、8.7×10^14Hzより振動数が大きい光を当てる。が正解のようです。
ですが、仕事関数とエネルギーの関係が分からず、この振動数の求め方も分かりません。

授業で配られた光に関する演習プリントからなのですが、よろしくお願いします。

494 :ご冗談でしょう?名無しさん:2015/01/19(月) 10:04:57.68 ID:???.net
>>493
高校物理の教科書読め

495 :ご冗談でしょう?名無しさん:2015/01/19(月) 11:04:18.14 ID:???.net
>>493
来年また来てね

496 :ご冗談でしょう?名無しさん:2015/01/19(月) 11:39:04.50 ID:6/Q+1wgjJ
>>492
最後の行で正しい
>>493
(1)絵でなきゃ無理
(2)正しい
(3)νを振動数,hをプランク定数として、条件は hν>3.6eV

497 :ご冗談でしょう?名無しさん:2015/01/19(月) 12:26:36.14 ID:???.net
>>492
ビデオで撮るときに照明光を当てることで観測による系への作用が生じる。
ビデオの電源が入っているかどうかは無関係。

498 :ご冗談でしょう?名無しさん:2015/01/19(月) 12:32:20.32 ID:???.net
>>492が混乱してるけど
その混乱のもとである通俗書によく使われる「観測者」って表現の初出はどこだ?
この表現のせいで人間の意志が物理法則を変えられるとか思っちゃう奴出てきて困る

499 :ご冗談でしょう?名無しさん:2015/01/19(月) 12:50:29.11 ID:???.net
物理学者が書いた本でもノイマン-ウィグナー理論は観測者の概念を導入するものとするものが多い


>フォン・ノイマン-ウイグナーの統計的方法の示すところは、「観測結果に観測者の積極的な役割を取り入れるべきだ」というものである[8]

500 :ご冗談でしょう?名無しさん:2015/01/19(月) 12:58:26.16 ID:???.net
二重スリット実験はスクリーンに到達した光子を観測している。

501 :ご冗談でしょう?名無しさん:2015/01/19(月) 13:00:53.53 ID:x/trXARS.net
>>497
回答ありがとう!

ということは、観測してるとかは関係なくて、照明が当たってるかどうかが問題なんだね。

じゃあ、それは対象の光子(?) にどれだけ強い光が当たっているかだけが重要で、ビデオがあるかどうかは全く関係ないんだね。

よく「量子力学の不思議」みたいな説明で、観測するだけで、という表現があるけど、それは不思議さを煽ってるだけで、本当は単に観測可能になる程度の強さの光が当たると振る舞いが変わる、って程度の話なんだね。

その説明だとスッキリ分かるし、特に不思議とは感じない。

502 :ご冗談でしょう?名無しさん:2015/01/19(月) 13:04:55.79 ID:x/trXARS.net
>>500
あれ?そうなの?
俺が見た動画ではスリットを通る時の光子を観測しようとしたら、観測した時だけ振る舞いが変わる、って説明だったけど。

それに、この性質を利用したのが量子暗号通信なんじゃなかったっけ?
通信経路の途中で情報を見られたら振る舞いが変わるから、盗聴されたかどうか分かる、みたいな。

503 :ご冗談でしょう?名無しさん:2015/01/19(月) 13:07:03.54 ID:???.net
まあでもなぜ統計的な振る舞いをするのか
どこで波束の収縮が起こるのかの説明をしようとすると
1スレくらい簡単に煽り合いになるけどね

504 :ご冗談でしょう?名無しさん:2015/01/19(月) 13:09:16.68 ID:???.net
スリットを片方閉じて、光が通る経路が分かるようにしたら干渉は生じなくなって
経路が分からなければ干渉が生じる

505 :ご冗談でしょう?名無しさん:2015/01/19(月) 13:13:48.94 ID:x/trXARS.net
>>498

一般向けの説明には必ずこんな感じで書いてあるよね?

俺もこの説明にロマンを感じてしまったアホだけど、この説明はやめて欲しいね。

単に光子に光子が当たると振る舞いが変わるってだけなんだね。

なんか観測者という説明には悪意を感じる。
宗教を信じている人がアホを相手に神様の存在を信じさせようとしてるのか?

506 :ご冗談でしょう?名無しさん:2015/01/19(月) 13:15:01.73 ID:???.net
そこまで簡単じゃないから

507 :ご冗談でしょう?名無しさん:2015/01/19(月) 13:22:06.10 ID:x/trXARS.net
>>503
どこで収束とか、そこまで専門的になると、俺みたいな一般人には不思議を感じられないかも。

統計的な性質、というのには少しロマンを感じるがw

どこで収束するかって、そんなに大事な話なの?
そもそも何が問題とされてるのかすら、よく分からんw

508 :ご冗談でしょう?名無しさん:2015/01/19(月) 13:27:45.97 ID:???.net
決定論に還元できない
系の時間発展で観測の過程だけが不連続になる

上はアインシュタインが嫌った

509 :ご冗談でしょう?名無しさん:2015/01/19(月) 15:07:12.10 ID:7RGad6TY.net
>>506
どういうこと?

510 :ご冗談でしょう?名無しさん:2015/01/19(月) 15:20:07.97 ID:???.net
>>507
どこで収束するかってんで有名なのがシュレディンガーの猫、一般人にも十分ロマン。

511 :ご冗談でしょう?名無しさん:2015/01/19(月) 15:34:30.19 ID:???.net
そもそも>>492は電子の実験なのか光子の実験なのかわからんが
量子現象を記述するシュレーディンガー方程式は存在確率の時間発展しか記述しない
結果衝突現象も起こる確率しか計算できない
しかし実験結果は必ずどこかで当たっている
確率がいつ実験結果に変わったのか?
誤解を恐れずに言えばこれが波束の収縮

別にそんなの考えなくてもとりあえずいいじゃん
ていうのが現代の一般的な態度

それをはぁ?って言ったのがアインシュタインやシュレーディンガー

512 :ご冗談でしょう?名無しさん:2015/01/19(月) 17:32:49.67 ID:6/Q+1wgjJ
>>505
だいたい光子に光子が当たる現象など起こるか
ないとは言わんが超高エネルギーでも低確率だ

513 :ご冗談でしょう?名無しさん:2015/01/19(月) 17:36:41.08 ID:???.net
メモ

「至高」の候補

・痛みに耐える
・痛む
・「痛む」ということに価値を置く
・どうでもいい
・「どうでもいい」ということに価値を置く
・考えない
・「考えない」ということに価値を置く
・「全」
・「無」

これに加え、新しい候補

・決めない
・「決めない」ということに価値を置く

でも、結局どれが至高なのか分かんねえ。
「無」が至高な気がしてきたんだが、実際どうなんだろうな?

514 :ご冗談でしょう?名無しさん:2015/01/19(月) 17:46:05.70 ID:???.net
>>513
死ねば解るんじゃね?

515 :ご冗談でしょう?名無しさん:2015/01/19(月) 18:05:14.52 ID:???.net
死んだらどうなるのでしょうか?
無ですか?
それとも、何かの生物に転生するのでしょうか?
それとも、天国とか地獄とかがあってそこに行くのでしょうか?

516 :ご冗談でしょう?名無しさん:2015/01/19(月) 18:22:29.77 ID:Wg45xo3A.net
わしの研究によると波動関数自体粒子の集団についての記述であって

それが1粒子なら確率と解釈しよいうということで十分な集団ならそのまま粒子数だよ。

それが第二量子化なんて言ってるがこじつけよ。

517 :ご冗談でしょう?名無しさん:2015/01/19(月) 18:31:52.83 ID:???.net
鍵は量子もつれである

518 :ご冗談でしょう?名無しさん:2015/01/19(月) 18:32:12.86 ID:Wg45xo3A.net
相対論まで考慮して演算子の置き換えによって得られる波動関数がそれだ。

負の確率?それは負の粒子数だよ。確率というからおかしくなるんだよ。

1粒子だけを記述するなんで土台無理なのさ。何故かって?一粒子を分離するには

不確定性原理で何かを犠牲にするのでね。

519 :ご冗談でしょう?名無しさん:2015/01/19(月) 18:41:38.62 ID:???.net
巣に帰れ間抜け

520 :ご冗談でしょう?名無しさん:2015/01/19(月) 19:15:51.34 ID:???.net
>>516
徳真一

妄想を垂れ流す詐欺爺さんなのでスルーよろしく

「部分と全体」 W.ハイゼンベルク を読もうの会
http://wc2014.2ch.net/test/read.cgi/sci/1412678565/

物理を勉強してたら統合失調症になった 2
http://wc2014.2ch.net/test/read.cgi/sci/1348403037/

521 :ご冗談でしょう?名無しさん:2015/01/19(月) 19:17:56.50 ID:???.net
質問。「体、羽毛布団、毛布」の順が一番暖かいという
寝具業界のプロパガンダの根拠を物理で教えて。

「体、毛布、羽毛布団」のケースと何割くらい違うのかも

522 :ご冗談でしょう?名無しさん:2015/01/19(月) 19:23:03.43 ID:Wg45xo3A.net
陽電子が負の確率いや負の粒子数だ。で電子が正の確率いや正の粒子数だ。

正の粒子数と負の粒子数お手手を合わせて南無阿弥陀仏。消滅だ。

確率もゼロだな。わかったか馬鹿!

523 :ご冗談でしょう?名無しさん:2015/01/19(月) 19:28:15.75 ID:Wg45xo3A.net
そもそも湯川博士がクライン・ゴルドンの方程式で中間子論を見出したか。

それはそれを電子密度と解釈し知られている電磁方程式の解として解いたのだ。

それは電子密度だから電子一個ではないぞえ。

524 :ご冗談でしょう?名無しさん:2015/01/19(月) 19:32:08.27 ID:Wg45xo3A.net
ちなみに、湯川博士は小さい時お寺の坊さんになりたかったそうだ。

わしも中学出てお寺の坊さんになりたいと言ってたそうだ。

やっぱり偉大な人は違うんだなあ。

525 :ご冗談でしょう?名無しさん:2015/01/19(月) 20:06:00.26 ID:???.net
ロスチャイルドとアブラハムはどっちの方が偉大ですか?

526 :ご冗談でしょう?名無しさん:2015/01/19(月) 20:09:08.01 ID:???.net
人間が知るまで状態は収束してない、という主張を否定することはできない。直感的にそんなことあるはずがないとは思うが、証明する方法がない。

527 :ご冗談でしょう?名無しさん:2015/01/19(月) 20:13:45.05 ID:???.net
気に入るモデルが提示できないだけでしょ
ボーム解釈なら収束なんて概念がない
ただもっと不自然なモデルになる

528 :ご冗談でしょう?名無しさん:2015/01/19(月) 20:14:42.93 ID:???.net
1000京円ぐらい稼ぎたいのですが、どうすればそれだけの金を稼ぐことができますか?

529 :ご冗談でしょう?名無しさん:2015/01/19(月) 20:56:39.28 ID:???.net
大規模なクーデターでもおこしてハイパーインフレにする。
平和的な方法としては、政治家になり通貨切り上げのデノミを行う。

530 :ご冗談でしょう?名無しさん:2015/01/19(月) 21:59:00.89 ID:???.net
>>528
日本をジンバブエみたいにすればいい

531 :ご冗談でしょう?名無しさん:2015/01/19(月) 22:21:14.71 ID:???.net
さすがにそんなんなるまえにデノミするだろ

532 :ご冗談でしょう?名無しさん:2015/01/20(火) 03:12:02.72 ID:???.net
>>521
魔法瓶

533 :ご冗談でしょう?名無しさん:2015/01/20(火) 10:23:04.81 ID:???.net
>>529
もうちょっと簡単な方法は無いですか?

534 :ご冗談でしょう?名無しさん:2015/01/20(火) 13:00:10.90 ID:???.net
>>533
京楽にパチスロのコインの単位の名称を京円にしてもらう。

535 :ご冗談でしょう?名無しさん:2015/01/20(火) 13:25:48.78 ID:???.net
剛体の衝突応答について質問があります

2つの剛体が衝突したとき撃力が接触法線方向にはたらくとありました
これがいまいち理解できません
簡単に納得できる理由かこの説明がある本を教えてもらえないでしょうか

536 :ご冗談でしょう?名無しさん:2015/01/20(火) 13:42:05.70 ID:???.net
>>535
摩擦力が働かないんだったら、垂直抗力以外に働く力がない。
摩擦力が働くんだったら、そのかぎりでない。

537 :ご冗談でしょう?名無しさん:2015/01/20(火) 13:49:48.47 ID:???.net
>>535
剛体は定義上変型せず内部自由度を持たない。
このため衝突時に振動して音をたてることもないし、
摩擦熱を生じて内部エネルギーが上昇することもない。
つまり剛体衝突時に関与するのは保存力である垂直抗力しか有り得ない。
よって接触面に垂直な力のみが関与する。

538 :ご冗談でしょう?名無しさん:2015/01/20(火) 13:55:29.29 ID:???.net
返答ありがとうございます
垂直抗力自体がいまいち理解できません
垂直抗力が保存力だからというのも理解できていません

539 :ご冗談でしょう?名無しさん:2015/01/20(火) 14:08:49.76 ID:b/FrXHhKR
>>526
「人間が収束したと決めるまで」だろ、収束なんてそんなもんだ

540 :ご冗談でしょう?名無しさん:2015/01/20(火) 14:17:36.58 ID:???.net
>>538
抗力とは、弾性体においては物体が変型することによる復原力だ。
弾性体の復原力は保存力であり、そのポテンシャルはよく知られている通りkx^2/2で与えられる。
垂直抗力は剛体における抗力で、
剛体はk→∞の極限をとったものであるから
垂直抗力のポテンシャルは剛体同士が重なる領域で∞に発散し、
剛体同士が離れているときはゼロになる関数として表現できる。
このポテンシャルの微分である垂直抗力は剛体が接しているときにのみ∞の値をもち、
かつ剛体の表面に垂直となることがわかるだろう。

541 :ご冗談でしょう?名無しさん:2015/01/20(火) 14:44:08.47 ID:???.net
なんとなく自分の疑問が分かってきました

"任意の形状の剛体どうしの衝突は各剛体の接触面は同じ方向"なのかという不安
連続的な曲面同士では直観的には明かと考えてるのですが問題ないでしょうか

またこのときなぜ各剛体の速度方向に関係なくこの接触面の方向に力が働くのか
これは衝突を弾性体の極限を考えると
接触面方向へ押す力とその復元力と考えることができるということでしょうか

このあたりって一般的な力学の本で解説されているものなのでしょうか
今手元には原島しかなく確認できませんでした

542 :ご冗談でしょう?名無しさん:2015/01/20(火) 14:59:44.28 ID:???.net
摩擦とかを考えなければ、力は接触面に垂直に働くというのは、実験的な経験則と考えていいと思う。
あと、仮に摩擦が働いたとしても、「滑る」ことさえなければ回転運動エネルギーになってエネルギーは保存するはず。

543 :ご冗談でしょう?名無しさん:2015/01/20(火) 15:02:35.85 ID:???.net
なるほど経験則と考えるわけですね
床にどのように球を当てても垂直抗力がはたらくと考えるのは納得できます

ありがとうございました

544 :ご冗談でしょう?名無しさん:2015/01/20(火) 15:05:14.29 ID:???.net
アブラハムとパウロはどっちの方が凄いですか?

545 :ご冗談でしょう?名無しさん:2015/01/20(火) 15:26:37.34 ID:???.net
ビリヤードなんか垂直抗力を実感できるいい例だよ
今では体験できる場所も少ないけど

546 :ご冗談でしょう?名無しさん:2015/01/20(火) 16:35:52.90 ID:???.net
>>392にお答えいただいた方、ありがとうございます。お礼が遅くなって申し訳ありません。
大問2の規格化なんですが、(1]はできたんですが、(2),(3)がわかりません。今一度ご教授願えないでしょうか?よろしくお願いします。

547 :ご冗談でしょう?名無しさん:2015/01/20(火) 17:32:10.68 ID:???.net
>>546
(2)を解いてみた、間違えてたらごめん
規格化因子N1=N2=(α/π)^(1/2)
<ψ,φ>=i*(1/2α)
<ψ+φ,ψ+φ>=1+i*(1/2α)+(-i*(1/2α))+1=2

548 :ご冗談でしょう?名無しさん:2015/01/20(火) 17:38:24.64 ID:???.net
>>546

ごめん間違えてた
N1=N2=α* (π)^(-1/2)
<ψ,φ>=i*(1/2)
<ψ+φ,ψ+φ>=1+i*(1/2)+(-i*(1/2))+1=2

が正しいと思う

549 :ご冗談でしょう?名無しさん:2015/01/20(火) 17:59:41.02 ID:???.net
>>546
>>392の1の解答は>>456まであるから注意してくれ

ガウス積分の導出とのときと同じく、積分変数を極座標に直しても解けるし、多分直さなくても解ける。
∫∫dxdy(x^2+y^2)exp(-x^2-y^2)
=∫dy exp(-y^2)∫dx (x^2)exp(-x^2) + ∫dx exp(-x^2)∫dy (y^2)exp(-y^2)
と書けるけど、部分積分で
∫dx (x^2)exp(-x^2)
=∫dx x(exp(-x^2))'(-1/2)
=[-(1/2)x exp(-x^2)] + (1/2)∫dx exp(-x^2)
となる。第1項には∞と−∞を代入するけど、そしたら0になる。
よって、結局ガウス積分だけになる。
簡単のためにaは省いてるから、自分で直してくれ。
この部分積分の方法を使えば、(x^n)exp(-x^2)は必ず解ける。

550 :ご冗談でしょう?名無しさん:2015/01/20(火) 18:00:02.81 ID:???.net
>>546

(3)も解いてみた
N1=((α^5)/(π^3))^(1/4)
N2=(√2)*((α^7)/(π^3))^(1/4)
<ψ,φ>=(√2)*i*(α^(-1/2))
<ψ+φ,ψ+φ>=2

551 :ご冗談でしょう?名無しさん:2015/01/20(火) 18:07:05.11 ID:???.net
この手の問題は微分と積分の順序を入れ替えるテクニックを知っていれば簡単になる
∫∫dxdy(x^2+y^2)*exp(-α*(x^2+y^2))=-∂/∂α(∫∫dxdy exp(-α*(x^2+y^2))=-∂/∂α(π/α)=π/α^2
このテクニックを駆使して頑張ってね

552 :ご冗談でしょう?名無しさん:2015/01/20(火) 18:11:12.61 ID:???.net
>>551
おーこの解き方初めて見たわ
なるほどなあ

553 :ご冗談でしょう?名無しさん:2015/01/21(水) 00:07:00.74 ID:???.net
強相関系は末期って本当?

554 :ご冗談でしょう?名無しさん:2015/01/21(水) 00:29:53.38 ID:???.net
温度計は対象の系と熱平衡状態になることによって温度を測定するものらしいのですが、
測定対象の温度は温度計と熱平衡状態を作ることによって本来の温度より低く(または高く)なりませんか?
そのことによって本来の温度は測定できないのではないですか?

555 :ご冗談でしょう?名無しさん:2015/01/21(水) 00:38:05.60 ID:???.net
なる
どの程度なるかは熱容量で計算できる

556 :ご冗談でしょう?名無しさん:2015/01/21(水) 00:55:05.87 ID:???.net
原子核周囲軌道にある電子が、電磁放射によるエネルギー散逸
により核に落ち込まないのは、量子条件だと教わりましたです。


でも電子を加速器で核にぶつければ、電磁力で核に電子がくっ
ついた核はできないのでしょうか?

でもこんな核は聞いたことありません。

不安定で電子はくっつけないのでしょうか?陽子が中性子に
変わるのでしょうか?または???どうなんでしょうか??

557 :ご冗談でしょう?名無しさん:2015/01/21(水) 01:14:11.27 ID:???.net
>>556
原子核 電子捕獲 でぐぐれ

558 :ご冗談でしょう?名無しさん:2015/01/21(水) 05:15:25.36 ID:???.net
>>546
度々申し訳ない
見なおしたらいろいろと間違えてるわ

(1)
<ψ,φ>=0
<ψ+φ,ψ+φ>=2

(2)
<ψ,φ>=0
<ψ+φ,ψ+φ>=2

積分範囲が[-∞,∞]だから偶関数と奇関数の積の積分が0になる性質を使う
∫dx (x^(2n+1))*exp(-αx^2)=0

積分範囲が[0,∞]ならちゃんとした値になるけど、それでも上の解答は間違ってた

559 :ご冗談でしょう?名無しさん:2015/01/21(水) 10:10:48.07 ID:???.net
>>558
奇関数出てこなくね?

560 :ご冗談でしょう?名無しさん:2015/01/21(水) 11:14:36.27 ID:???.net
>>559
内積を計算するときに出てくると思う
(2)だと
<ψ,φ>=N1*N2*∫∫dxdy(x-iy)*(x-iy)*exp(-α*(x^2+y^2)) (ψは共役を取る必要がある)
=N1*N2*∫∫dxdy(x^2-2ixy-y^2)*exp(-α*(x^2+y^2))
=N1*N2*∫∫dxdy(-2ixy)*exp(-α*(x^2+y^2)) (xとyの対称性からx^2とy^2の項は互いに打ち消し合う)
=-2i*N1*N2*(∫x*exp(-α*(x^2))dx)*(∫y*exp(-α*(y^2))dy)
=0

561 :ご冗談でしょう?名無しさん:2015/01/21(水) 11:26:01.84 ID:???.net
>>560
あーごめん。今見れば何を勘違いしてたのかもわからないぐらい当たり前だったわ。

562 :ご冗談でしょう?名無しさん:2015/01/21(水) 11:57:04.92 ID:???.net
アホ乙

563 :ご冗談でしょう?名無しさん:2015/01/21(水) 22:10:04.36 ID:???.net
試験管に水入れてわずかな隙間もないピストンを使って [転載禁止]©2ch.net・

http://viper.2ch.net/test/read.cgi/news4vip/1421844734/


これ気になったから答えて欲しいんだけど一応物理に属するよね

564 :ご冗談でしょう?名無しさん:2015/01/21(水) 22:13:50.19 ID:oSh8yblX.net
やたら先生がTeXをマスターすることを薦めてくるんだけど
TeXってそんなに卒業後も必要になります?
今のところWordの数式ツールで十分な気がしますが・・・

565 :ご冗談でしょう?名無しさん:2015/01/21(水) 22:33:16.71 ID:???.net
学部卒で就職するのが普通ってレベルの大学なら
別にワードでいい

566 :ご冗談でしょう?名無しさん:2015/01/21(水) 22:47:57.50 ID:???.net
テキストベースでレイアウトできたほうが便利だよ

567 :ご冗談でしょう?名無しさん:2015/01/22(木) 00:04:17.26 ID:mcCOLKE6.net
2012年の石川県の高校入試問題を解いていてどうしてもわからないことがあるので質問です。

問 図でおもりが糸を引く力とつりあう力を意図の方向に分解し、解答欄に作図せよ。

自分はおもりが糸を引く力とつりあう力は他のものが糸を引く力だと考えて、
他に糸に触れているものは天井しかないので、天井が糸を引く力を作図したのですが、
模範解答がなぜかおもりと糸の間から矢印が伸びています。
模範解答のこの矢印は何が何を引く力なのでしょうか?
またどのように考えればこの解答に行き着くのでしょうか、ご教授お願いします。
ttp://iup.2ch-library.com/i/i1368263-1421852382.png

568 :ご冗談でしょう?名無しさん:2015/01/22(木) 00:26:53.13 ID:???.net
>>557
情報ありがとうです。
「原子核 電子捕獲」でググってみました…
「捕獲された電子は原子核内の陽子と反応し中性子となり、」

なんか飛躍してて難しいです。レプトンの電子とハドロンの
反応は勉強します。

でも、安定な陽子が崩れるのですか?

また、中性子に押し込まれた陽子は、そのままなのでようか?

569 :ご冗談でしょう?名無しさん:2015/01/22(木) 01:11:11.94 ID:QPW22CfrY
安定な粒子というのは単独の状態で安定なだけ、別の状況なら反応する
できあがった原子核が中性子過剰なら適当な半減期で陽子に戻る、安定核ならそのまま

570 :ご冗談でしょう?名無しさん:2015/01/22(木) 00:50:39.70 ID:???.net
>>567
糸が糸を引く力
滑車付近の糸はそこから張っている糸に引っ張られている

571 :ご冗談でしょう?名無しさん:2015/01/22(木) 01:13:13.75 ID:QPW22CfrY
>>563
即座に答が出てるじゃねーか

572 :ご冗談でしょう?名無しさん:2015/01/22(木) 01:35:00.94 ID:lLvB7Yjz.net
二重スリット実験をやってみたいのですが、このサイトのようにすれば縞模様が出ることが分かりました。

ここから縞模様を消して波束の収縮を確認するにはどうすれば良いですか?

http://www.epii.jp/articles/note/physics/double_slit

上のレスを見た感じだと、横から光を当てると良いのでしょうか?

573 :ご冗談でしょう?名無しさん:2015/01/22(木) 05:35:33.23 ID:???.net
>>572
縞を消したきゃ片方のスリットをふさげばいいのさ。
だが波束の収縮なら縞が生じてる状態でも起きてるからな。
波束の収縮の意味をわかってないだろ?

574 :ご冗談でしょう?名無しさん:2015/01/22(木) 06:09:29.45 ID:???.net
サイトはただの光の実験が書いてるし、量子論は関係ない話では
波束の収束とか量子論は全く関係ないぞ

575 :ご冗談でしょう?名無しさん:2015/01/22(木) 06:11:21.66 ID:???.net
誤解されそうな書き方してしまった。
波束の収束と量子論は関係ないって意味でなくて、波束の収束といった量子論的現象はこの場合関係ないって意味な。

576 :ご冗談でしょう?名無しさん:2015/01/22(木) 06:22:02.17 ID:???.net
>>572
波束の収縮を実感したいのなら、レーザーを絞りに絞って、
かつ光子一個でも観測できるような観測装置を使う必要がある。
この時、光子は一個々々点として観測される。
時間が経つにつれ光子の分布は強いレーザー光の干渉縞と同じ形になる。
つまり、光子一個の波束は干渉縞の形をしているが、
観測されると一点に収縮し、点として観測される。
これが波束の収縮だ。

577 :ご冗談でしょう?名無しさん:2015/01/22(木) 06:25:19.66 ID:???.net
>>572
通常の出力のレーザー光の場合、あまりにも多数の光子の波束が
同時に別々の点において収縮しているため、
見た目には収縮を実感することはできない。
しかし、収縮が起こっていることは変わり無い。

578 :ご冗談でしょう?名無しさん:2015/01/22(木) 19:40:23.18 ID:???.net
爆音してたら、くびれてるペットボトルの、くびれ部分が最も振動しているような気がした。
円周が短いから?
それとも気のせいか。

579 :ご冗談でしょう?名無しさん:2015/01/22(木) 21:02:15.89 ID:???.net
>>578
死ね公害

580 :ご冗談でしょう?名無しさん:2015/01/22(木) 22:13:32.55 ID:???.net
物理の先生が低周波は人間の内臓を揺らすって言ってたんですけど本当ですか?

581 :ご冗談でしょう?名無しさん:2015/01/22(木) 23:03:59.06 ID:???.net
>>580
打ち上げ花火って見たことあるか?
あるならその時の感覚だ。

見たことないなら



さっさと見てこい。

582 :562:2015/01/22(木) 23:25:38.31 ID:lLvB7Yjz.net
レスありがとうございます。

波束の収縮を実験するには、光子一個レベルで観測する必要があるんですね。

でも、ちょっと分からないのは、スリットを通る前の光に上下から強力な光を当てたら、スリットを通る前に収縮が起きる光子が増えるので、結果としてスリットの部分だけに光が当たる確率が増え、その部分は明るく、それ以外は暗くなることが確認できたりしないんですか?

583 :ご冗談でしょう?名無しさん:2015/01/23(金) 00:09:12.88 ID:???.net
>>582
光は光と相互作用しない。
星の光は四方八方から届いているのに
別の星の光に関わらず星が見え続けていることからわかるだろう。
光に横から光を当てても全く何も起こらない。

584 :ご冗談でしょう?名無しさん:2015/01/23(金) 01:02:43.76 ID:???.net
軽度の病気レベルの思い込み

585 :ご冗談でしょう?名無しさん:2015/01/23(金) 01:34:00.61 ID:???.net
マックスウェル方程式はリニアだからな

586 :562:2015/01/23(金) 01:35:59.74 ID:lf3P4mvJ.net
あれ?でも >>497 で光を当てた時に影響を受けるというような説明がありますよね?

もしかして光を当てるだけじゃなくて、他にも何か条件があるのですか?

587 :ご冗談でしょう?名無しさん:2015/01/23(金) 07:03:33.08 ID:???.net
>>492の話は多分電子の二重スリット実験で、電子がスクリーンに当たったら、その当たった場所が発光するとする。
ビデオでその様子を観測したら、波束の収束はいつ起こるんだろうか、電子がスクリーンに当たった瞬間か、ビデオが観測した瞬間か、人間がビデオを確認した瞬間か、という話、だと思う。

588 :ご冗談でしょう?名無しさん:2015/01/23(金) 07:58:16.73 ID:???.net
光子の実験でもスクリーンは物質で出来ている

589 :ご冗談でしょう?名無しさん:2015/01/23(金) 09:57:25.32 ID:Sy3WCwEf.net
「発光」は物理現象だが、「波束の収縮」は人間の思い込み。
実際に、収縮が現象とは限らんし、そもそも、波束が実在するかも定かならず。
たまたま「そう考えたら説明がつく」ってだけにすぎない。

590 :ご冗談でしょう?名無しさん:2015/01/23(金) 11:06:59.53 ID:???.net
一般的には収束が問題になってるのではなく
自然な解釈が存在しないのが問題になってるんでしょ
射影仮説ない解釈と言えばボーム解釈あるけど
あれはあれで量子ポテンシャルが不自然極まりないし
コペンハーゲン解釈を情報論的にとらえ直す動きあるけど
長距離相関は認める立場だし

591 :ご冗談でしょう?名無しさん:2015/01/23(金) 12:14:08.18 ID:???.net
>>589
波束が「いつ」「どうやって」収縮するかはまだ未解決の量子観測論上の問題だが、
観測により収縮するかどうかは別として収縮が起こっていることは確立した観測事実だ。
エヴェレット解釈だろうが量子デコヒーレンスだろうが、波束の収縮は認めている。
それを「確率波の収縮」であると解釈しないというだけの話だ。

592 :ご冗談でしょう?名無しさん:2015/01/23(金) 12:34:48.84 ID:???.net
それこそ言い回しの問題だがボーム解釈のようにすれば
パイロット波が収縮したとは普通言わないな

593 :ご冗談でしょう?名無しさん:2015/01/23(金) 12:50:47.97 ID:Sy3WCwEf.net
>収縮が起こっていることは確立した観測事実だ。

「収縮」を観測は出来ない。
観測結果を「収縮」と解釈しているに過ぎない。
つまり、「収縮」が現象とは限らん。

594 :ご冗談でしょう?名無しさん:2015/01/23(金) 13:07:02.57 ID:???.net
>>593
収縮した「結果」は観測可能だ。
「収縮しつつある何か」は観測不能だが。
結果の存在だけで現象の存在を確認出来ないなら
ビッグバンも存在を確認出来ないな。
まあビッグバンは無かった厨なのかもしれんが。

595 :ご冗談でしょう?名無しさん:2015/01/23(金) 13:18:49.40 ID:???.net
屁理屈の領域

596 :ご冗談でしょう?名無しさん:2015/01/23(金) 13:28:56.07 ID:???.net
これを屁理屈と感じるのなら理論物理には向いてないな

597 :ご冗談でしょう?名無しさん:2015/01/23(金) 13:31:06.37 ID:???.net
キリッ!

598 :ご冗談でしょう?名無しさん:2015/01/23(金) 15:16:11.99 ID:???.net
言葉遊びはやめよう

599 :ご冗談でしょう?名無しさん:2015/01/23(金) 16:08:27.44 ID:???.net
物理学は数学の応用分野ってのは本当ですか?

600 :ご冗談でしょう?名無しさん:2015/01/23(金) 16:54:39.40 ID:c/z8v4ApL
荒らしは去れ

601 :ご冗談でしょう?名無しさん:2015/01/23(金) 17:10:07.10 ID:???.net
原理的に測定不能な波束について縮んだだの広がっただのを論じるのはナンセンス

602 :ご冗談でしょう?名無しさん:2015/01/23(金) 18:07:46.08 ID:???.net
波束の大きさを5割だけ収縮するみたいなことができるようになったって聞いたんだけど、誰かその実験のソース知らない?
1,2年前に、最近ようやく量子論の検証に必要な精度の実験ができるようになってきた、みたいな話を聞いたんだけど。

603 :ご冗談でしょう?名無しさん:2015/01/23(金) 21:36:31.64 ID:???.net
数学と物理には決定的な違いがある。
数学も物理も、数学的な操作で解を求める理論を作ろうとする手法は共通していて、その段階ではほぼ似通った活動をしている。
しかし、物理は解が求まった後に各種物理定数に数値を代入して、解が物理的に意味があるか無いかを、実験結果という自然からの要求に応える必要がある。
数学者はこのような要求と無関係に、自由に数学的操作を追求するだけで良い。
物理学者はそのような追求をしながら、その活動の成否を自然から突き付けられる。

604 :ご冗談でしょう?名無しさん:2015/01/23(金) 22:25:37.28 ID:???.net
球Aを取り囲むようにすこし隙間をあけて中心を同じくする球殻Bを用意する
球Aを固定したまま球殻Bを自転させれば
平衡に達したときその隙間にはどういう空気の流れができるでしょうか
偏西風、偏東風みたいな感じになると思うんですが

605 :ご冗談でしょう?名無しさん:2015/01/23(金) 22:30:44.18 ID:???.net
間違えた
貿易風みたいな感じです

606 :ご冗談でしょう?名無しさん:2015/01/24(土) 09:05:45.91 ID:???.net
>>604
実験した訳じゃないから確かなことは言えんが、ならんだろうな。
偏西風や貿易風とは全くメカニズムが違うし。

607 :ご冗談でしょう?名無しさん:2015/01/24(土) 10:25:43.64 ID:???.net
そういうのって分野的には非平衡定常状態って言うんじゃないっけ

608 :ご冗談でしょう?名無しさん:2015/01/24(土) 10:47:16.74 ID:???.net
>>607
分野的が意味不明だがその通りだな。

609 :ご冗談でしょう?名無しさん:2015/01/24(土) 12:47:43.10 ID:???.net
球殻に接した部分は球殻と同じ速度になるから空気が流れ
球に接した部分は球と同じ速度になるから空気が静止するわけで
高度が上がるにつれて風が強くなる一方向の流れの層流みたいなモデルになると思うんだが
極が特異点になってるし他の動きも発生するかな

610 :ご冗談でしょう?名無しさん:2015/01/24(土) 16:51:34.57 ID:pOxirHPEj
全エネルギーEをもつ質量mの粒子がポテンシャルエネルギーU(x)の箱中を一次元で運動している。ポテンシャルエネルギーU(x)がx<0ではU∞、0≦x≦LではU=0、L<xではU=∞である。

(1)時間に依存しないシュレディンガー方程式をかけ
(2)0≦x≦Lでは方程式のひとつの解がΨ(x)=Asinknxであらわされることを示せ。
(3)境界値条件を用いてエネルギーEnを求めよ。
(4)ドブローイの関係式と波動関数の境界値条件から箱の中の粒子のエネルギー準位を求めよ


大学二年の化学科です。物理の量子力学を学習しているのですが、
家庭の事情でこの問題の範囲の授業に出席できませんでした。

どなたか解説解答していただけないでしょうか?
よろしくおねがいします。

611 :ご冗談でしょう?名無しさん:2015/01/25(日) 10:33:23.74 ID:Mj9VYdeZI
ググったらそのまんまがあるだろ

612 :ご冗談でしょう?名無しさん:2015/01/25(日) 11:53:19.84 ID:???.net
http://pbs.twimg.com/media/A9g0DFeCQAANL2H.jpg

613 :ご冗談でしょう?名無しさん:2015/01/25(日) 18:00:02.40 ID:???.net
600

614 :ご冗談でしょう?名無しさん:2015/01/25(日) 19:56:47.65 ID:???.net
>>609
緯度によって風速が違うので渦を生じる可能性があるけど低速ならそうなるな

615 :ご冗談でしょう?名無しさん:2015/01/25(日) 22:41:58.90 ID:a93cpFvp2
http://clubpenguin.web.fc2.com
貿易風みたいな感じです

616 :ご冗談でしょう?名無しさん:2015/01/25(日) 23:26:27.38 ID:???.net
>>375が死ねばみんな丸く収まる

617 :ご冗談でしょう?名無しさん:2015/01/25(日) 23:28:47.58 ID:???.net
>>409が死ねばみんな丸く収まる

618 :ご冗談でしょう?名無しさん:2015/01/26(月) 06:57:30.06 ID:???.net
京大、ガラスが確かに固体であることを示す有力な証拠を発見
http://daily.2ch.net/test/read.cgi/newsplus/1422215120/

619 :ご冗談でしょう?名無しさん:2015/01/26(月) 11:41:55.45 ID:Twk10Rxtf
それが有力な証拠なのかよ

620 :ご冗談でしょう?名無しさん:2015/01/27(火) 15:37:29.45 ID:RVNq7V8Hn
難系もってる方に質問です。
例題6の(3)なんですけど、M2の加速度はαーβって
なっているんですがなぜでしょうか?
α+βと思ったんですが
教えてくださいm(__)m

621 :ご冗談でしょう?名無しさん:2015/01/27(火) 19:19:12.82 ID:Q/9UslwB.net
完全弾性衝突では常に自分が相手にした仕事が自分が相手からされた仕事に等しいんですか?
何故ですか?

622 :ご冗談でしょう?名無しさん:2015/01/27(火) 20:05:35.97 ID:???.net
>>621
エネルギー保存則が成り立つから等しくなる。

623 :ご冗談でしょう?名無しさん:2015/01/28(水) 01:00:07.44 ID:???.net
>>621
非完全弾性衝突の場合は衝突時に音や光、熱としてエネルギーの一部が逃げていき、その結果、運動エネルギーの和が保存されない

624 :ご冗談でしょう?名無しさん:2015/01/28(水) 16:29:01.26 ID:???.net
半導体検出器だと外から飛んできた粒子が電子を伝導体のバンドに持ち上げて云々という
ふうに粒子から検出器側へのエネルギーの受け渡しが起こりますが、外から電子が
飛び込んできたり入射粒子が蹴ったδ線がエネルギーを半導体側に与え尽くした結果
どの電子のギャップ間のエネルギーにも飛んでる電子のエネルギーが符合せずしかも自分は
禁止帯のエネルギーになっているという状態にたまたま陥ってしまったら
その電子はどうなるんでしょうか?

ほかの効果でエネルギーをちびちび失って許容帯に落ちてこれるまで原子に捕獲されずに
吸収されない光子のようにひたすら散乱を繰り返すことになるんでしょうか?

625 :ご冗談でしょう?名無しさん:2015/01/28(水) 17:53:55.34 ID:???.net
>>624
> しかも自分は
> 禁止帯のエネルギーになっているという状態にたまたま陥ってしまったら
陥らないから禁止帯なんだろアホか

626 :ご冗談でしょう?名無しさん:2015/01/28(水) 18:44:23.73 ID:???.net
>>625
完全に周期ポテンシャルに束縛された定常状態の電子なら許容帯にしか
行けないのはわかるのですが、外から中途半端なエネルギーを持って
飛び込んできた電子が時間発展的にどんな経緯を辿って周期ポテンシャル中に
落ち着くのかなという部分が気になりまして。

627 :ご冗談でしょう?名無しさん:2015/01/28(水) 20:47:49.80 ID:???.net
virtual state でググれ

628 :ご冗談でしょう?名無しさん:2015/01/28(水) 22:42:59.93 ID:???.net
>>626
そりゃフォノンとエネルギー交換しながら散乱されるんだから
だんだんエネルギー失ってって最終的に基底状態に落ちるのさ

629 :ご冗談でしょう?名無しさん:2015/01/29(木) 01:06:59.34 ID:???.net
少し変な質問ですがここでさせてもらいます。
何年か前、物理の有名予備校講師が講義のなかで
「未来を予言することは不可能」
ということを言っていました。そして、その説明をしていたのは保存力についての講義の中であったと曖昧ながら記憶しています。
今になって思うと量子力学の非決定論のお話をしていたのかなあと思うのですが、これと保存力の(経路によらないなどの)説明に関係性が見いだせません。もしこの関係性がお分かりになるかたいましたら教えてください。聞き間違えの頓珍漢かもしれないのですが…。
頭に引っ掛かってずっと気持ちが悪いのでお願いします。

630 :ご冗談でしょう?名無しさん:2015/01/29(木) 01:07:42.50 ID:???.net
量子論は決定論だが

631 :ご冗談でしょう?名無しさん:2015/01/29(木) 01:30:19.53 ID:???.net
>>629
予言の困難性の核心は非線形系に現れるカオス的振る舞いだよ。
バタフライ効果と言えば聞いたことがあるかもしれないが、
非線形系においては微小な初期条件の差が時間発展の末に
根本的な振る舞いの差になって表れてくる。
よって未来を予言するには現在を無限の精度で観測する必要がある。
そして、それは不可能である。
つまり、未来を予言するのは不可能だということ。

その先生は三体問題の話をしてたんじゃないかな?

632 :ご冗談でしょう?名無しさん:2015/01/29(木) 01:52:45.19 ID:???.net
>>631
ご回答ありがとうございます。長年のトゲが溶けていくようでとても嬉しいです。
つまり、未来の予言が不可能だということと、多体問題は数学的に厳密に解くことができないということは同値であるということですか?
いかんせん薄い記憶なもので、そういえば熱力学のお話のときにアボガドロ数個の粒子が〜というような話をしていたので、その時にこのお話が出たのかもしれません。
保存力というキーワードと未来の予言というキーワードが妙に頭から離れなかったのですがこの二つは関係ないと考えてよろしいのでしょうか。
本当に訳のわからない質問で申し訳ありません。。。

633 :ご冗談でしょう?名無しさん:2015/01/29(木) 02:04:37.91 ID:???.net
>>632
> つまり、未来の予言が不可能だということと、多体問題は数学的に厳密に解くことができないということは同値であるということですか?
同値ではない。厳密に解けたとしてもカオス的な振る舞いは変わらない。
もちろんカオス的な振る舞いをするからこそ(数値的にすら)解くのが難しい側面はあるが。

> 保存力というキーワードと未来の予言というキーワードが妙に頭から離れなかったのですがこの二つは関係ないと考えてよろしいのでしょうか。
関係ない。散逸力が関与するか否かによって予言の困難性は変わらない。

634 :ご冗談でしょう?名無しさん:2015/01/29(木) 02:33:06.24 ID:???.net
>>633
ご回答ありがとうございます。
非常にスッキリしました!理解は追い付いていませんが!(笑)
物理とそれに関係した数学にはとても興味があるので、これから追い付けるように勉強したいと思います。
今まで、保存力だから経路によらないからどこいくかわからなくてーなどとわけのわからない理論で納得しようとしていましたがご回答いただけて良かったです。
ありがとうございました。

635 :ご冗談でしょう?名無しさん:2015/01/29(木) 03:20:51.21 ID:???.net
エネルギーの散逸があると初期条件によらず
同一の状態へと変化していくという、カオスとは逆の振る舞いをするから
「保存力」というのは重要なキーワードなんじゃないか?
散逸系でも外部から強制力が働きカオスになる場合や、
散逸性と非線形性がバランスする場合があるから必須というわけではないだろうけど。

636 :ご冗談でしょう?名無しさん:2015/01/29(木) 07:52:07.07 ID:???.net
>>628
ありがとうございます。

637 :ご冗談でしょう?名無しさん:2015/01/29(木) 09:49:10.58 ID:???.net
お前「来年は合格できますかね?」
予備校講師「未来を予言することは不可能」

638 :ご冗談でしょう?名無しさん:2015/01/29(木) 11:24:32.05 ID:???.net
>>635
回答ありがとうございます。
できればもう少し分かりやすく教えていただけますでしょうか。
アホなもので申し訳ありません

639 :ご冗談でしょう?名無しさん:2015/01/29(木) 13:22:21.75 ID:8QUifFR2V
君に何が分かるか他人に分かるわけが無かろう
まず自分に分かる事を説明しろ

640 :ご冗談でしょう?名無しさん:2015/01/29(木) 13:27:53.65 ID:???.net
二股に別れた経路で簡単に説明していたような記憶があります

641 :ご冗談でしょう?名無しさん:2015/01/29(木) 13:57:14.50 ID:???.net
>>635
それは平衡状態を予測してるだけで時間発展を予測してる訳じゃないから除外したんだが、
まあ「予言」という言葉のとりかたしだいか。
「十分な時間の経った後お前は既に死んでいる」のような類いの言説も予言であるには違いないとすれば。

642 :ご冗談でしょう?名無しさん:2015/01/29(木) 14:37:42.93 ID:???.net
>>641
元の話をみると、この講師の話は時間発展を正確に追えるかというより
ハミルトン系でのリウビルの定理やエルゴード性に関する話みたいじゃない?
なので保存力が強調されたのだと思うけどな。

643 :ご冗談でしょう?名無しさん:2015/01/29(木) 15:30:10.06 ID:???.net
保存力というのはただの記憶違いかもしれないので、未来の予言ていうことの不可能性については多体問題の時間発展で考えるということでいいですか?
平行状態で〜という話ではなく、ある系の時間発展についての予言という話だったと記憶しています。

644 :ご冗談でしょう?名無しさん:2015/01/29(木) 20:16:54.22 ID:???.net
勘違いしないように言っておくけど
初期値のずれに対して十分遠い未来の予測が不可能なわけで
方程式が解けなくてもある誤差の幅を持たせた短期的な未来予測は数値解析で可能だからね

645 :ご冗談でしょう?名無しさん:2015/01/29(木) 20:32:27.11 ID:v0JSGVA3.net
時間は幻想である。差分が出るから。行列とベクトル解析すれば解る。登記

646 :ご冗談でしょう?名無しさん:2015/01/29(木) 20:35:52.48 ID:???.net
まあでも散逸力に予言の困難性が関係ないというのは間違いか。
散逸力の根源はエントロピーの増加、つまり情報の欠落だから、
散逸力は予言の困難性から生じるということもできるか。
無限の情報を持っている存在にとって全ての力は保存力なんだから。

647 :ご冗談でしょう?名無しさん:2015/01/29(木) 20:46:30.65 ID:v0JSGVA3.net
時間は幻想である。差分が出るから。行列とベクトル スクエア ベクトル すれば解る。登記

648 :ご冗談でしょう?名無しさん:2015/01/29(木) 21:07:25.83 ID:???.net
>>644
カオスの説明のなかに初期値の僅かなずれが〜という説明と初期値が厳密にわかったとしても完全解析は不可能という二つの説明があると思うのですがこれは同じことをいっているんですか?

649 :ご冗談でしょう?名無しさん:2015/01/29(木) 21:18:35.19 ID:???.net
>>646
さすがに予備校の授業中のことなのでそこまでの議論はなかったですが、いろいろと参考になります。
いろいろと自分の中で整理していくと、恐らく多体問題のお話の中で出てきた話であり、仮に保存力の話をしていた時に予言の話があったとした場合は、あるポテンシャルに遷移するとき、その運動はカオスになり解析できないから無数にある経路を予言することはできない。
といったところでしょうか。

650 :ご冗談でしょう?名無しさん:2015/01/29(木) 21:42:38.64 ID:???.net
多体問題=三体問題のイメージがあるから違和感を拭えない

651 :ご冗談でしょう?名無しさん:2015/01/29(木) 22:35:48.60 ID:???.net
>>649
その通り。そして、個々の物体を解析するのを諦めた先にエントロピーという概念があり、
熱や自由エネルギーといった熱力学的諸量へと繋がっていく。

652 :ご冗談でしょう?名無しさん:2015/01/29(木) 23:17:13.06 ID:c8mOeA3W.net
>>623 定性的な説明はいいので定量的な説明をおねがいします

653 :ご冗談でしょう?名無しさん:2015/01/29(木) 23:26:05.10 ID:???.net
===回答者へ===
・丸投げは専用スレに誘導
・不快な質問は無視、構った方が負け
・質問者の理解度に応じた適切な回答をよろしく
・単発質問スレを発見したらこのスレッドへの誘導をよろしくね
・逆に議論が深まりそうなら新スレ立てて移動するのもあり
・板違いの質問は適切な板に誘導を
・不適切な回答は適宜訂正、名回答は素直に賞賛

654 :ご冗談でしょう?名無しさん:2015/01/29(木) 23:36:11.12 ID:???.net
M^(abc) = M^(acb) = -M^(bac)
が成り立つ時
M^(abc)=0
になる
そのことを導出するのに結構式変形したんだけど、何か直感的にすぐわかる方法ないかな?

655 :ご冗談でしょう?名無しさん:2015/01/29(木) 23:52:23.81 ID:???.net
M, a, b, c ってなんだよ

656 :ご冗談でしょう?名無しさん:2015/01/30(金) 00:08:43.76 ID:???.net
任意のテンソルと添え字です。
別に3変数関数だったとしても、この対称性と反対称性があれば恒等的に0になるはずだよ。

657 :ご冗談でしょう?名無しさん:2015/01/30(金) 00:11:32.79 ID:???.net
>>652
質量mの物体A、質量Mの物体Bを用意して、Aが速度vでB(速度V)に衝突する。衝突後の速度はそれぞれv'、V'とする。
この時完全弾性衝突したとすればe=-(V'-v')/(V-v)=1
⇔V'-v'+V-v=0
⇔V'=-V+v+v' (T式)
⇔v'=-v+V+V' (U式)

ここで
mv+MV=mv'+MV' (運動量保存則)
にT式U式を代入すれば
v'=((m-M)v+2MV)/(m+M)
V'=(2mv+(M-m)V/(m+M)

を用いれば
K'=1/2・mv'^2+1/2・MV'^2
 =1/2・mv^2+1/2MV^2
 =K
となり、衝突の前後において運動エネルギーは保存されている。

658 :ご冗談でしょう?名無しさん:2015/01/30(金) 00:44:22.51 ID:???.net
>>654
後ろの式でサイクリックに回すとマイナスがでるとわかってる。
三回サイクリックに回すと元に戻る。

こんだけでいいんでは。

659 :ご冗談でしょう?名無しさん:2015/01/30(金) 00:56:04.17 ID:???.net
>>658
おー、ありがとう!
なるほど、それはすごくわかりやすい。
オッカムの剃刀という言葉を実感したよ。
二つ目の等号を見つけた時点で、最初の等号はもう考えなくて良かったのか。

660 :ご冗談でしょう?名無しさん:2015/01/30(金) 01:08:11.14 ID:???.net
最初の関係がないとサイクリックにならないだろ

661 :ご冗談でしょう?名無しさん:2015/01/30(金) 01:20:45.64 ID:???.net
M(acb)=-M(bac)より
M(bac)=-M(cba)
M(cba)=-M(acb)
よって
M(acb)=((-1)^3)M(acb)=0
で問題ないぞ

662 :ご冗談でしょう?名無しさん:2015/01/30(金) 01:27:05.68 ID:???.net
対称性 M^(abc) = M^(acb)
反対称性 M^(abc) = -M^(bac)
これより、
M^(acb) = -M^(bac)
とサイクッリックに回すと−1倍されることがわかる。
っていうのに気付けなかったんだよね。
対称性と反対称性からダイレクトに0になることがわかるんじゃないかと脳みそが固まってしまった。

663 :ご冗談でしょう?名無しさん:2015/01/30(金) 01:29:26.24 ID:???.net
初期値が厳密にわかっても〜
→そもそも一般の多体問題が解析的に(微分方程式の解という形で)解けないということ

初期値の僅かなズレが〜
→初期値の僅かなズレが時間発展で指数関数的に増大するから、数値解析(あきらめて誤差を認めてシミュレーションすること)でも注意が必要なこと

664 :ご冗談でしょう?名無しさん:2015/01/30(金) 05:54:38.80 ID:???.net
>>650
三体問題も十分カオスじゃん

665 :ご冗談でしょう?名無しさん:2015/01/30(金) 11:46:19.86 ID:???.net
a b c
|├┤
├┤|
|├┤
├┤|
|├┤
├┤|
a b c

666 :ご冗談でしょう?名無しさん:2015/01/30(金) 11:52:36.85 ID:???.net
星の場合のよくある三体の挙動って結局
AとBの連星系にCが飛んでくる
→@Cがそのままとんでいく
→AAとCの連星系が形成されBがとんでいく
のどっちかなんだよね

667 :ご冗談でしょう?名無しさん:2015/01/30(金) 13:45:19.48 ID:bSaknb7w5
カオスでねーじゃん

668 :ご冗談でしょう?名無しさん:2015/01/31(土) 15:06:00.32 ID:???.net
ムーアの法則は今でも成り立ってますか?
今後どうなると思いますか?

669 :ご冗談でしょう?名無しさん:2015/01/31(土) 15:34:13.36 ID:???.net
宿題は自分で考えな

670 :ご冗談でしょう?名無しさん:2015/02/01(日) 12:46:11.81 ID:bihn93oxo
今でも成り立つ
いずれは対象が意味不明になって再定義

671 :ご冗談でしょう?名無しさん:2015/02/01(日) 13:23:13.93 ID:???.net
素粒子実験?とかの機械装置の話では、よく「エネルギーが大きい」などの表現をしますが。
このエネルギーとはなんでしょうか?
電子を光の速度で飛ばして正面衝突させる話ですけど、
エネルギーとはスピードのことをいうの?

672 :ご冗談でしょう?名無しさん:2015/02/01(日) 13:25:26.16 ID:???.net
>>671

===質問者へ===
重要 【 丸 投 げ 禁 止 】

・質問する前に
1. 教科書や参考書をよく読む
2.
http://www.google.com/
  などの検索サイトを利用し、各自で調べる

673 :ご冗談でしょう?名無しさん:2015/02/01(日) 13:46:26.30 ID:???.net
調べた末にわからない問題なら、ちょっとした物理の質問じゃなくなるじゃねぇかよ

674 :ご冗談でしょう?名無しさん:2015/02/01(日) 14:11:59.41 ID:???.net
>>671
http://kids.kek.jp/accelerator/

675 :ご冗談でしょう?名無しさん:2015/02/01(日) 14:14:32.55 ID:???.net
>>673
事前に調べておけば自分がどこに納得いかないのかを説明できる。
それすら出来ない質問者に納得できる解答など存在するはずもない。

676 :ご冗談でしょう?名無しさん:2015/02/01(日) 15:10:11.86 ID:???.net
めぶ、じぇぶ、てぶ

677 :ご冗談でしょう?名無しさん:2015/02/01(日) 15:10:59.60 ID:???.net
けぶ、めぶ、じぇぶ、てぶ、ぺぶ

678 :ご冗談でしょう?名無しさん:2015/02/01(日) 19:42:47.67 ID:???.net
神様と魔王はどっちの方が強いのでしょうか?

679 :ご冗談でしょう?名無しさん:2015/02/01(日) 23:42:00.77 ID:???.net
散乱断面積等の最新の実験データが欲しい場合、どこのWebサイトを参照するのが良いでしょうか?

ニュートリノ+電子 → ニュートリノ+電子
の散乱や
電子+電子 → ミューオン+ミューオン
の散乱のデータが欲しいです

680 :ご冗談でしょう?名無しさん:2015/02/01(日) 23:53:08.25 ID:???.net
アイソトープ手帳

681 :ご冗談でしょう?名無しさん:2015/02/02(月) 00:01:27.67 ID:z3g4OFqE.net
磁場はなくて時間変化する電場のみが存在する状態
あるいは逆に電場はなくて時間変化する磁場のみが存在する状態って両方ともありえませんよね?
真空中として

682 :ご冗談でしょう?名無しさん:2015/02/02(月) 00:08:13.67 ID:???.net
あり得ないよ。
両者は本来同じもの。
其れを物理世界では、二つの方向から見ているに過ぎないから。

683 :ご冗談でしょう?名無しさん:2015/02/02(月) 00:11:44.39 ID:???.net
なるほど
ありがとうございました

684 :ご冗談でしょう?名無しさん:2015/02/02(月) 01:27:55.28 ID:???.net
実は、電場にはベクトルの大きさが距離の二乗に反比例するものと、そうで
ないものとがある。電磁波は後者で、距離の一乗に反比例するものや、
距離に依存しないものもある。

前者はたとえ時間変動しても必ずしも磁場を伴わない場合があると思われる。

685 :665:2015/02/02(月) 01:31:45.55 ID:???.net
もとい。

前者でも全く磁場を伴わない場合は無いかもしれないが、電磁波の場合の
電場と磁場の(直交)関係とは違う場合があると思われる。

686 :ご冗談でしょう?名無しさん:2015/02/02(月) 01:51:58.12 ID:???.net
>>684
Maxwell方程式とどう折り合いつけるんだ?

687 :ご冗談でしょう?名無しさん:2015/02/02(月) 02:16:41.69 ID:???.net
宇宙は無数に存在するのでしょうか?
それとも、一つだけでしょうか?

688 :ご冗談でしょう?名無しさん:2015/02/02(月) 02:22:15.60 ID:???.net
ある特定の方向から来た光だけを選択的に曲げるようなフィルターを作ることは可能ですか?

689 :ご冗談でしょう?名無しさん:2015/02/02(月) 02:27:55.55 ID:???.net
極端に視野角の狭いモニタがあったとして、それの逆の作用
(正面からは見えないが、少し角度をずらすと見えはじめる)ようなもののことです

690 :ご冗談でしょう?名無しさん:2015/02/02(月) 11:51:58.44 ID:???.net
すっげーいまさらな話ですが
てこの原理って支点・力点・作用点の相対位置関係が変化しないという前提ですよね?

691 :ご冗談でしょう?名無しさん:2015/02/02(月) 13:34:20.21 ID:???.net
>>690
瞬間瞬間でテコの原理を考えることはできる

692 :ご冗談でしょう?名無しさん:2015/02/02(月) 14:19:43.90 ID:???.net
>>691
ありがとう。
相対位置が変化する道具の例とかあります?

693 :ご冗談でしょう?名無しさん:2015/02/02(月) 14:53:03.46 ID:???.net
>>692
昔ながらの天秤棒を使う秤とかは?

694 :ご冗談でしょう?名無しさん:2015/02/02(月) 15:08:15.84 ID:???.net
>>693
それは使用している最中には変化しないでしょ。

695 :ご冗談でしょう?名無しさん:2015/02/02(月) 15:20:17.94 ID:???.net
>>694
いや変化させて釣り合わせて測るんだが、
ちょっと型は違うがこんなやつ。
http://ja.wikipedia.org/wiki/%E5%A4%A9%E7%A7%A4%E3%81%B0%E3%81%8B%E3%82%8A#mediaviewer/File:Fairbanks_person_weigh_scale.jpg

696 :ご冗談でしょう?名無しさん:2015/02/02(月) 15:39:08.75 ID:???.net
>>695
んー、探してもらって恐縮だけど、釣り合う位置を探る過程で変化させるだけで一連の動作の中では変化してないですよね。
力点に力をかけたら作用点で反応するまでに各点の位置関係が変わるような道具があるかどうかが知りたいです。

697 :ご冗談でしょう?名無しさん:2015/02/02(月) 15:53:15.37 ID:???.net
>>696
んー、お題の「支点・力点・作用点の相対位置関係が変化」する道具という意味では、
測る過程で位置関係が変化する正にそのままの道具なんだが。
あと
>力点に力をかけたら作用点で反応するまでに各点の位置関係が変わる
ってどういう意味?力の伝わる弾性波の速度以上の速さで位置を変えるという話なら、
さすがに>>691も成り立たないよ。

698 :ご冗談でしょう?名無しさん:2015/02/02(月) 15:58:48.28 ID:???.net
ついでに静的な力の釣り合いでなく仕事の伝達におけるトルクの変化なんかを考えてるんなら、
CVT(いろんなタイプが有るが)はそういう装置。

699 :ご冗談でしょう?名無しさん:2015/02/02(月) 16:12:38.04 ID:???.net
>>697
>>瞬間瞬間でテコの原理を考えることはできる
ここ何故かIDが見えないんで貴方とは特定できないんだけど、こういう返信があったので変化する道具があるのかと。

ええと、意図としては力点・支点・作用点の順番はともかくこの三つの点が一つの物体上にない物はあるのかという質問です。
例えば力点・支点・作用点で並んではいるが「力点と支点」「支点と作用点」がそれぞれ別の部品にあり、繋がってはいるが支点が可動であるというような道具はありますか、ということです。
道具次第では力点・作用点・支点というものもありますが、↑のは例ということで。
それが>>690
「てこの原理って支点・力点・作用点の相対位置関係が変化しないという前提ですよね?」
の質問の意図となります。

700 :ご冗談でしょう?名無しさん:2015/02/02(月) 16:17:05.30 ID:???.net
微分を使用する際dxやdyやdzという量をよく置きますが
これはdx、dy、dzがマイナスであっても良いものなのですか?

701 :ご冗談でしょう?名無しさん:2015/02/02(月) 16:25:10.86 ID:???.net
書き方が悪かったですね。
「てこの原理は力が働いている間に力点・支点・作用点の位置関係が変化しない前提ですよね?」
と書けば良かったかな。

>>700
基本的にdxとかdyとかは変数だと思っていたから中身は関係ないんじゃないかな・・・と思うんですがどうでしょ。

702 :ご冗談でしょう?名無しさん:2015/02/02(月) 16:30:37.80 ID:???.net
>>701
微小な変化量をΔxとおいてそれを無限に小さくするときにdxになりますよね
この変化量というものは変化の向きを定めればそれに対してプラスに変化するかマイナスに変化しますが
マイナスに変化する場合はマイナスを含めた変化量をdxと置くのかどれぐらい変化したかの絶対値をdxとおくのか分からなかったので質問しました

703 :ご冗談でしょう?名無しさん:2015/02/02(月) 16:42:34.92 ID:???.net
>>702
変化量の絶対値をdxとおくのでしたら
∫[0,1]xdx = ∫[1,0]xdx
となりますね。

704 :ご冗談でしょう?名無しさん:2015/02/02(月) 16:58:39.40 ID:???.net
>>700
勿論正負どちらでもよい。
片側極限の値がそれぞれ異なるような関数は通常微分可能とは言わない。

705 :ご冗談でしょう?名無しさん:2015/02/02(月) 17:11:20.88 ID:???.net
dx、dyなんかの数学的な合理化には、関数を線形近似したときの変数という合理化や
接空ベクトルの双対ベクトル(微分形式)としての合理化なんかがある。
前者ではdxはただの変数だから大きさや符号は好きなようにとれる。

706 :ご冗談でしょう?名無しさん:2015/02/02(月) 17:12:37.22 ID:???.net
座標系をとったとき逆向きに進行方向を持つだけだからね
ところで、>>689のような素材はありませんか

707 :ご冗談でしょう?名無しさん:2015/02/02(月) 17:21:17.98 ID:???.net
>>706
市松模様の格子2枚を位相をずらして配置すればそういう機能を待たせられるが、
実際そういう素材が有るかは知らない。

708 :ご冗談でしょう?名無しさん:2015/02/02(月) 17:28:05.97 ID:???.net
ありがとうございます
シェルピンスキー四面体でもいけそうですね
角度の局所性をもたせるには市松の大きさと両者の距離の比を調整すればいいわけですね
しかし他の角度だとほぼ100パーセント透過、というのは無理でしょうか

709 :ご冗談でしょう?名無しさん:2015/02/02(月) 17:56:00.05 ID:???.net
>>708
用途が分かりそうで分からんw
極端にすればモニター2つを角度変えて配置し真ん中だけ開けておく・・・・みたいなことをすれば視覚的には可能だけど。
フィルターだと特定の屈折率(角度)でしか透過しない素材・・・・・ではダメだね。

710 :ご冗談でしょう?名無しさん:2015/02/02(月) 18:09:06.55 ID:???.net
ひなかわいいの〜

711 :ご冗談でしょう?名無しさん:2015/02/02(月) 18:12:54.95 ID:???.net
遮光円盤を使わずにコロナ観察とか
向きのきまったバックグラウンドを殺す方法はないのかなあ、と

712 :ご冗談でしょう?名無しさん:2015/02/02(月) 21:30:54.43 ID:???.net
>>711
http://i.imgur.com/3xCjGvF.jpg

713 :ご冗談でしょう?名無しさん:2015/02/02(月) 22:53:37.67 ID:???.net
モーメントのつりあいで壁に棒が立てかけられてる・・という問題で
力に対して距離を垂直にとるのか、距離に対して力を垂直にとるのか、どっちが正しいやり方ですか?

714 :ご冗談でしょう?名無しさん:2015/02/02(月) 23:02:22.90 ID:???.net
>>713
計算結果いっしょやん

715 :ご冗談でしょう?名無しさん:2015/02/02(月) 23:07:36.67 ID:???.net
ドイツ最大の物理学者は誰ですか?

716 :ご冗談でしょう?名無しさん:2015/02/02(月) 23:17:23.41 ID:???.net
>>714
あ、いや計算結果は同じなのですが
どっちの考え方だとここで詰まる、みたいなことがあるのかと思いまして
難問は慣らした方法でやりたいので

717 :ご冗談でしょう?名無しさん:2015/02/02(月) 23:21:09.73 ID:???.net
>>716
計算結果一緒というのはつまるかつまらないかも含めて一緒。

まぁそもそもつまるところなんてないが。

まとめる時にまとめやすい方でやればいい。
こんな事気にしても益はない。

718 :ご冗談でしょう?名無しさん:2015/02/02(月) 23:27:38.55 ID:???.net
>>717
ありがとうございます
自分は力に合わせた方が好きなのでそっちでやります

719 :ご冗談でしょう?名無しさん:2015/02/03(火) 00:45:12.34 ID:???.net
700

720 :ご冗談でしょう?名無しさん:2015/02/03(火) 00:50:55.48 ID:kkQ+rXWZ.net
力を合わせて合力

721 :ご冗談でしょう?名無しさん:2015/02/03(火) 01:04:25.14 ID:???.net
調弦理論の話なんですけど
畳み込まれた次元の中に何らかの情報体や物質などが入っていると考えることはできますか?

722 :ご冗談でしょう?名無しさん:2015/02/03(火) 01:07:41.30 ID:???.net
考えるだけなら>>721の自由じゃね?

723 :靖国参拝、皇族、国旗国歌、神社神道を異常に嫌うカルト:2015/02/03(火) 01:18:08.38 ID:vmm+byMF.net
★マインドコントロールの手法★

・沢山の人が偏った意見を一貫して支持する
 偏った意見でも、集団の中でその意見が信じられていれば、自分の考え方は間違っているのか、等と思わせる手法

・不利な質問をさせなくしたり、不利な質問には答えない、スルーする
 誰にも質問や反論をさせないことにより、誰もが皆、疑いなど無いんだと信じ込ませる手法


偏った思想や考え方に染まっていたり、常識が通じない人間は、頭が悪いフリをしているカルト工作員の可能性が高い


10人に一人はカルトか外国人

「ガスライティング」で検索を!
.....

724 :ご冗談でしょう?名無しさん:2015/02/03(火) 01:29:45.12 ID:???.net
>>722
自分でも書いててそう思ったけどww
言葉のあーや

何らかの矛盾が出てくるか否か

725 :ご冗談でしょう?名無しさん:2015/02/03(火) 12:51:08.22 ID:6vU7VP540
>>721
入ってるに決まってる
T-双対なんて入ってる弦が巻き付かなきゃ出来ない話だ

726 :ご冗談でしょう?名無しさん:2015/02/03(火) 18:45:57.32 ID:2PXlDo6l.net
国内および海外の
メジャーなフォーラムをご教示ください。

海外では、
www.physicsforums.com以外では
何が有名でしょうか。

宜しくお願いします。

727 :ご冗談でしょう?名無しさん:2015/02/03(火) 19:58:46.03 ID:UXKp3D8a.net
ビリアル定理について質問なんですが、

http://ja.wikipedia.org/wiki/%E3%83%93%E3%83%AA%E3%82%A2%E3%83%AB%E5%AE%9A%E7%90%86

Wikipediaに乗っているビリアル定理の証明のところで、
G=Σr・p
としていて、そのあとにGについて極限をとっていると思うんですが、
なんで時間T→∞だとGは有限だといえるんですか?
G=Σr・pはrは位置ベクトルで範囲は有限であるとしているのでわかるのですが、
運動量pに関してはp=mvで表せられるので質量mも有限であるのであるのですが、
速度vは有限であるとは言えないので、Gの極限は0であるとはいえないんじゃないでしょうか?

よろしくお願いします。

728 :ご冗談でしょう?名無しさん:2015/02/03(火) 21:03:02.25 ID:???.net
>>727
つ エネルギー保存則

729 :ご冗談でしょう?名無しさん:2015/02/03(火) 21:05:25.73 ID:???.net
そもそも今現在有限なものが何故時間発展の後に無限になりうるなどと思うのか。
無限の時間積分したわけでもないのに。

730 :ご冗談でしょう?名無しさん:2015/02/03(火) 21:06:06.24 ID:UXKp3D8a.net
>>728
返信ありがとうございます!
もう少し詳しく説明していただけるとありがたいです。
けっこう、物理に関して未熟なもので、、、

731 :ご冗談でしょう?名無しさん:2015/02/03(火) 21:14:23.30 ID:UXKp3D8a.net
≥>729
教授が言うには、加速器では無限に速さをあげることができるじゃないかということを言われました。
しかし、加速器の中でもビリアル定理は成り立つから、
GについてT→∞の極限をとっても=0にはなるとは言えない、と言われまして、
なぜ、=0 と言えるのか論理的に説明しなさいと言われました。

732 :ご冗談でしょう?名無しさん:2015/02/03(火) 21:47:15.72 ID:???.net
ガウスとロスチャイルドはどっちの方が凄いのでしょうか?

733 :ご冗談でしょう?名無しさん:2015/02/04(水) 07:42:59.04 ID:???.net
>>731
> >729
> 教授が言うには、加速器では無限に速さをあげることができるじゃないかということを言われました。
できるかアホ。
> しかし、加速器の中でもビリアル定理は成り立つから、
> GについてT→∞の極限をとっても=0にはなるとは言えない、と言われまして、
0になるわけないだろアホ。
> なぜ、=0 と言えるのか論理的に説明しなさいと言われました。
有限の値になると言ってるんだよアホ。

734 :ご冗談でしょう?名無しさん:2015/02/04(水) 10:04:39.85 ID:???.net
仮に左辺が無限大になるとしたら、十分大きいtに対して、G>at^2と書くことができる。位置は有限なので、運動量が時間の二次関数以上の関数と言える。運動量を時間微分すると力になるので、力がいくらでも大きい値を取れなければならないと言える。
エネルギー保存則で言えば、無限大の運動量を与えるには無限大のエネルギーを与えないといけないので、無理と言える。

735 :ご冗談でしょう?名無しさん:2015/02/04(水) 10:21:09.35 ID:???.net
もう少しちゃんと言えば、エネルギーが有限なので、Gも有限。Gの最大値をMとするとGの積分は必ずMt以下となる。故に時間平均すると有限となる。

736 :ご冗談でしょう?名無しさん:2015/02/04(水) 10:33:54.75 ID:???.net
>>734
>仮に左辺が無限大になるとしたら、十分大きいtに対して、G>at^2と書くことができる
無限大になると主張しているのは t -> ∞ の極限だからそれが成り立つとは言えない

>無限大の運動量を与えるには無限大のエネルギーを与えないといけないので、無理と言える
エネルギーではなく力積.これは t -> ∞ で無限大になりうる

737 :ご冗談でしょう?名無しさん:2015/02/04(水) 11:02:44.20 ID:???.net
>>736
無限大になるというのは、任意の実数Mよりも、十分大きい時間tをとれば大きくなるという意味じゃんか。
まあat^2じゃなくて、at^p(p>1)だったけど。
十分大きいtで、G(t)/t > Mより、
G(t) > Mt^p(p>1)
と言えると思うけど。

運動量が無限になれば、運動エネルギーは無限になるじゃないか。

738 :ご冗談でしょう?名無しさん:2015/02/04(水) 11:07:08.96 ID:???.net
力積だけじゃなく、仕事も同じところをぐるぐる回れば、無限大の大きさになりうるよね。

739 :ご冗談でしょう?名無しさん:2015/02/04(水) 12:46:00.35 ID:???.net
力積や仕事が無限大でも運動量やエネルギーが無限大になるわけではない

740 :ご冗談でしょう?名無しさん:2015/02/04(水) 13:33:48.51 ID:???.net
>>739
どういうこと?力積も仕事もそれぞれ運動量とエネルギーの差に等しいでしょ?

741 :ご冗談でしょう?名無しさん:2015/02/04(水) 14:16:37.09 ID:???.net
>>731
加速器は外部からエネルギー入ってきてるからビリアル定理なんか成り立たないよ。
>>738
その場合も外力働きまくってるからビリアル定理なんか成り立たないよ。

742 :ご冗談でしょう?名無しさん:2015/02/04(水) 14:57:13.45 ID:???.net
ビリアルが有界ならビリアル定理は成り立つ
ビリアルが有界であれば外部からエネルギーを貰おうが関係なし

743 :ご冗談でしょう?名無しさん:2015/02/04(水) 15:37:34.12 ID:???.net
ビッ●カメラ札幌店の佐藤伸弦が暴行事件を起こしていた

744 :ご冗談でしょう?名無しさん:2015/02/05(木) 03:20:59.95 ID:6wHv8E0D.net
相対論で出てくる粒子数束n u^μ (nは粒子数密度、u^μは四限速度)のu^μに関するリー微分
Lu(nu)は成分で書くとどう書いたら良いのでしょうか

745 :ご冗談でしょう?名無しさん:2015/02/05(木) 03:46:24.05 ID:ns3jwdcmA
四元ポテンシャルから電磁場出すのと同じだろ

746 :ご冗談でしょう?名無しさん:2015/02/05(木) 09:10:19.52 ID:/uGxAN5+.net
光ファイバーを伝わる光の群速度はグースヘンシェンシフトの染み出し長にどのように依存しますか?

747 :ご冗談でしょう?名無しさん:2015/02/05(木) 14:38:36.03 ID:???.net
>>744
Lu = diu + iud
を使えばいいだけでは

748 :ご冗談でしょう?名無しさん:2015/02/05(木) 17:55:27.62 ID:aZRgu+pn.net
>>747
それを成分で書くとどうなるんでしょう

nを4form, uをベクトルと思ってやるのでしょうか

749 :ご冗談でしょう?名無しさん:2015/02/05(木) 22:03:34.24 ID:UFvlAiyJ.net
運動量の定義って何ですか?
あと運動量の単位はkg・m/sとJ・s/mがありますが、
速さvで水平方向に動いてる質量mの物体があるとしたら運動エネルギーは1mv^2/2、運動量はmv
だけど運動エネルギーを速さで割っても運動量に一致しません
どういうことですか?

750 :ご冗談でしょう?名無しさん:2015/02/05(木) 22:13:10.18 ID:???.net
>>749
ラグランジアンを速度で微分したものが運動量です。

751 :ご冗談でしょう?名無しさん:2015/02/05(木) 22:14:10.96 ID:UFvlAiyJ.net
>>750
大学受験生でも分かるように教えてください

752 :ご冗談でしょう?名無しさん:2015/02/05(木) 22:28:40.44 ID:???.net
>>749
運動方程式は運動量の時間変化が力となっている。有限の時間冲で考えると
(mv) / 冲 = F
(mv) = F冲 で右辺は力積。この式はよく使われる。

力Fが行った仕事は F*v冲 = v(mv) = (1/2mv^2) = 凵i運動エネルギー)
微分と同じことですが、教科書はこんな具合だったかな?

753 :ご冗談でしょう?名無しさん:2015/02/05(木) 23:17:43.65 ID:???.net
>>749
>>751
定義は普通に p = mv だよ。
運動エネルギーとの関係は割り算じゃなくて微分・積分。
v^2 を v で微分すれば 2v になるので mv^2/2 の微分は mv になる。
このことは v を ε だけ変化させたとき、v^2 の変化の割合が
 ((v+ε)^2 - v^2)/ε = 2v + ε
となることと、v の変化量 ε が充分小さいときに上の式が 2v に近似できることから確認できる。

なんで p = mv なのかというと、はじめに操作的な量として力積を考えたとき、
それに付随して系が持つべき量として得られる物理量がそういう形をしているから。

ラグランジアンうんぬんはかいつまんで言うと、
運動方程式を与える関数としていろんな形のエネルギーやそれに似たものを定義することができて、
それをひっくるめたものがラグランジアンと呼ばれている。
適当な(物理的に正当性のある)条件の下でラグランジアンが満たすべき方程式を書くと、
その系の運動方程式がラグランジアンの方程式として得られる。
あとは適当にラグランジアンを用いて運動量とかを定義すればよく知る形になる。

あと、単位についてだけど、これは一番単純な次元解析の方法が通用しない良い例だと思う。
同じような例としては、力のモーメントと仕事は単位は同じになるけど全く別の量、とか。

754 :ご冗談でしょう?名無しさん:2015/02/05(木) 23:31:05.33 ID:???.net
>>753
コイツやばいな
学部1-2年とかならまだ黒歴史で済むが

755 :ご冗談でしょう?名無しさん:2015/02/05(木) 23:40:28.78 ID:???.net
>>749
ラグランジアンL
L=( 運動エネルギー ) − ( ポテンシャルエネルギー )
という量導入し、これを速さで微分したものが運動量
速度で割るんではなく微分する

756 :ご冗談でしょう?名無しさん:2015/02/05(木) 23:54:20.26 ID:???.net
>>748
数密度はスカラー(座標変換で値が変わらない量)で0-formとして考えればいいと思うけど。
u = u_\mu dx^\mu
で1-formとして、nuも1-formとして計算すればいいんじゃない?
リー微分は微分形式の次数を変えないから、リー微分しても1-formのままだから、普通に成分はわかると思う。
見当違いなこと言ってたらごめん。

757 :ご冗談でしょう?名無しさん:2015/02/06(金) 00:07:04.83 ID:???.net
あ、微分形式の意味でのベクトル場との内積に言及してなかった。
iu = u^μ ∂/∂(dx^μ)
で、∂/∂(dx^μ)とdx^μは反交換(グラスマン代数に従う)であること以外は普通の微分と同じで、
∂(dx^σ)/∂(dx^μ) = δ^σ_μ.
ってただの定義を言ってるだけだけど。
Lu(nu) = X_μ dx^μ
として、X_μが求めたい成分では。

758 :ご冗談でしょう?名無しさん:2015/02/06(金) 01:26:33.19 ID:MRl9ESfR.net
>>756
nは3formだと思うのですが。数密度は座標変換して体積が変わると変化しますよね
だから3formの基底をつけてスカラーになるはずです

実際三次元だとLu n = ∇•(un)なので
もし1ならLu n = u•∇nになっちゃいますよね?

759 :ご冗談でしょう?名無しさん:2015/02/06(金) 01:36:45.84 ID:???.net
四元速度は微分形式じゃなくて接ベクトルだよ。

760 :ご冗談でしょう?名無しさん:2015/02/06(金) 01:46:32.25 ID:/cbUP78B.net
>>758
あ、そっか。
例えば電荷密度は空間に電子しかなければ数密度と同値だけど、ローレンツ変換で値変わるもんね。
3-formというか、微小体積をかけた値は不変になるということか。
でもそれだとuは1-formじゃなくて、0-formのベクトルと考えるということになるの?
それともn∧uはtop formなの?

761 :ご冗談でしょう?名無しさん:2015/02/06(金) 01:51:59.41 ID:???.net
接ベクトルなら
n = Ndx∧dy∧du
un = u^μ ∂N/∂x^μ dx∧dy∧du
ってことでいいのか

762 :ご冗談でしょう?名無しさん:2015/02/06(金) 02:38:46.65 ID:???.net
わからなくなった。
そもそも
>>744
>相対論で出てくる粒子数束n u^μ (nは粒子数密度、u^μは四限速度)のu^μに関するリー微分
の粒子数束って何だ。
4元保存カレントではないのか。

763 :ご冗談でしょう?名無しさん:2015/02/06(金) 04:18:25.49 ID:MRl9ESfR.net
粒子数束は連続の式
∂μ(nu^μ)=0を満たすやつです

764 :ご冗談でしょう?名無しさん:2015/02/06(金) 04:36:32.42 ID:/cbUP78B.net
つまり電磁気のカレントとほぼ同じものなんだよね?
なら密度はベクトルの一成分じゃないの?
座標変換で不変なのは、カレントの1-formないしベクトル場、それとホッジ作用素で双対な3-formであって、密度の3-formもそれだけじゃ座標変換で不変ではないと思うんだけど。

765 :ご冗談でしょう?名無しさん:2015/02/06(金) 08:59:03.05 ID:???.net
>>744

http://homepage2.nifty.com/ysc/Dirac21.pdf#search='%E7%B2%92%E5%AD%90%E6%95%B0%E6%9D%9F'

ここ読むと、nは粒子数密度ではなく、固有粒子数密度ですな。
つまり、思いっ切りスカラー。

766 :ご冗談でしょう?名無しさん:2015/02/06(金) 13:52:21.59 ID:???.net
熱力学における圧力についての質問です
圧力があるということは作用反作用の法則によって圧力と同じ力を外部から受けていることになると思います
熱力学はよく「孤立系」という言葉を使って外部と何のやりとりもしていない状態のことを論じますが
圧力というものが存在してる時点ですでに外部からの影響を受けてると思うんですけどこのあたりをどう考えればいいですか?

767 :ご冗談でしょう?名無しさん:2015/02/06(金) 13:58:53.05 ID:???.net
箱に入ってれば箱にかかる圧力の総和は0になるから、箱と一緒に考えれば問題ない。平衡状態なら、箱と気体の温度も等しく、箱について考える必要もなくなる

768 :ご冗談でしょう?名無しさん:2015/02/06(金) 14:04:17.01 ID:???.net
力学の浮力の項を真面目に理解していれば納得出来ると思う

769 :ご冗談でしょう?名無しさん:2015/02/06(金) 14:43:06.96 ID:???.net
物理でdiffeomorphismって言うと、微分同相写像ではなく、一般座標変換と訳していいんでしょうか。

770 :ご冗談でしょう?名無しさん:2015/02/06(金) 15:00:08.66 ID:???.net
>>766
壁がなくても圧力はあるってこった

771 :ご冗談でしょう?名無しさん:2015/02/06(金) 16:47:11.33 ID:nZayrKCKF
>>766
圧力があるだけではエネルギー変化はない

772 :ご冗談でしょう?名無しさん:2015/02/06(金) 23:49:43.34 ID:3TMuTQpR.net
http://wc2014.2ch.net/test/read.cgi/rikei/1324180992/730
  ↑ ↑ ↑ ↑ ↑ ↑ 

773 :ご冗談でしょう?名無しさん:2015/02/07(土) 00:29:36.40 ID:byhPRsEDG
ゴミを貼るな

774 :665:2015/02/07(土) 01:01:50.05 ID:???.net
>>686
普通に折り合いは付く。

775 :ご冗談でしょう?名無しさん:2015/02/07(土) 19:42:06.16 ID:???.net
初歩の質問ですみません
極座標の単位ベクトルについて質問です

OPベクトルの大きさがrのとき、OPベクトルと同じ向きにerを、OPベクトルに垂直でθの増加する向きにeθを
それぞれ単位ベクトルとして取るという風に教科書に書いてあるのですが
このeθがよくわかりません
極座標はどれくらい大きいかをrで、どれくらい傾いてるかをθで表現するものだと思って考えたんですが
eθが、中心Oで半径rの円弧を描くようなベクトルになる気がしてしまうので何か考え方が間違ってるみたいです
どうしてeθをOPベクトルに垂直でθの増加する向きにまっすぐに取るんでしょうか?
どなたかお願いします
数学は高校と微積および線形の最初まで習いました

776 :ご冗談でしょう?名無しさん:2015/02/07(土) 20:00:02.50 ID:???.net
極座標における単位ベクトルは、場所ごとに異なる。
つまり、点Aにおける単位ベクトルと点Bにおける単位ベクトルは一般に平行ではない。
対して、直交座標ではどの点の単位ベクトルも全て平行なので、簡単である。
単位ベクトルが場所ごとに異なるということ、単位ベクトルは位置の関数ということになる。
故に、例えば単位ベクトルの微分も考えなければならない。
直交座標では単位ベクトルは定数だから、そういう操作はあり得ない。微分すると0になる。

全く新しい概念だから面食らうのは普通のことだと思う。

777 :ご冗談でしょう?名無しさん:2015/02/07(土) 20:01:40.51 ID:N4caj1ma.net
コリオリの力は赤道ではゼロという説明があるんですが
コリオリ力は速度だけに依存してどこでも一定ですよね?
だから赤道でもゼロじゃないですよね?

778 :ご冗談でしょう?名無しさん:2015/02/07(土) 20:06:27.24 ID:OsYYBuHz.net
ニュートンの運動方程式が成り立つのは直交座標(デカルト座標)の話であって、
極座標で考えたい時は、直交座標と極座標の対応関係を使って、
直交座標での運動方程式を式変形することで極座標の運動方程式が得られて、
その極座標の運動方程式の見た目は直交座標の運動方程式のそれとは全然異なるものになる。

779 :ご冗談でしょう?名無しさん:2015/02/07(土) 20:16:11.53 ID:???.net
>>775
その意味での「単位ベクトル」ってのは、
座標変数(xとかyとかrとかθとか)がちょっとだけ増えたときに
その座標で表される点がどっちに動くか?
を表すための大きさ1のベクトルなんだよ。
x座標が増えたら点はex方向に動くだろ?

780 :ご冗談でしょう?名無しさん:2015/02/07(土) 20:20:10.33 ID:???.net
>>777
地球上のコリオリ力は緯度と速度に依存して赤道ではゼロです。
北半球と南半球で台風の渦は逆なんだから
赤道ではゼロに決まってるでしょ。

781 :ご冗談でしょう?名無しさん:2015/02/07(土) 20:35:58.80 ID:???.net
コリオリの力がないわけではなくて、コリオリの力の地表に平行な成分がないんじゃないの。

782 :ご冗談でしょう?名無しさん:2015/02/07(土) 21:48:35.06 ID:???.net
コリオリなら俺のベッドで寝てるよ

783 :ご冗談でしょう?名無しさん:2015/02/07(土) 22:09:18.57 ID:???.net
>>776
>>778
>>779
レスありがとうございます
なんとなくわかってきました
でももう一つ聞きたいです
θが増加する向きなら垂直じゃなくても鋭角でも鈍角でもいいと思うんですが
垂直だと都合のいいことがあるんですか?

784 :ご冗談でしょう?名無しさん:2015/02/07(土) 22:17:38.03 ID:???.net
高校レベルの物理なのだが
「ある物体を壁に押し付けて10Kgの力をかける」のと「ある物体の両側から10kgずつの力をかける」のは物体にかかる力はどちらも10kgとかいうのが全然納得できん。
誰か上手に説明して俺を納得させてくれ。

785 :ご冗談でしょう?名無しさん:2015/02/07(土) 22:39:43.63 ID:???.net
異なる単位ベクトルの内積が0だと便利だから
xyzが直交してるのと同じ
xyzが直交していることに疑問を持たないなら極座標でも持たないだろ

786 :ご冗談でしょう?名無しさん:2015/02/07(土) 22:40:17.19 ID:???.net
>>784
「物体にかかる力」の定義は?

787 :ご冗談でしょう?名無しさん:2015/02/07(土) 22:44:44.60 ID:???.net
>>783
すまん、その変数「だけ」を微小変化させたときの点の変化方向なんだよ。
極座標の場合erとeθは直交する。
勿論座標系の構成方法によっては直交しないこともありうる。
しかし直交した方が色々と便利なことが多いので、
直交するように座標系を構成することが多い。

788 :ご冗談でしょう?名無しさん:2015/02/07(土) 22:52:44.69 ID:???.net
>>784
納得できんポイントがわからん。
結果は同じだろ?
それとも力のかかる面積を気にしてるのか?
それなら違うのは圧力であって力は同じだ。

789 :ご冗談でしょう?名無しさん:2015/02/08(日) 00:20:00.63 ID:???.net
>>785
>>787
レスありがとうございます
三角関数に関係があるんですね
θを微小変化させたときの点の動く向きに大きさ1の単位ベクトルを取るのは
どうしても微小変化させた点をP'としてOP'-OPの微分を積分したら円弧になる気がしてよくわかりません
でももうちょっと考えてみます
ありがとうございました!

790 :ご冗談でしょう?名無しさん:2015/02/08(日) 00:38:40.85 ID:???.net
>>789
円弧になるよ。
地球で生活してて、球面上で生きてると実感できないように、
半径に対し、円弧の長さが十分短ければ、それはほぼ直線と考えられる。
そして無限小の極限ではその方向は円の接線方向に等しい。

791 :ご冗談でしょう?名無しさん:2015/02/08(日) 01:08:55.65 ID:g4yriiG4O
>>781
あったまいいー

792 :ご冗談でしょう?名無しさん:2015/02/08(日) 02:02:03.07 ID:Q2Wj3BFE.net
学部2年
工学部です。
物理が最近好きになりました。
長文かつ質問が多々ありますこと、拙い質問であることお許しください。
1.エネルギーとはなにか?運動量とは何か。
ぼんやりとしかわかっておりません。とくに運動量とは実際どのようなものなのでしょう?それともmvで定義された以外の何者でもないのでしょうか。また、力積について詳しく教えてください。
2.保存場とはなにか?
保存場とは経路によらず仕事が一定であるということや、一周したら仕事がゼロということを他人にうまく説明できません。誰かお教えください。
3.円運動における遠心力がよくわからない。
円運動の遠心力とはどのようなものなのでしょうか?
4.臨海振動などは実学的には何に使われているのか?

まずはこの4つをお願い申し上げます。

793 :ご冗談でしょう?名無しさん:2015/02/08(日) 02:14:56.87 ID:???.net
123は高校物理で習うことじゃないか

794 :ご冗談でしょう?名無しさん:2015/02/08(日) 02:33:37.49 ID:???.net
>>792
> 〜は実学的には何に使われているのか?
このへんの言い回しは工学屋って感じ。
理学系の、特に理論屋の前で言ったら小一時間は説教されると思うから、あんまり「実学」とか言わないほうがいいよ。

795 :ご冗談でしょう?名無しさん:2015/02/08(日) 02:49:34.35 ID:Q2Wj3BFE.net
>>793
1番 エネルギーってそもそも定義ってなんなのかわからなくなってきました。運動量もなぜそんなものが生まれたのか、何のためなのか

2番目は偏微分を使って説明してたことはうっすら覚えているのですが、記憶から消し去られてしまいました。ご教授願います

3番目は、絶対座標系?実験系?よくわからないのですがその場合は向心力があれば、接線方向の力と合成されてかいてんされているのはわかります。もし、質点の慣性系?に固定した時、なぜ遠心力なるものが生まれるか不明です。

4番目は、どのような技術に応用されているか知りたかったです。あと、エネルギーと臨界制動の関係も知りたいです。

796 :ご冗談でしょう?名無しさん:2015/02/08(日) 02:53:55.01 ID:???.net
>>786
力の定義といわれてもイマイチピンとこないけど、手で押したとかそんなん。

>>788
物体に10Kgかかるのと20kgかかるの違いがでるんじゃ実用上問題あるじゃろーよ。
俺は両側から10Kgで押したら20Kgの力がかかると思ってるんだ。

797 :ご冗談でしょう?名無しさん:2015/02/08(日) 03:06:08.86 ID:a1QqdtjH.net
物理は高校1年の時にかじった程度です。
今、『米海軍が発表、最新兵器「レールガン」の全貌〜砲弾を200キロ先まで飛ばせる 』なんて記事を見てました。
そこには戦艦大和の事も引き合いに出されていたのですが、ここでちょっとした疑問です。
@アメリカは重さ15kgの砲弾を2.5q/sで打ち出すようですが、仮に10q先の地面(密度5g/p3)に命中したとしてどれぐらいの被害が出るか。
A大和の九一式徹甲弾と同じサイズの普通の弾(全長1980o、直径458.5o、重さ1460s)を2.5q/sで打ち出したとして、仮に10q先の地面(密度5g/p3)に命中したとしてどれぐらいの被害が出るか。
それを算出できる式も含めて、お願いいたします。

(補足:大和は三式弾、九一式徹甲弾を初速780m/sで撃ちだしていたそうです。)

798 :ご冗談でしょう?名無しさん:2015/02/08(日) 03:18:20.58 ID:g4yriiG4O
技術に応用なんて工学屋に聞く事だろ
その前に高校の教科書を読むんだな
基礎知識前提の疑問なら答えるが、最初からなんて無駄な事を要求するな

799 :ご冗談でしょう?名無しさん:2015/02/08(日) 03:21:05.76 ID:???.net
>>795
運動量は物体の力学(時間が進むとどのように運動するか)を知るために生まれた概念かな。
運動方程式ma=Fの左辺は運動量の時間微分だしね。
エネルギーは位置エネルギーとか化学エネルギーとか物体が動いてなくてもためることができる量で動いてる場合も運動エネルギーとして存在する。
位置エネルギーを定義することができる力、としか数式を使わないなら言えない。
数式的に言えば、ストークスの定理から、divergence(発散)が0になる力が保存力となる。
運動方程式は慣性系でしか成り立たず、回転座標系などの非慣性系では慣性力を考えないと成り立たない。回転座標系での慣性力は特別に、遠心力とコリオリ力という名前が付いている。

800 :ご冗談でしょう?名無しさん:2015/02/08(日) 04:44:48.55 ID:???.net
宇宙一進んでいる文明は、宇宙の謎をどれくらい解き明かしているのでしょうか?

801 :ご冗談でしょう?名無しさん:2015/02/08(日) 06:19:40.22 ID:???.net
>>796
両方の条件で(お前の定義によると)20kgの力がかかるんだが?
違いは押してるのが壁なのか壁じゃないのかだけだ。

802 :ご冗談でしょう?名無しさん:2015/02/08(日) 06:22:56.51 ID:???.net
>>795
エネルギーと運動量は間並進及び空間並進対称性に対応するネーターチャージだよ

803 :ご冗談でしょう?名無しさん:2015/02/08(日) 06:25:31.41 ID:???.net
>>797
物理板で聞くべき質問じゃないから軍事板で聞きな。
「被害」を物理学的に定義できるなら別だけどね。

804 :ご冗談でしょう?名無しさん:2015/02/08(日) 06:33:34.66 ID:???.net
>>796
壁を押したら押し返される、というのが理解できてないだけでは?

805 :ご冗談でしょう?名無しさん:2015/02/08(日) 09:42:14.07 ID:???.net
高校レベルの物理を微分積分をつかって説明してくれる本で何かいいのってありますか?

806 :ご冗談でしょう?名無しさん:2015/02/08(日) 10:11:53.09 ID:???.net
>>801
あるぇー?
教科書とかだとどっちも10Kgしかかからんのじゃねぇの?


>>804
作用反作用はわかんだよ。
釣り合いの話じゃねーんだよぅ。
物体がつぶれるかつぶれないかで考えた時物体にかかるトータルの力はいくつだってんだって話だ。

807 :ご冗談でしょう?名無しさん:2015/02/08(日) 10:25:51.91 ID:???.net
トータルの力ってなんだよ…
力積でも復習してろ

808 :ご冗談でしょう?名無しさん:2015/02/08(日) 11:30:32.61 ID:???.net
>>807
所詮ここにいる連中ではまともな答えは出んという代表例か、お前は。

809 :ご冗談でしょう?名無しさん:2015/02/08(日) 11:31:09.20 ID:???.net
トータルの力って言うなら、前後からそれぞれ10キロで押されていて左右からはそれぞれ10キロで引っ張られていたらトータルはゼロでいいの?

810 :ご冗談でしょう?名無しさん:2015/02/08(日) 12:33:15.13 ID:???.net
>>808
まともな質問でなきゃマトモな答は出んわな

811 :ご冗談でしょう?名無しさん:2015/02/08(日) 14:47:02.62 ID:???.net
>>808
よくいる逆切れする「俺様」の代表だな、お前は。

812 :ご冗談でしょう?名無しさん:2015/02/08(日) 18:46:27.24 ID:???.net
ma=F
って、力の定義式?質量の定義式?加速度の定義式?
http://ja.wikipedia.org/wiki/%E3%82%AA%E3%83%83%E3%82%AB%E3%83%A0%E3%81%AE%E5%89%83%E5%88%80
によると、「力」という「わけのわからないもの」を物理に導入すること反対する(至極まっとうな)考えかたもあるみたいだけど。

813 :ご冗談でしょう?名無しさん:2015/02/08(日) 19:15:19.55 ID:???.net
>>812
p = mv という関係が成り立つ限りにおいて、力の定義式。

814 :ご冗談でしょう?名無しさん:2015/02/08(日) 19:22:06.15 ID:???.net
>>813
その定義が必要無い、というのがオッカムの剃刀の考え方なんだけど
「力」という補助概念をいつまでも内包し続ける意味が、物理の進展にとってどういう利点があるのかを
そろそろ明らかにしても良い時期なのかな、と思う。

815 :ご冗談でしょう?名無しさん:2015/02/08(日) 19:27:06.94 ID:???.net
実際使っててめちゃ便利じゃん

816 :ご冗談でしょう?名無しさん:2015/02/08(日) 19:38:12.10 ID:???.net
「力」が不要なら「速度」だって不要だよな?

817 :ご冗談でしょう?名無しさん:2015/02/08(日) 19:39:34.88 ID:???.net
力=運動量の時間変化率
これが必要無いと言えるのが分からんわ

818 :ご冗談でしょう?名無しさん:2015/02/08(日) 19:49:42.15 ID:???.net
>>816
力は不要だけど、速度は必要。
なぜなら、速度は長さと時間という基本的な単位で厳密に定義できるから。

>>817
「力」を導入せずとも、「運動量の時間変化率」を使えば事足りる。

819 :ご冗談でしょう?名無しさん:2015/02/08(日) 19:53:00.58 ID:???.net
>>808
だったら、頭のいいあんたが回答サクッと書いて終わらせればいいだろ
出来もしないくせにいちゃもんだけは一丁前

820 :ご冗談でしょう?名無しさん:2015/02/08(日) 19:53:28.93 ID:???.net
強度計算どうすんだよ

821 :ご冗談でしょう?名無しさん:2015/02/08(日) 19:56:03.25 ID:???.net
>>810
>>811
この程度の煽りで切れるたぁ、器が小さいよ?

後、大学レベルや哲学的な話にせんでくれw
高校レベルだっつってんだろw

822 :ご冗談でしょう?名無しさん:2015/02/08(日) 19:59:28.24 ID:???.net
>>819
悪いな、俺は学がないんだ。
この程度の事を聞いてる時点でそれは察してもらおうか。

823 :ご冗談でしょう?名無しさん:2015/02/08(日) 20:00:35.27 ID:???.net
「速度」を導入せずとも、「変位の時間変化率」を使えば事足りる。

824 :ご冗談でしょう?名無しさん:2015/02/08(日) 20:08:17.12 ID:???.net
>>812
何かの定義式ではなく、異なった方法で定義された3つの物理量に関する物理法則である、
と間違いだらけの物理概念 (パリティブックス) かその続編に書いてあった。

825 :ご冗談でしょう?名無しさん:2015/02/08(日) 20:09:51.96 ID:???.net
運動方程式は力の定義式ではなく、力と運動量の関係式だと思う。
ハミルトニアンとエネルギーが異なるように、運動量の時間微分と力も異なる。
じゃあ力はどう定義されてるかというと、やはりニュートン力学では曖昧なままだと思う。

826 :ご冗談でしょう?名無しさん:2015/02/08(日) 20:14:14.59 ID:???.net
ニュートン力学において力はdp/dtで定義される曖昧さ0の物理量である

827 :ご冗談でしょう?名無しさん:2015/02/08(日) 20:15:26.75 ID:???.net
なにが言いたいかさっぱりわからん

828 :ご冗談でしょう?名無しさん:2015/02/08(日) 20:19:42.65 ID:???.net
力は一般に位置と時間の関数だろ。
運動量の時間微分は時間だけの関数じゃないのか。

829 :ご冗談でしょう?名無しさん:2015/02/08(日) 20:24:03.26 ID:???.net
位置と時間を独立に考える限りpは位置と時間の関数でその時間微分もまた位置と時間の関数

830 :ご冗談でしょう?名無しさん:2015/02/08(日) 20:25:34.93 ID:???.net
>>807の書き込みをバカにしてるからいつまでたっても正解にたどり着けない

831 :ご冗談でしょう?名無しさん:2015/02/08(日) 20:27:37.74 ID:???.net
d^2x/dt^2 = (1/m)F(x,dx/dt,t)
を満たす、位置と速度と時間の関数が存在するというのが運動方程式であって、
これが仮に定義式ならば、力は時間の関数ということになると思う。

832 :ご冗談でしょう?名無しさん:2015/02/08(日) 20:28:44.21 ID:???.net
>>831
何が定義だと思う?

833 :ご冗談でしょう?名無しさん:2015/02/08(日) 20:29:23.99 ID:???.net
d^2x(t)/dt^2 = (1/m)F(x,dx/dt,t)
こう書いた方がわかりやすいか

834 :ご冗談でしょう?名無しさん:2015/02/08(日) 20:34:33.29 ID:???.net
md^2x/dt^2=F
xがtだけの関数で表せるのなら当然力は時間の関数
xがtだけで表せないのなら左辺はtだけの関数でないのだからそれで定義されるFもtだけの関数ではない

835 :ご冗談でしょう?名無しさん:2015/02/08(日) 20:45:10.35 ID:???.net
>>806
お前の定義によると、とちゃんと書いただろうが。
お前が「物体がつぶれるかつぶれないかで考えたときの力」と呼んでるのは、
専門用語では圧縮応力と呼ばれるものに近い。
ただ、圧縮応力はかかる面積で力を割るという違いはあるが。

そして、圧縮応力は片側のみの値を使うことになっている。
何故なら、釣り合い条件が成り立っているとは限らない場合、
右からの圧縮応力と左からの圧縮応力とを別々に考える必要があるからだ。
それに、こう定義しておけば片側だけを計測すればすむので計測上も便利だ。
勿論お前のように左右の合計を使っても問題はないが、
釣り合い条件にないときには使えなくなってしまうし色々と不便なので、
片側にしましょうねってことに昔の偉い人が決めたんだよ。

勿論それに従う必要はないが、その時は他の人に自分がどういう定義でその言葉を使っているか
ちゃんと説明しておかなければ今日のこのスレの流れみたいにグダグダになる。
建設的な議論を楽にしたいのなら長いものには巻かれておいた方がいい。
別にそうしろと命令する訳じゃないがな。

836 :ご冗談でしょう?名無しさん:2015/02/08(日) 20:56:10.92 ID:???.net
>>834
xは常にtの関数でしょ
まあ、xとdx/dtもtの関数だから、Fも常にtの関数であるとも言えるんだけど。

ある微分方程式があったとして、右辺は左辺の単なる定義とは考えないでしょ?

837 :ご冗談でしょう?名無しさん:2015/02/08(日) 20:59:58.53 ID:???.net
バネの問題を考える時、力の定義は
F = -kx
であって、運動方程式は
md^2x/dt^2 = -kx = F
は関係式じゃないか。

838 :ご冗談でしょう?名無しさん:2015/02/08(日) 21:26:20.85 ID:???.net
>>836
xはtによって決まるべきだが屡々tを陰にする
その場合Fはx等の関数になるだけでFはt,x,vの関数云々は的外れ

dx/dt=v
があったとして右辺は左辺の単なる定義と考える人は多くいるだろう

839 :ご冗談でしょう?名無しさん:2015/02/08(日) 21:30:14.86 ID:???.net
>>837
バネの力-kxは定義式ではない
dp/dt=FというFの定義式から解析した結果バネの力Fは-kxと表される事がわかった
(現実問題としては定義式から導かれる観測により適した定理によってだろうが)
F=-kxといのは関係式である

840 :ご冗談でしょう?名無しさん:2015/02/08(日) 21:37:22.96 ID:???.net
>>835
おおう。圧縮応力と力の釣り合いをごっちゃにするなってことなのか。
なるほどなるほど。高校レベルに留まってた話ではなかったのだな。

定義か、ややこしいものだな。学がないんで分からなかったのは許せ。
お前怒ってる割りにいい仕事するじゃないか。
GJと言っておこう。

841 :ご冗談でしょう?名無しさん:2015/02/08(日) 21:52:26.92 ID:???.net
>>839
仮にただの定義式なら、運動方程式は何の意味もない式じゃないか。

842 :ご冗談でしょう?名無しさん:2015/02/08(日) 22:16:43.88 ID:???.net
運動方程式が力の定義式になるのは順問題のときだけ

843 :ご冗談でしょう?名無しさん:2015/02/08(日) 22:24:18.93 ID:???.net
>>841
それはdp/dt=Fで定義されたFの性質による
全ての場合においてdp/dtによって直接Fを求めるしか出来ないのなら意義は薄い
dp/dtを陽に使わずFを求められるのならそれから逆に運動を解析することが出来る
また陽に使ったとしてもそこから得られたFの規則から一般化出来るのなら未知の運動の解析につながる

844 :ご冗談でしょう?名無しさん:2015/02/08(日) 22:28:06.37 ID:???.net
>>843
運動方程式はオイラーラグランジュ方程式と同値な訳で、オイラーラグランジュ方程式は定義式ではないだろ

845 :ご冗談でしょう?名無しさん:2015/02/08(日) 22:33:15.62 ID:???.net
例えば、熱力学のエントロピーの定義に合うように、統計力学ではエントロピーは定義されるのと一緒で、
運動方程式を満たすように力は定義される(バネならF=-kx)が、運動方程式が定義な訳ではないと思う

846 :ご冗談でしょう?名無しさん:2015/02/08(日) 22:37:36.35 ID:???.net
>>844
ハミルトンの原理を認めた上でそれは導出される(この段階でLは定義されている)
Newton力学ではハミルトンの原理を仮定していない
それ故ハミルトンの原理から導出されるのだから数学的に等価な運動方程式は定義式ではないと言うのはおかしい
表面だけをなぞっているとこのような誤りを犯す

847 :ご冗談でしょう?名無しさん:2015/02/08(日) 22:41:00.24 ID:???.net
ハミルトンの原理って、運動方程式と同等の、単なる仮定にすぎないと思うんだけど
仮定以上の何かがハミルトンの原理にあるという実験的な確証って得られているんだっけ?

848 :ご冗談でしょう?名無しさん:2015/02/08(日) 22:48:36.96 ID:???.net
>>845
力の具体的表式は力という物理量の定義には必要ない
運動方程式で力が定義され、それの性質を調べることによってより一般の運動を解析する
力の具体的表式はdp/dt=Fで力が定義されていないと意味を成さない

849 :ご冗談でしょう?名無しさん:2015/02/08(日) 23:00:45.39 ID:???.net
Lを持ち出すことで物理現象の統一的表現に長けている
Lを上手く選んでやる必要はあるがそれは別の話
Newton力学の力同様Lの具体的表式というものは基本原理には必要ない
Lという概念があればよい

850 :ご冗談でしょう?名無しさん:2015/02/08(日) 23:15:42.16 ID:???.net
>>846
ハミルトンの原理と運動方程式は等価でしょ?
ハミルトンの原理が成り立てば運動方程式が成り立ち、運動方程式が成り立てばハミルトンの原理が成り立つんだから。

851 :ご冗談でしょう?名無しさん:2015/02/08(日) 23:23:01.38 ID:???.net
ハミルトン言いたいだけやろお前

852 :ご冗談でしょう?名無しさん:2015/02/08(日) 23:40:28.82 ID:???.net
まさにばかばっか

853 :ご冗談でしょう?名無しさん:2015/02/09(月) 00:17:51.14 ID:???.net
>>850
基本原理であるハミルトンの原理が運動方程式と等価となるためにはLをある形で定義する必要がある
故にFが運動方程式で定義されても問題ない
寧ろ対応を考えれば定義されるべきだ

854 :ご冗談でしょう?名無しさん:2015/02/09(月) 00:39:17.93 ID:???.net
>>853
ハミルトンの原理は仮定でしょ。
その仮定のおかげで、物体の運動を記述することができる。
ただ定義が書かれただけの式は、何もこの世界の法則に言及しないでしょ。
運動方程式が仮に定義式だとして、運動方程式にそれ以上の意味はあるの?

855 :ご冗談でしょう?名無しさん:2015/02/09(月) 01:26:30.17 ID:???.net
定義なくして認識はない
(ニーチェ)

856 :ご冗談でしょう?名無しさん:2015/02/09(月) 01:32:34.26 ID:???.net
>>854
第三法則でdp/dt=Fで定義されたFに関する基本原理が登場する
それを用いることでFを調べることが出来、そして解析する
基本原理を独立でしか見ることが出来ないと意味を見出せないのかもしれない
Lの定義がハミルトンの原理を通してその体系を基礎づけるがLだけ見る馬鹿は話にならない

857 :ご冗談でしょう?名無しさん:2015/02/09(月) 01:38:30.79 ID:???.net
堂々巡り

858 :ご冗談でしょう?名無しさん:2015/02/09(月) 01:48:52.81 ID:???.net
力は静力学からだけでも、定数分の不定性を除いて定義できる。この力はあの力の二倍、この力はあの力の三倍だって具合に。
それが運動量の時間微分と比例関係にあるということを仮定するのが運動方程式と考えるのが、俺は一番納得できる。
つまり、運動方程式は質量の定義式ではあるが、さらに力と運動量の時間微分が比例するという仮定を同時にしているということ。

859 :ご冗談でしょう?名無しさん:2015/02/09(月) 01:59:22.67 ID:???.net
あ、力の係数分の不定性を定めるという意味でなら、運動方程式は力の定義式というのは理解できる。

860 :ご冗談でしょう?名無しさん:2015/02/09(月) 02:42:44.39 ID:???.net
http://youtu.be/I-VRGyYx8Ok

861 :ご冗談でしょう?名無しさん:2015/02/09(月) 02:55:41.30 ID:???.net
>>814
「力」なる概念が持つ哲学的意味は明らかじゃないか?
運動量の変化に「原因」をもたらし、因果律を定義するための道具だろ?

因果律まで懐疑している向きの人間なら物理などに興味はなかろうし。

862 :ご冗談でしょう?名無しさん:2015/02/09(月) 09:02:05.16 ID:r+CQTWQ8.net
導波管の金属表面に平行な電場ベクトルと表面抵抗と電磁波の群速度がわかれば導体損失は計算できますか?

863 :ご冗談でしょう?名無しさん:2015/02/09(月) 12:46:06.50 ID:eal03DnCv
ハミルトンの原理は「ハミルトンの原理の形にする事が出来る」だけの意味
実験など関係なく、数式上で確認できるだけの事
ゲージ原理などと同じく認識形式なのさ

864 :ご冗談でしょう?名無しさん:2015/02/09(月) 13:54:14.81 ID:???.net
物理で一番難しい分野は何ですか?

865 :ご冗談でしょう?名無しさん:2015/02/09(月) 13:56:52.56 ID:???.net
人間の思考を物理方程式で表現できますか?

866 :ご冗談でしょう?名無しさん:2015/02/10(火) 08:00:32.70 ID:syKAIkCoJ
http://nlab.itmedia.co.jp/nl/articles/1409/20/news017.html
http://jp.reuters.com/article/marketsNews/idJPL3N0MG13T20140319、プログラミング、http://chinareaction.com/blog-entry-890.htmlで、
仮想空間移動ができる。ソードアートオンラインのような電脳世界ができ、実際のセックスもできる。登記

867 :ご冗談でしょう?名無しさん:2015/02/09(月) 14:54:19.28 ID:???.net
できます。
周りに流されやすい人は思考質量が小さく、逆に思考質量が大きすぎる人は頑固者と呼ばれたりします。

868 :ご冗談でしょう?名無しさん:2015/02/09(月) 17:07:14.26 ID:???.net
>>862
スレ違いです

869 :ご冗談でしょう?名無しさん:2015/02/09(月) 17:20:49.67 ID:???.net
重力が存在しないとしたら、引力の原因は何ですか?

870 :ご冗談でしょう?名無しさん:2015/02/09(月) 17:37:11.55 ID:???.net
>>869
質量によって引き起こされる空間の歪み

871 :ご冗談でしょう?名無しさん:2015/02/09(月) 18:33:47.46 ID:???.net
重力って空間の歪みだよね。
重力が存在しないってことは、現代物理学にたいする挑戦か?

872 :ご冗談でしょう?名無しさん:2015/02/09(月) 20:01:43.22 ID:HX8K3nlx.net
空間じゃなくて時空の歪み

873 :ご冗談でしょう?名無しさん:2015/02/09(月) 23:46:25.52 ID:???.net
時間が歪むわけねえだろw
このスカタンが

874 :ご冗談でしょう?名無しさん:2015/02/09(月) 23:49:37.16 ID:???.net
空間が歪むわけねえだろw
このスカタンが

875 :ご冗談でしょう?名無しさん:2015/02/10(火) 00:04:32.67 ID:???.net
実体は重力だよな。
其れを感覚的に説明するために空間が歪むとかいってるけど、歪んだら物体が引き寄せられるロジックが不明なので説明としてもイマイチ。
空間が歪む
→空間を平面表示したものも歪む
→平面表示された空間が「下に」凹むように歪む
→下に凹んでるから物体が落ちる
→其れを上から見ると物体が引き寄せられるように見える

とかいう子供騙しな説明。
でも、なんで下に凹んでると物体が落ちるんですかね。
→下に凹んでると物体が落ちるのは、重力のせい。

あれれ?
重力の説明するのに、また違う重力出てくるの?
馬鹿なの?
死ぬの?

876 :ご冗談でしょう?名無しさん:2015/02/10(火) 00:05:28.80 ID:???.net
だから空間じゃなくて歪むのは時空だと言ってるだろ

877 :ご冗談でしょう?名無しさん:2015/02/10(火) 00:12:20.53 ID:???.net
子供じみた質問で申し訳ないのですが、半径1mmで温度が1億度の球体がそこら辺のオフィスに1秒間出現した場合、周囲の人間にどの程度影響があるのでしょうか?
半径1メートル以内の人間が蒸発するくらいの影響があるのか・・1秒くらいであればどうってことないのか・・・

878 :ご冗談でしょう?名無しさん:2015/02/10(火) 00:17:26.67 ID:???.net
質量が大きいと空間が歪むのは何故?

質量大きい
→とっても重い
→空間を平面表示すると重いから凹む

なんで凹むのw

重いと重力で下にビヨーンと平面が引っ張られるから

その重力って何?
さっきの重力と違うよね?

879 :ご冗談でしょう?名無しさん:2015/02/10(火) 00:22:35.75 ID:???.net
>>877
温度だけではなんとも言えないな

880 :ご冗談でしょう?名無しさん:2015/02/10(火) 00:24:50.20 ID:???.net
http://livedoor.blogimg.jp/worldfusigi/imgs/d/8/d8be1fb2.jpg

881 :ご冗談でしょう?名無しさん:2015/02/10(火) 01:05:18.45 ID:???.net
質量で空間が歪むことは、実験で確かめたのですか?

882 :ご冗談でしょう?名無しさん:2015/02/10(火) 01:12:09.90 ID:???.net
>>881
確かめられてるよ。
鉛の塊のそばでは光が遅くなることがちゃんと測定されている。
これを使って万有引力定数を精度よく決定するということも行われている。

883 :ご冗談でしょう?名無しさん:2015/02/10(火) 01:14:03.07 ID:???.net
>>881
おっと、見間違えた。
歪むのは時空だから光が遅くなることが証明になるのね。

884 :ご冗談でしょう?名無しさん:2015/02/10(火) 01:15:01.97 ID:???.net
それは空間が歪んだことの証拠にはならない。
単に鉛の塊のそばでは光が遅くなることが立証されただけだ。

885 :ご冗談でしょう?名無しさん:2015/02/10(火) 01:22:40.57 ID:???.net
重力波の測定ってどのくらい進んでるんだろ

886 :ご冗談でしょう?名無しさん:2015/02/10(火) 01:26:23.21 ID:???.net
波であることの証拠もないのに測定とな

887 :ご冗談でしょう?名無しさん:2015/02/10(火) 01:42:02.36 ID:???.net
重力理論が間違ってるとしても、その線形近似理論は多分あってるだろうし、
重力波って線形近似の現象だから、あると思うけどね

888 :ご冗談でしょう?名無しさん:2015/02/10(火) 01:59:35.93 ID:???.net
宇宙は真空なんですか?
それとも、プラズマで満たされているんですか?

889 :ご冗談でしょう?名無しさん:2015/02/10(火) 02:22:35.55 ID:???.net
ビッグバンから人間が出現するまで、偶然の連続だったのですか?
それとも、誰かが設計したのですか?

890 :ご冗談でしょう?名無しさん:2015/02/10(火) 05:29:16.32 ID:???.net
そもそも宇宙とは何なんですか?

891 :ご冗談でしょう?名無しさん:2015/02/10(火) 10:19:33.60 ID:???.net
>>877
黒体輻射と仮定してオーダーで計算します。
ステファン・ボルツマン定数はだいたい510^(-7)[Wm^(-2)K^(-4)]なので
10^(-7)*(10^8)^4*10^(-8) = 10^17[W]。

距離1mならば、人間の受ける面積を1[m^2]とすると、1秒間なので10^16[J]の熱輻射を受けることになります。
人間の体重を10^5[g]とすると、1gあたり10^11[J]で10^10[cal]となる。

これはとんでもない値ですね。核融合の最低限の条件はこの温度ですが。

なお、1億度の黒体輻射の光子のエネルギーは10^8/10^4=10^4[eV]=10[keV]でX線の領域になります。

892 :ご冗談でしょう?名無しさん:2015/02/10(火) 10:38:04.63 ID:???.net
数値の後の単位に括弧は要らん

893 :ご冗談でしょう?名無しさん:2015/02/10(火) 10:51:17.82 ID:???.net
どうでもいい

894 :ご冗談でしょう?名無しさん:2015/02/10(火) 12:41:43.11 ID:???.net
これまでにウルトラマンを苦戦させた怪獣は多数いるもののウルトラマンを完膚なきまでに敗北させた(”死”に追いやった)のはゼットンが初である。
武器は顔の発光器官から放つ「一兆度の火球」
スペシウム光線を当てられるも、吸収し波状光線として撃ち返してカラータイマーを破壊し、ウルトラマンを倒す。

895 :ご冗談でしょう?名無しさん:2015/02/10(火) 14:23:02.49 ID:B39QseFs/
>>884
それを時空の歪みと言うんだよ

896 :ご冗談でしょう?名無しさん:2015/02/10(火) 16:14:51.31 ID:???.net
物理学の研究はロボットに取ってかわられることはないですか?
たとえばもしワープ技術が確立されれば物流業界は壊滅するわけですけど
職業としての物理の研究は科学技術の発達によって淘汰されることはないと思われますか?

897 :ご冗談でしょう?名無しさん:2015/02/10(火) 16:43:19.52 ID:???.net
重力波が存在すると仮定し、連星パルサーの軌道周期の減少の仕方を調べたところ、理論値と観測値が誤差1%以内で一致した。
理論と観測は独立な事象にも関わらずこれほどの一致がみられたのだから、重力波は存在すると言ってもいいだろう。

898 :ご冗談でしょう?名無しさん:2015/02/10(火) 17:00:03.11 ID:???.net
あらゆる学問を包括している学問は何ですか?
脳科学あたりですか?

899 :ご冗談でしょう?名無しさん:2015/02/10(火) 17:18:24.56 ID:???.net
重力は存在しない、が真実のようだ。
いつ、公開するんだろう。

900 :ご冗談でしょう?名無しさん:2015/02/10(火) 19:11:00.36 ID:???.net
馬鹿は去れ

901 :ご冗談でしょう?名無しさん:2015/02/10(火) 19:22:17.98 ID:GpaC+ee6.net
重力は光速で伝播するのだから
重力波も光速で伝播する

902 :NAS6 ◆n3AmnVhjwc :2015/02/10(火) 19:33:04.99 ID:8hIGfW4c.net
重力波が光速度ならば
古典天文学の光行差で
なんで計算が合うの?

903 :ご冗談でしょう?名無しさん:2015/02/10(火) 19:34:07.11 ID:???.net
重力波にも郡速度とか位相速度とかあるの?

904 : ◆xmy3qIqh96 :2015/02/10(火) 21:58:46.88 ID:???.net
2ちゃんねる様。突然の質問申し訳ありません。

児童向けの飛行機の簡単な実験&工作のイベントの準備をしているのですが、分から
ないことがあり、どこに質問すればいいのかも分かりません。ぜひここで相談させて
ください。

やろうとしている実験内容はだいたいこんな感じです。

簡単な飛行機の工作をして、それの頭にゼムクリップを5〜6個付けて、「重心を前
の方にすると飛ぶようになるね」という感じの事を行いたいと考えています。

やろうとしている実験内容はだいたいこんな感じです。

簡単な飛行機の工作をして、それの頭にゼムクリップを5〜6個付けて、「重心を前
の方にすると飛ぶようになるね」という感じの事を行いたいと考えています。

その説明の際に、話の流れとして

【1】物のバランスが取れる所=重心の説明

【2】物の端っこに「おもり」を付けたら重心はそっちの方にずれるという話

【3】飛行機の翼の浮力の話

【4】風圧中心の話(翼の前から四分の一)

【5】工作した飛行機の風圧中心と重心の位置を合わせよう

ここで、【4】の時に以下のような説明をするつもりです。

905 : ◆xmy3qIqh96 :2015/02/10(火) 22:02:51.69 ID:???.net
厚紙に糸を中心からずれた箇所にセロテープで貼り付け、
(端から四分の一の位置)

     ┃
┌──╂─────┐
│    ┃        │
│    ┃        │
│    ┃         │
│    ┃        │
└──╂─────┘
      ┃

この向きで糸の上と下をギュッと引っ張って、
そのまんまひたすら走り回る。
すると、厚紙は走る方向から斜めを向いた方向を向く。
で、「だいたいこのあたりに風圧中心があるんだよ」と説明。
----------------------------------------------

上記の実験は家でも簡単に行える非常に簡単なものであり、
ぜひ今回の実験に取り入れたいと考えています。

揚力関連の話は難しいので厳密は話をするつもりは無いのですが、
さすがに児童相手であってもデタラメの話をするのは
まずいと思い、類似の実験(簡易に風圧中心を実感できるような実験)
を探しても見つかりません。
そのような状況なので「この実験と説明に何か根本的な間違いがあるのでは?」と
心配になっています。

ただ、他の人に相談しようにも「風圧中心」という専門用語の
話になるので、一体どなたに相談すればいいのかという状態です。

上記の実験&説明はだいたい合っているのでしょうか。よろしくお願いいたします。

906 :ご冗談でしょう?名無しさん:2015/02/10(火) 22:07:01.20 ID:???.net
便所の落書きでこの手の相談をするのもどうなのかと思わないでもないが・・・・w



むしろこの板だとお子様の手の届かない位置に置いてしまいそうだ。

907 :ご冗談でしょう?名無しさん:2015/02/10(火) 22:15:53.42 ID:???.net
>>891
ご回答ありがとうございました。やばいんですね。

908 :ご冗談でしょう?名無しさん:2015/02/10(火) 22:17:03.32 ID:???.net
>>891
Σ(`Д´ )マヂデ!?

909 :ご冗談でしょう?名無しさん:2015/02/10(火) 23:30:05.65 ID:???.net
>>905
その実験で風圧中心は測れないでしょ。
何で重力が無視されているの?
勿論紙飛行機を作る上では風圧と重力の釣り合いが問題なんだから
その実験をやることに意味はあるけどさ。
下手に風圧中心なんて言葉を出さない方がいいんじゃないの?

910 :ご冗談でしょう?名無しさん:2015/02/10(火) 23:35:11.74 ID:LOmW13K6.net
つぅかよぉ、ぽまいら、電車で車両間のドアがないつーつーな電車で
加速時と減速時で車内に風が吹き、かつそれぞれ逆方向であることを
説明できるやつどんだけいるの?
まあ、そもそもそういう現象があることに気づいた人なんざぁ俺ぐらいだと思うけどな

911 :ご冗談でしょう?名無しさん:2015/02/11(水) 00:23:16.00 ID:???.net
>>910
雨降りの日曜日で退屈していた四歳息子、PCをいじっていたが突然
「ママ!パパの写真、頭がたいへんなことになってる!!」と叫びだす。
どれどれと覗いてみると、どこから出したのか結婚数年前の夫が笑顔でそこにいた。

うん、今と髪型ずいぶん違うね。
君は知らないけど、むしろ「たいへんなことになってる」のは今のほうなんだ…

912 :ご冗談でしょう?名無しさん:2015/02/11(水) 00:41:12.12 ID:3H0LdRP5.net
それはそれはご愁傷さまでございました

913 :ご冗談でしょう?名無しさん:2015/02/11(水) 06:28:48.37 ID:???.net
>>909

881さま。返事ありがとうございます。
私の説明が分かりにくくて申し訳ございません

糸を持って走るとき糸は垂直方向にします。
すなわち風見鶏のような動きになります
ですから重力は無視できます。

また風圧中心でなければ厚紙が風の方向の斜めを向く
説明がつかないと思うのです。

それから風圧中心という言葉は出さないかもしれませんが、
上記の実験は家でも非常に簡単に行えて
それでいて厚紙が風に向かって斜めを向くという意外性も
十分あるので、この実験は是非紹介したいと考えております。

914 :ご冗談でしょう?名無しさん:2015/02/11(水) 09:36:42.22 ID:???.net
>>913
つまり、厚紙が鉛直方向を向くまで紐の位置を調整するわけか。
それなら測れるね。

915 :ご冗談でしょう?名無しさん:2015/02/11(水) 09:44:16.04 ID:???.net
>>905
揚力と抵抗力の作用点が風圧中心なのですか?
糸は端から4分の1でなくても斜めに安定するでしょうね

916 :ご冗談でしょう?名無しさん:2015/02/11(水) 10:57:36.37 ID:???.net
>>902

相対論によれば、重力波があり、その作用速度は光速度らしい
しかし、古典天文学では、重力作用速度が無限大として、
光速度との差で、見た目と実際の位置のずれとして、
光行差を計算する、しかも、その計算は合っている
もし重力波を採用するならば、光行差があるとした既存の天文定数、
惑星等の質量、距離などを全て破棄し、重力波が光速度を
満足するような定数に計算し直さなければならない


したがって、既存の天文定数を使っている限り、重力波が見つかることはない

917 :ご冗談でしょう?名無しさん:2015/02/11(水) 11:44:43.32 ID:???.net
日本語でおk

918 :ご冗談でしょう?名無しさん:2015/02/11(水) 11:52:06.37 ID:???.net
米海軍が発表 最新兵器「レールガン」の全貌
東洋経済オンライン 2月7日
http://zasshi.news.yahoo.co.jp/article?a=20150207-00060167-toyo-int

■ 200キロ先まで弾丸を飛ばせる! 

レールガンは火薬を使う大砲と比べて、より速く、より遠くへ飛ばし、そしてより大きな打撃を与えることができる。既存の海軍の主力といえる5インチ砲の射程距離は13カイリ(約24q)。
米海軍によると、レールガンの射程距離は110カイリ(約204q)。ちなみに、戦艦大和の主砲の射程距離は42q、御坂美琴の電磁砲の射程距離は25m に過ぎない。

米海軍研究所長マット・ウィンター准将は、誇らしげに語る。

919 :ご冗談でしょう?名無しさん:2015/02/11(水) 12:44:18.42 ID:???.net
重力波の速度と光の速度が一緒と言っていない文章だから、
>>916はゴミ文。NASに関わるのも無駄だが。
「光速度との差で、見た目と実際の位置のずれとして」この時点で光速度だよね。=>重力波の速度が無限大が破棄。
移動(事象)を観測した時には。一緒の速度という事を後半で、なぜか破棄する。
で、>>917だな。

920 :ご冗談でしょう?名無しさん:2015/02/11(水) 13:22:33.12 ID:???.net
>>910
何時も乗ってりゃ誰でも気付くだろ

921 :ご冗談でしょう?名無しさん:2015/02/11(水) 13:48:22.15 ID:???.net
光行差と重力は関係がないだろう
重力波と重力とは別だろう

922 :ご冗談でしょう?名無しさん:2015/02/11(水) 15:31:10.41 ID:???.net
平均応力について教えて下さい。
一般的に "最大応力と最小応力の平均値" ということですが、

例えば、家を支えている柱の平均応力は

1) 家の自重だけで発生する応力が平均応力。

2) 家の自重+変動荷重(地震や人・物の重量)を考慮した
  最大最小応力を平均した値が平均応力。

どちらが正解なのでしょうか?

923 :NAS6 ◆n3AmnVhjwc :2015/02/11(水) 15:35:05.44 ID:fpfa50Vb.net
>「光速度との差で、見た目と実際の位置のずれとして」この時点で光速度だよね。=>重力波の速度が無限大が破棄。

古典天文学での光行差は
実際は重力作用速度が無限大で惑星万有引力の計算をするんだけど
見た目の光速度と実際の位置のずれが光行差です

また、その光行差は地球から見た視位置の計算のみに使う

924 :ご冗談でしょう?名無しさん:2015/02/11(水) 15:55:02.02 ID:???.net
>914
>915

886さま、887さま。ご返事ありがとうございます。

> つまり、厚紙が鉛直方向を向くまで紐の位置を調整するわけか。

そうです。その通りです。糸を上下にぎゅっと引っ張ります。

> それなら測れるね。

という事は、この実験とこの説明はだいたい合っているという事ですね。
ありがとうございます。

> 揚力と抵抗力の作用点が風圧中心なのですか?

揚力、抵抗力話は私が下手な説明をするよりも
素晴らしい解説ページがあるので、それを紹介します
http://www.aero.kyushu-u.ac.jp/birdman/introduction/theory.html

> 糸は端から4分の1でなくても斜めに安定するでしょうね

自分で糸を1センチ刻みで10本ほどつけた紙を工作してやってみたのですが、
まあだいたい3分の1よりも内側の糸だと風の方向に対して
垂直の向きになります。

925 :NAS6 ◆n3AmnVhjwc :2015/02/11(水) 16:02:11.37 ID:fpfa50Vb.net
訂正

古典天文学での光行差は
実際は重力作用速度が無限大で惑星万有引力の計算をするんだけど
地球からの見た目の地球と惑星との距離からの光速度と時間の関係と
実際の位置のずれが光行差です

よって、その光行差は地球から見た視位置の計算のみに使う


ttp://ja.wikipedia.org/wiki/%E5%85%89%E8%A1%8C%E5%B7%AE

光行差

天体の位置計算 増補版 長沢工著
が古典天文学について詳しく載っている

この問題は誘導、こっちでね
ttp://wc2014.2ch.net/test/read.cgi/sci/1423607411/l50

926 :ご冗談でしょう?名無しさん:2015/02/11(水) 19:33:03.89 ID:???.net
>>910

私は流体力学は全くのしろうとですでが私の考えを述べます。
おそらく誰一人として説明できないと考えます。

そう考える論拠は以下に示します。

【1】私の質問に書いている「風圧中心が翼の前から四分の一」みたいなのは、
例えば模型飛行機工作の入門書に書いてあるレベルの話である。

【2】ライト兄弟が1903年に飛行機で飛んでから112年経過したが、
その112年で人間が作り出した「一般人向けの飛行機の説明」はこれである
「翼後端が尖っていてクッタ条件が満たされる時に、そこに渦ができ、
ヘルムホルツ第一原理を満たすために翼の周りに循環が発生し、
翼の上面の風が加速され、ベルヌーイの原理で揚力が発生する」

【3】↑の説明を読んで
http://www.aero.kyushu-u.ac.jp/birdman/introduction/theory.html
ここの「空気による合力、負圧、正圧」の図の圧力分布が思い描けますか?
私には無理です。この図があれば【1】の「風圧中心が翼の前から
四分の一」みたいなのは何となく想像できますが、そこまで辿りつけないのです。

927 :ご冗談でしょう?名無しさん:2015/02/11(水) 19:34:07.90 ID:???.net
>>925
光行差ではなく光差のことですね。これなら言いたいことはわかる。

928 :ご冗談でしょう?名無しさん:2015/02/11(水) 19:34:55.19 ID:???.net
【4】人間が100年考えた飛行機の「一般向け」の説明では、模型飛
行機工作の入門書の話すら説明できないのです。

【5】電車の話ですが、「空気による合力、負圧、正圧」の図を見てください。
場所によって圧力が違いますね。その2か所をホースでつないだら
風が流れると思いませんか?。電車の話も、おそらく窓が開いている車両が
あってこの空気の負圧、正圧みたいな圧力分布の話に帰結すると思います。
そして、この「圧力分布」の話は、人間が100年かけても「一般向け」の
説明では説明できないのです。

【6】余談となりますがwikipediaの「ナビエ-ストークス方程式の解の
存在と滑らかさ」に面白い事が書いてあった「これらの方程式は空間の
中の流体(つまり、液体や気体)の運動を記述する。(中略)しかし
ながら、これらの方程式の理論的な理解は不完全である(中略)
ナビエ?ストークス方程式の解の基本的性質さえ、証明されていない」

929 :ご冗談でしょう?名無しさん:2015/02/11(水) 19:41:43.08 ID:???.net
>>910
空気の慣性と思っていましたが、間違ってますか?

930 :ご冗談でしょう?名無しさん:2015/02/11(水) 19:54:05.17 ID:???.net
>>929
確かこれって加速時に前向きの風が吹き、減速時に後ろ向きの風が吹く話だったと思う。

931 :ご冗談でしょう?名無しさん:2015/02/11(水) 20:26:53.86 ID:???.net
>>929
空気が動こうとしても先が塞がってるんだから慣性は無理だわな
>>928みたいな隙間風も運転席が前に開いてない限り無理
隙間が横にある場合は霧吹きの原理(ベルヌーイの原理は間違い)で負圧にしかならんから近くの空気しか吸い込まず風にはならん
というわけで正解は…おしえなーい

932 :ご冗談でしょう?名無しさん:2015/02/11(水) 21:03:06.78 ID:???.net
実効的な重力の方向と、空気の温度勾配の方向が一致しなくなり対流が起きる。

933 :ご冗談でしょう?名無しさん:2015/02/11(水) 21:09:44.46 ID:???.net
車両の前後で気圧を測ったサイトを見かけたことがある。

空気って膨張・圧縮するよね♡

934 :ご冗談でしょう?名無しさん:2015/02/11(水) 21:10:13.66 ID:???.net
>>931
> 空気が動こうとしても先が塞がってるんだから慣性は無理だわな

元スレに加速減速書いてあるので、空気が圧縮されている可能性はある。
車両間が空いている電車と書いてあるので、かなりの量の空気の話になるし。

> 隙間が横にある場合は霧吹きの原理(ベルヌーイの原理は間違い)で

霧吹きの原理が何者か書いてないので、説明になっていないような

935 :ご冗談でしょう?名無しさん:2015/02/11(水) 21:12:46.18 ID:???.net
>>932
> 実効的な重力の方向と、空気の温度勾配の方向が一致しなくなり対流が起きる。

これ面白そうだね。上と下で逆向きの風が流れるわけか。
こんど電車に乗ったら調べて見る

936 :ご冗談でしょう?名無しさん:2015/02/11(水) 23:29:04.69 ID:???.net
###重力波が光速度っていうウソ### [転載禁止]©2ch.net
http://wc2014.2ch.net/test/read.cgi/sci/1423607411/

937 :ご冗談でしょう?名無しさん:2015/02/12(木) 00:39:45.03 ID:???.net
>>932
「温度勾配」でなくて「密度勾配」なら正しい説明になるんだがな。

938 :ご冗談でしょう?名無しさん:2015/02/12(木) 00:39:56.69 ID:???.net
ゴミ巣を貼るな

939 :ご冗談でしょう?名無しさん:2015/02/12(木) 00:54:23.54 ID:???.net
>>937
密度の違いの主因は温度だろうから結局同じじゃね

940 :ご冗談でしょう?名無しさん:2015/02/12(木) 01:06:47.10 ID:???.net
>>933
概算してみよう
空気の比重は窒素分子の分子量が56だから56g/(22.4×1000cc)=0.0025
比重1の水は1Gの重力で10mで1気圧だから空気なら同じく1Gで10m/0.0025=4000mで1気圧
電車の長さを50mとして横に加速度が0.1G働くとすると気圧差は0.1×50m/4000m=0.00125気圧
4車両分の空気に気圧差が働くと体積変化は4車両×0.00125=0.005車両
つまり1車両分の空気の0.5%が動く事になる
そよ風くらいにはなるかなー?

941 :ご冗談でしょう?名無しさん:2015/02/12(木) 02:14:22.98 ID:???.net
>>940
は?

942 :ご冗談でしょう?名無しさん:2015/02/12(木) 02:23:40.35 ID:???.net
うわ、安価ミスった>>941>>939宛ね。

943 :ご冗談でしょう?名無しさん:2015/02/12(木) 04:45:08.87 ID:eRDspUTj.net
http://nlab.itmedia.co.jp/nl/articles/1409/20/news017.html
http://jp.reuters.com/article/marketsNews/idJPL3N0MG13T20140319、横国大、量子テレポーテーションを可能にする新原理を実証
http://www.scienceplus2ch.com/archives/4963081.html、プログラミング、http://chinareaction.com/blog-entry-890.htmlで、
仮想空間ができる。ソードアートオンラインのような電脳世界ができ、実際のセックスもできる。登記

944 :ご冗談でしょう?名無しさん:2015/02/12(木) 05:20:28.91 ID:???.net
量子の世界では「時間の逆流」が起こっている 未来を知ることで過去が変化
http://daily.2ch.net/test/read.cgi/newsplus/1423671633/
カーター・マーチ教授は、粒子の未来の状態を知ることによって、
その粒子の過去の状態が変化することに気付いた。つまり、未来の事象を知ることによって
過去を変えることができるということであり、これがもしも古典力学の世界にもあてはまるとしたら、
私たちが現在取っている行動は、未来の私たちの意思決定によって影響を受けていることになる。

945 :ご冗談でしょう?名無しさん:2015/02/12(木) 05:27:09.03 ID:???.net
MARCHレベルだなw

946 :ご冗談でしょう?名無しさん:2015/02/12(木) 05:38:21.59 ID:eRDspUTj.net
http://nlab.itmedia.co.jp/nl/articles/1409/20/news017.html
http://jp.reuters.com/article/marketsNews/idJPL3N0MG13T20140319、横国大、量子テレポーテーションを可能にする新原理を実証
http://www.scienceplus2ch.com/archives/4963081.html、プログラミング、http://chinareaction.com/blog-entry-890.htmlで、
仮想空間ができる。ソードアートオンラインのような電脳世界ができ、実際のセックスもできる。登記

947 :ご冗談でしょう?名無しさん:2015/02/12(木) 05:40:49.44 ID:???.net
スペクトル項に4Pと4P°って表記があったんですが°の有無はどのような違いによりますか?

948 :ご冗談でしょう?名無しさん:2015/02/12(木) 07:23:07.01 ID:???.net
>>931
> 隙間が横にある場合は霧吹きの原理(ベルヌーイの原理は間違い)で

霧吹きの原理はこれのことか?
http://www.jsme-fed.org/experiment/2013_8/003.html
http://www.geocities.jp/f1tw_idea/aerodynamics/drag/report.html
http://ja.wikipedia.org/wiki/%E5%BE%8C%E6%96%B9%E4%B9%B1%E6%B0%97%E6%B5%81

つまり後方乱気流に引っ張られるという話みたいですね

949 :ご冗談でしょう?名無しさん:2015/02/12(木) 09:16:12.90 ID:???.net
物理を勉強する意味って何ですか?

950 :ご冗談でしょう?名無しさん:2015/02/12(木) 10:16:01.70 ID:???.net
>>949
それよりオマエが生きてる意味を考えろ

951 :ご冗談でしょう?名無しさん:2015/02/12(木) 10:23:18.59 ID:???.net
>>950
それよりテメエが生きてる意味を考えろ

952 :ご冗談でしょう?名無しさん:2015/02/12(木) 10:43:18.43 ID:???.net
>>949
人類最高の英知を追体験したい。
自然の法則と神の意思を知りたい。

と言うのは、表向きの理由で、
理系の方が就職有利だから。

文系よりスペック高そうな人間と思われるだろうから、
何かと都合がいい。

と、オレのトモダチの友達が言ってた。

953 :ご冗談でしょう?名無しさん:2015/02/12(木) 11:03:06.48 ID:???.net
物理の質問にのみ答えろ

954 :ご冗談でしょう?名無しさん:2015/02/12(木) 11:24:15.91 ID:???.net
あなたもネットで告発しませんか?

ビッ苦カメラ札幌店 佐藤伸弦 暴行事件

955 :ご冗談でしょう?名無しさん:2015/02/12(木) 11:44:05.56 ID:???.net
ゴミは要らん

956 :ご冗談でしょう?名無しさん:2015/02/12(木) 11:49:27.12 ID:???.net
偉そうだな。
なんでそんなに偉そうなんだ?

957 :ご冗談でしょう?名無しさん:2015/02/12(木) 14:16:45.16 ID:???.net
偉そうかな?素直な感想に過ぎんと思うが。

958 :ご冗談でしょう?名無しさん:2015/02/12(木) 16:48:37.47 ID:tS2L0FSh.net
>>944
それどうよ。+の訳はちんぷんかんぷんだが、ホィーラーの遅延選択実験の結果や
量子エンタングルメントの非局所性を説明する助けにはならないの?

959 :ご冗談でしょう?名無しさん:2015/02/12(木) 17:43:56.83 ID:???.net
ホィーラーの遅延選択実験を追試して、
その結果から仮説を構築したのが>>944ってことでしょ。

960 :ご冗談でしょう?名無しさん:2015/02/12(木) 18:15:02.11 ID:???.net
そもそも宇宙とは何ですか?

961 :ご冗談でしょう?名無しさん:2015/02/12(木) 18:18:00.06 ID:???.net
スピン注入について質問があります。
強磁性体と非磁性体を接合し電流を流したとき↑と↓のスピンの状態密度の差により、
非磁性体側の界面付近にスピン偏極した電流が流れる現象を勉強したのですが
非磁性体側のスピン拡散長が大きいほどスピンが注入されにくくなるのはなぜですか?
スピン注入効率の式からそうなるというのはわかるのですが
物理的には何が起こってスピン注入が抑制されるのかがわかりません。

962 :ご冗談でしょう?名無しさん:2015/02/12(木) 18:29:43.48 ID:1EuAuJ7d.net
長文ですが読んでいただけると幸いです高校二年生で物理の研究をしてるものです 授業は物理基礎の範囲を履修してます
エッグドロップについて研究していて、紙で作った卵を覆うプロテクターを製作し、
卵の代わり加速度センサーを入れて10メートルの高さから落としてその際の加速度を計測するという実験を行っています
その加速度なんですが10メートルの高さから落とした瞬間は0になり、落下した瞬間は下向きつまり地面に向かう加速度となりました
この下向きの加速度が生じる理由がわからず研究が止まってしまいました
当たった時の衝撃で上向きの加速度が生じるという仮説を立てたんですか…
下向きの加速度が生じる理由として考えられる要因を教えてください

963 :ご冗談でしょう?名無しさん:2015/02/12(木) 18:34:17.73 ID:???.net
>>962
止まったから、重力加速度を測ってんじゃないの?

964 :ご冗談でしょう?名無しさん:2015/02/12(木) 19:06:50.41 ID:1EuAuJ7d.net
935さんコメントありがとうございます
ごめんなさい、間違えました
説明下手なので1から説明させていただきます
落下させる前の状態では重力が働き下向きの加速度がみられ、落下開始時には加速度は自由落下するため0となるが空気抵抗が少しずつ大きくなり下向きの加速度が働きます
そして落下した瞬間は上向きの加速度が見られました
しかし落下時のセンサーに加わる力は空気抵抗と同じく落下運動を妨げる下向きの加速度が見られるべきなのではないかという考えです

965 :ご冗談でしょう?名無しさん:2015/02/12(木) 19:08:08.65 ID:???.net
そもそも静止してる時はどうなんだ?

966 :ご冗談でしょう?名無しさん:2015/02/12(木) 19:15:50.92 ID:1EuAuJ7d.net
静止してるときには下向きの加速度です

967 :ご冗談でしょう?名無しさん:2015/02/12(木) 20:02:00.42 ID:???.net
>>964
俺が勘違いしてるのかもしれないが、空気抵抗で減速してるのだから上向きの加速度じゃないのか。
現に落下した瞬間は上向きの加速度が観測されている(瞬間過ぎて計測できていないのかと思った)。
ちゃんと測定できてないのでは。

968 :882:2015/02/12(木) 20:47:40.17 ID:HHoA0Slx.net
>>910です。
まさか、こんなに多くの方に考えていただきレスをいただけるとは
思っていませんでした。まずお礼をさせてください。どうも有り難うざいます。
そして、変な言葉遣いで書き込んでしまったことを謝ります。

さて、私の考察なのですが、端的に言うと>>929さんと同じです。
どこかの車両の窓が開いている想定はしていませんでしたが、
仮に全ての窓が閉まっていたとしても起こりうると考えています。
その理由は、車両がものすごく長いからです。山手線とかだと
10両以上ありますよね?そうすると、両端が塞がったとても細長い
管のような状況になっていてその中に空気が溜まっている感じです。
そして、このくらい長いと、車両の両端から押されて空気が車両と
共に動き出すまでにかなりのタイムラグが生じると考えています。
そのため、一緒に空気が動き出すまでの間、電車に乗ってるひとから
見ると風が吹いているように感じられるのだと思います。
この仮説を裏付けるために、試しに端っこの先頭車両に乗ってみたことがあります
そうすると、風は吹きませんでした。逆に、中央付近の車両が最も
風が強く感じられました。そのことからも、概ね上記のような理由で
風が吹くのかなと思っています。ちなみに、風の吹く向きは、加速時は
後ろ向きに、減速時は進行方向に吹いていました。

969 :ご冗談でしょう?名無しさん:2015/02/12(木) 21:24:43.36 ID:???.net
窓が締まっていた場合に体感出来るほどの空気の移動があるかなあ?
その場合、つまり、列車の加速時には後部のほうに空気が圧縮されるってことだろ?
風を感じるほど圧縮するとしたら、前部の方では空気薄くなっちゃうほどってことじゃないか?

970 :ご冗談でしょう?名無しさん:2015/02/12(木) 21:32:49.49 ID:???.net
電車の気密性なんてザルだろ

971 :ご冗談でしょう?名無しさん:2015/02/12(木) 21:43:45.52 ID:???.net
電車の加速なんて0.1G程度。
その程度で高々数百mで生じる気圧差なんて微々たるもの。
もし、気密性があったとすると風を体感出来るとは思えない。
つまり、体感出来るということはオープンカーで風を感じるのに準じた現象が起きているに過ぎない。

972 :ご冗談でしょう?名無しさん:2015/02/12(木) 21:46:22.30 ID:???.net
電車より大きい加速度の飛行機の離陸時に風を感じるのかって事だな。
俺は感じたことねえよ。

973 :882:2015/02/12(木) 21:46:42.02 ID:HHoA0Slx.net
>>969
>その場合、つまり、列車の加速時には後部のほうに空気が圧縮されるってことだろ?
>風を感じるほど圧縮するとしたら、前部の方では空気薄くなっちゃうほどってことじゃないか?
はい、そうなんだと思っています。
車両編成が短い場合は、空気の流れが生まれるためには大きな圧力差が必要だと
思いますが、それが長い編成になると、そこまで大きな圧力差でなくとも、空気の量自体が
大きいので、ちょっとした圧力差でも、その圧力差を緩和するまでに流れる空気の絶対量が増え、
かつ圧力差が緩和されるまでの時間も長くなるので、体感できる風の流れとして現れるのではないかと
思っています。
ただし、>>970さんのおっしゃる通り、機密性が高いわけではないので、上記以外の要因を
完全に否定しているわけではありません。

974 :882:2015/02/12(木) 21:49:31.79 ID:HHoA0Slx.net
この仮説を実証するためには、比較的好感度の気圧計を持って電車に乗り込み
例えば加速時に先頭車両では気圧が下がり、最後尾車両では気圧が上がる
様子が観測できれば、確からしさを高めることはできると思います。
逆に、ほとんど気圧差が生まれていない(先頭車両から最後尾車両まで気圧分布が
一定である)のなら、この仮説は反証されることになります。

975 :ご冗談でしょう?名無しさん:2015/02/12(木) 21:58:26.62 ID:???.net
>>969
pt=nrt
また電車は1分程度で巡航速度の時速130キロまで加速する。
で、その上流に電車10台分の空気があるから、
電車内の空気の体積は82.5%に圧縮される

976 :882:2015/02/12(木) 21:58:57.63 ID:HHoA0Slx.net
>>971
なぜ今の所このような見解を持っているかというと、注意深く観測していると、
風が生まれるタイミングは決まって加速度があるときだったからです。
例えば、加速時や減速時です。
加速し終わって、一定速度になった車両(つまりほぼ等速運動時)は、
車両が走っていても時間が経つと風が止んでしまいます。

>>972
飛行機の場合は、加速度は大きいですが、断面積と長さの比が、長編成車両ほど
大きくないため、感じられるほどの流量がないまま、空気が機体と一緒に動き出してしまう
のかなと想像します。

977 :ご冗談でしょう?名無しさん:2015/02/12(木) 22:03:19.17 ID:???.net
空気1リットルの重さは、約1.2g
電車一両の長さが10メートルとして、
それが10両つながっていたら、
長さは100メートル。重さは1平方メートルあたり100グラムの質量となる

978 :ご冗談でしょう?名無しさん:2015/02/12(木) 22:05:46.63 ID:???.net
>>972

雷みたいな大電圧であっても、避雷針などの電気の逃げ道があるとぜんぜんビリビリこないよね

電車の連結部分に人が立つと、空気の流れの逃げ道が比較的小さくなる

979 :ご冗談でしょう?名無しさん:2015/02/12(木) 22:11:03.30 ID:???.net
>>964

加速度センサーの
スペック調べてみるがいいんでないかい

地面にぶつかった直後は物質の反発率をかけた係数で上に運動するが、
それが動ける上限にぶつかるとか。

つまり、エレベータの中にビックリボールを入れて、
エレベータごと地面に落としたら、

地面にぶつかった直後ビックリボールは飛び上がるが
エレベータの天井にぶつかり下向きの加速度を得るみたいな

980 :ご冗談でしょう?名無しさん:2015/02/12(木) 22:13:25.94 ID:???.net
電車が加速すると、電車内の空気の分子は加速度の影響を受ける。加速というのは重力と等価だから、
高空では空気が薄くて地表近くでは濃いというのと同じだ(上下が左右になっただけ)。だから、しばらくの
間は電車後方に向けて空気の分子は移動する。これが「風」だ。

電車の加速度が一定の場合、やがて定常状態に達する。だから「風」は止む。電車前方では空気密度は
低く、後方では高いままだ。

電車がブレーキをかけると、天地(左右)がひっくり返る。だからしばらく「風」が生まれ、減速が一定だと
再び定常状態に達して「風」は止む。

そんなけのことだろう。

981 :ご冗談でしょう?名無しさん:2015/02/12(木) 22:18:57.89 ID:???.net
>>964
まず加速度の向きについてだけどさ、
>落下させる前の状態では重力が働き下向きの加速度がみられ
この状態でのその物体の加速度は、君達観察者から見れば0、自由落下する系から見れば上向きでしょ。
それを下向きと言ってるのは、単に加速度計の計測している(試験質量に働く)力(この場合重力)の向きを加速度の向きと誤解してるんじゃない?

で本題だけど、
>かし落下時のセンサーに加わる力は空気抵抗と同じく落下運動を妨げる下向きの加速度が見られるべきなのではないかという考えです
これはまあ当然の疑問で、私だったらまずその加速度計の測定範囲と時間応答性をチェックするな。

982 :ご冗談でしょう?名無しさん:2015/02/12(木) 22:19:10.52 ID:???.net
加速時には減圧している前方の隙間から外気が流入、加圧している後方の隙間から流出し、前方から後方へ風が吹き続ける
減速時には減圧している後方の隙間から外気が流入、加圧している前方の隙間から流出し、後方から前方へ風が吹き続ける
こんなんじゃないの?

飛行機も気密がなければ風が吹くだろうし、列車も気密があればわずかな空気の移動で圧力勾配ができ、風も持続しないんじゃないの

983 :ご冗談でしょう?名無しさん:2015/02/12(木) 22:27:29.65 ID:???.net
>>982
気密云々は枝葉末節のような気がするな。俺は専門家じゃないし、偉そうに聞こえれば勘弁。

984 :882:2015/02/12(木) 22:39:28.40 ID:HHoA0Slx.net
>>980
そうですね。
それだけのことと言ってしまえばそうなんだと思います…。

>>982
確かにそういった現象も複合的に起こっている可能性は十分にありますね。
隙間風がビュービュー吹き込んでいる電車はあまり見たことはないですが、一方で
車両が長ければ、ちょっとした隙間風もチリツモで車両中央付近まで足し合わせた
ころには、風として感じられる流れの一部として寄与している可能性はあると思います。

985 :ご冗談でしょう?名無しさん:2015/02/12(木) 22:41:42.62 ID:???.net
>>983
気密については、圧力勾配が形成されて加速度一定の定常状態になった時に、
風が止むか吹き続けるかの違いに関わると思うけど

986 :ご冗談でしょう?名無しさん:2015/02/12(木) 22:43:41.52 ID:???.net
>>985
>>976

987 :ご冗談でしょう?名無しさん:2015/02/12(木) 22:54:32.30 ID:???.net
>>986
加速度一定と速度一定は違うぞ?
>>976速度一定の時、電車では風がやんでいるという話だろ

988 :ご冗談でしょう?名無しさん:2015/02/12(木) 22:56:50.91 ID:???.net
>>976は速度一定の時、…

989 :ご冗談でしょう?名無しさん:2015/02/12(木) 22:59:45.99 ID:???.net
>>987
確かにそうだ、俺がボケていた(それはいつもだが)。定速運行に移行する前に風が吹いていたのだが
(加速度もしくは重量勾配が無くなったのだから)、本人が気づいていなかっただけだろうな。

990 :882:2015/02/12(木) 23:11:59.05 ID:HHoA0Slx.net
私が体験したのは、等速運動時に風が止んでいた事象ですが、
仮に等加速度運動が長い間続くようなことがあれば、機密性が
高ければいずれある圧力勾配で維持され、風は止むと思います

ただ、実際の電車の運行では等加速度運動が長い間続くことはなく
体験として語れる内容はありません

991 :ご冗談でしょう?名無しさん:2015/02/13(金) 00:12:33.27 ID:???.net
連結部分のベンチュリ効果忘れんじゃねーぞ

992 :ご冗談でしょう?名無しさん:2015/02/13(金) 03:19:41.49 ID:???.net
Nelsonの確率量子力学を数式レベルでちゃんと学べる本はあります
でしょうか?

今のところFoward微分(?)やBackward微分(?)、
ウィーン(?)過程(忘れました)などの意味が
分からないのでそれらが分かるようになることが目標です。

洋書(英語)でも構いませんが、省略や他の参考文献を読まなくても
分かるようになっている物であればと思います。

993 :ご冗談でしょう?名無しさん:2015/02/13(金) 07:59:04.92 ID:???.net
その加速度センサーの製品名を書け

994 :ご冗談でしょう?名無しさん:2015/02/13(金) 08:40:09.18 ID:???.net
>>992
Nelson先生のレビュー
http://iopscience.iop.org/1742-6596/361/1/012011/pdf/1742-6596_361_1_012011.pdf

本人のHPに引用文献にある本がおいてある

Wiener過程(ブラウン運動)を理解しなくてはいけないので一冊では難しいと思うけど

995 :ご冗談でしょう?名無しさん:2015/02/13(金) 10:15:07.59 ID:???.net
>>992
江沢先生による確率過程量子化の解説
物理学の視点-力学、確率、量子
の12章

996 :ご冗談でしょう?名無しさん:2015/02/14(土) 00:20:46.56 ID:???.net
>>994
>本人のHPに引用文献にある本がおいてある

http://iopscience.iop.org/1742-6596/361/1
のことでしょうか?

>>994のpdfの最後の「Reference」にある著者名を検索してみても
見つからないようですが。

997 :ご冗談でしょう?名無しさん:2015/02/14(土) 00:36:07.30 ID:???.net
書籍についての質問なのですが、よろしいでしょうか。

朝永振一郎氏の「量子の裁判」に書かれているように、素粒子が自分自身と干渉する(素人なので変な表現だったらすいません)という考え方に興味があります。
光の干渉において、なぜ他でもなく、粒子が左右のスリットを同時に通っているという考え方が合理的なのか、詳しく解説している本はありますか? 素人考えでは、もっと他に解釈のしようはないのかと思ってしまいます。
一般向けの本だと助かりますが、多少の数式が書いてあっても大丈夫です。よろしくお願いします。

998 :ご冗談でしょう?名無しさん:2015/02/14(土) 00:40:10.31 ID:0GjMFMbk.net
>>812
バネの数を2倍増やしたら加速が2倍になったししかもベクトル法則が使える!線形性これは使える!
ってな感じじゃないの? 実際どうだったか知らないけど
質量や力などは経験的なくるところだと思う その泥臭さが気に食わないのかもね

999 :779:2015/02/14(土) 00:41:46.62 ID:D8WQehbb.net
ばね定数98N/mの軽いばねを天井からつるし、
その先端に質量2.0sのおもりをつるした。
ばねの長さが自然になる位置で静かに手を離すと、
おもりは釣り合いの位置Oを中心に振動した

おもりが最下点に達したとき、ばねの伸びはいくらか

この問題なのですが方針がわかりません

1000 :ご冗談でしょう?名無しさん:2015/02/14(土) 00:57:56.28 ID:???.net
宿題なら自力で。

1001 :ご冗談でしょう?名無しさん:2015/02/14(土) 01:28:18.56 ID:???.net
>>999
http://journal.simplesso.jp/wp-content/uploads/20120609image02.png

1002 :ご冗談でしょう?名無しさん:2015/02/14(土) 03:38:18.95 ID:siYhp7aJ.net
なんで物理板にはキチガイばかり集まるんですか?

1003 :ご冗談でしょう?名無しさん:2015/02/14(土) 11:26:31.60 ID:pcSwvxlzD
自分に聞いてみたら?

1004 :ご冗談でしょう?名無しさん:2015/02/14(土) 10:03:51.51 ID:???.net
>>996
ここです
https://web.math.princeton.edu/~nelson/books.html

1005 :ご冗談でしょう?名無しさん:2015/02/14(土) 10:37:43.12 ID:???.net
>>999
授業聞いてないだろお前

1006 :ご冗談でしょう?名無しさん:2015/02/14(土) 11:01:51.10 ID:???.net
ケプラーは万有引力の原因を磁気力と考えていたんだな。

1007 :ご冗談でしょう?名無しさん:2015/02/14(土) 11:08:26.59 ID:???.net
>>997
量子力学でミクロ粒子の位置・運動量等は確率でしか予測できない。(不確定性原理)
現在までのあらゆる実験と矛盾しない、決定論的な解釈は人為的な仮説の追加が必要。

1008 :ご冗談でしょう?名無しさん:2015/02/14(土) 12:49:22.28 ID:???.net
量子力学自体が仮説なんだけどね

1009 :ご冗談でしょう?名無しさん:2015/02/14(土) 12:56:18.07 ID:???.net
間様の御成りw

1010 :ご冗談でしょう?名無しさん:2015/02/14(土) 13:32:13.59 ID:???.net
あらゆる原理、法則は、究極的には仮説である。

1011 :ご冗談でしょう?名無しさん:2015/02/14(土) 13:35:43.09 ID:???.net
個人的な好奇心で聞くんだけど
逆2乗則を数理物理的に導出しようっていう試みってないの?
未だに実験的には厳密に2乗なのかどうか定まってないとは聞いたけど

1012 :ご冗談でしょう?名無しさん:2015/02/14(土) 13:46:30.53 ID:tELNGjnP.net
逆2乗則が成立しない世界を解いてみれば

1013 :ご冗談でしょう?名無しさん:2015/02/14(土) 14:06:57.19 ID:???.net
メコス乗則が成立しない股間を舐めてみれば

1014 :ご冗談でしょう?名無しさん:2015/02/14(土) 17:30:57.08 ID:???.net
>>1011
>逆2乗則を数理物理的に導出しようっていう試みってないの?

力が伝わる仮想的な線の束の密度が距離の2乗に反比例するというアイデアから
ポアソン方程式に至るまで、とっくの昔に考え尽くされている。

>未だに実験的には厳密に2乗なのかどうか定まってないとは聞いたけど

実験とはそういうものだ。何かの数字を厳密に定められるわけがない。
しかし、たとえばクーロン則の2乗からのズレが、
0.0000000000000001の精度の評価実験で検出されないことを知らずに
そういうことを語ってはいけない。

1015 :ご冗談でしょう?名無しさん:2015/02/14(土) 17:47:43.20 ID:???.net
「個人的な好奇心」、「とは聞いたけど」、ブルーバックスでも読めよ

1016 :ご冗談でしょう?名無しさん:2015/02/14(土) 19:35:38.27 ID:TF2+7HfB.net
余剰次元があると重力逆2乗則が破れる、という話は聞いたな。否定するほうの
モデルだろうが。

1017 :ご冗談でしょう?名無しさん:2015/02/14(土) 19:44:26.81 ID:tELNGjnP.net
ファインマン物理学III電磁気学のP59-62で考察あるな
逆2乗則

1018 :ご冗談でしょう?名無しさん:2015/02/23(月) 18:18:06.62 ID:t0C4nUqC/
>>1016
リサちゃんの一発理論、実験と合わず破綻したような、NHKよいしょ倒れ。

1019 :ご冗談でしょう?名無しさん:2015/03/23(月) 14:15:13.11 ID:JYHyh3DzU
船殻をたった一つの素粒子「ヱルトリウム」(初期設定では一個の水素原子)で構築しているため、
理論上は「反ヱルトリウム」との対消滅でしか破壊できない[1]うえ、
「ヱルトリウム」は人工素粒子であるために「反ヱルトリウム」が存在
せず破壊不可能とされている。素粒子「ヱルトリウム」は、型のような
物を先に作り、その中に「ヱルトリウムのもと」を流し込んで成形され
る.ランナーのような物に覆われている。
デルタ鉄鋼か、デルタ宝石 上記 登記

1020 :ご冗談でしょう?名無しさん:2015/04/17(金) 11:05:18.66 ID:/5CLtO9TW
特殊系超巨大プラチナドラゴンのドラゴンドロップの作り方 ポカリスエットに、白砂糖をいれて、生命の水を作る。それに、上級剣聖神癒癒し回復食を繰り返したのを入れる。それに、ポッカの緑のキレイトを入れる。 登記
ハゲが治る。登記

1021 :ご冗談でしょう?名無しさん:2015/04/18(土) 20:29:54.33 ID:j2Vgr/83g
上級剣聖神癒癒し回復食を入れたウォーターベットを作る 登記

1022 :ご冗談でしょう?名無しさん:2015/04/18(土) 20:55:25.73 ID:j2Vgr/83g
牛乳でつくったやつのを入れたウォーターベットを作る 登記

1023 :ご冗談でしょう?名無しさん:2015/05/16(土) 06:20:42.54 ID:H1tMIGpB+
名前:ご冗談でしょう?名無しさん :2015/05/16(土) 06:09:46.46 ID:H1tMIGpB+
魔法石でディストンションフィールドを作ると魔法ディストンション
フィールドができる 上記 登記

153 名前:ご冗談でしょう?名無しさん :2015/05/16(土) 06:11:19.53 ID:H1tMIGpB+
貴せい石でディストンションフィールドを作ると貴せい石のディストンション
フィールドができる 上記 登記

154 名前:ご冗談でしょう?名無しさん :2015/05/16(土) 06:12:03.09 ID:H1tMIGpB+
貴世石でディストンションフィールドを作ると貴世石のディストンション
フィールドができる 上記 登記

155 名前:ご冗談でしょう?名無しさん :2015/05/16(土) 06:12:43.01 ID:H1tMIGpB+
貴世世石でディストンションフィールドを作ると貴世世石のディストンション
フィールドができる 上記 登記

156 名前:ご冗談でしょう?名無しさん :2015/05/16(土) 06:13:06.53 ID:H1tMIGpB+
貴世世世石でディストンションフィールドを作ると貴世世世石のディストンション
フィールドができる 上記 登記

157 名前:ご冗談でしょう?名無しさん :2015/05/16(土) 06:13:35.66 ID:H1tMIGpB+
貴世世世世石でディストンションフィールドを作ると貴世世世世石のディストンションフィールドができる 上記 登記

158 名前:ご冗談でしょう?名無しさん :2015/05/16(土) 06:14:09.64 ID:H1tMIGpB+
貴世世世世世石でディストンションフィールドを作ると貴世世世世世石のディストンションフィールドができる 上記 登記

1024 :ご冗談でしょう?名無しさん:2015/05/16(土) 06:32:10.59 ID:7FYVgp1Lc
貴世石でディストンションフィールドを作ると貴世石のディストンション
フィールドができる 上記 登記
155 :ご冗談でしょう?名無しさん:2015/05/16(土) 06:12:43.01 ID:H1tMIGpB+[4/9]
貴世世石でディストンションフィールドを作ると貴世世石のディストンション
フィールドができる 上記 登記
156 :ご冗談でしょう?名無しさん:2015/05/16(土) 06:13:06.53 ID:H1tMIGpB+[5/9]
貴世世世石でディストンションフィールドを作ると貴世世世石のディストンション
フィールドができる 上記 登記
157 :ご冗談でしょう?名無しさん:2015/05/16(土) 06:13:35.66 ID:H1tMIGpB+[6/9]
貴世世世世石でディストンションフィールドを作ると貴世世世世石のディストンションフィールドができる 上記 登記
158 :ご冗談でしょう?名無しさん:2015/05/16(土) 06:14:09.64 ID:H1tMIGpB+[7/9]
貴世世世世世石でディストンションフィールドを作ると貴世世世世世石のディストンションフィールドができる 上記 登記
159 :ご冗談でしょう?名無しさん:2015/05/16(土) 06:23:05.84 ID:H1tMIGpB+[8/9]
貴世世世世世世石でディストンションフィールドを作ると貴世世世世世世石のディストンションフィールドができる 上記 登記
160 :ご冗談でしょう?名無しさん:2015/05/16(土) 06:23:42.15 ID:H1tMIGpB+[9/9]
貴世世世世世世世石でディストンションフィールドを作ると貴世世世世世世世石のディストンションフィールドができる 上記 登記
161 :ご冗談でしょう?名無しさん:2015/05/16(土) 06:29:22.08 ID:7FYVgp1Lc[1/2]
貴世世世世世世世世石でディストンションフィールドを作ると貴世世世世世世世世石のディストンションフィールドができる 上記 登記

貴世世世世世世世世世石でディストンションフィールドを作ると貴世世世世世世世世世石のディストンションフィールドができる 上記 登記

貴世世世世世世世世世世石でディストンションフィールドを作ると貴世世世世世世世世世世石のディストンションフィールドができる 上記 登記
162 :ご冗談でしょう?名無しさん:2015/05/16(土) 06:30:00.70 ID:7FYVgp1Lc[2/2]
貴世世世世世世世世世世世石でディストンションフィールドを作ると貴世世世世世世世世世世世石のディストンションフィールドができる 上記 登記

1025 :ご冗談でしょう?名無しさん:2015/05/16(土) 06:37:42.24 ID:7FYVgp1Lc
貴世石でディストンションフィールドを作ると貴世石のディストンション
フィールドができる 上記 登記

154 名前:ご冗談でしょう?名無しさん :2015/05/16(土) 06:12:03.09 ID:H1tMIGpB+
貴世石でディストンションフィールドを作ると貴世石のディストンション
フィールドができる 上記 登記

155 名前:ご冗談でしょう?名無しさん :2015/05/16(土) 06:12:43.01 ID:H1tMIGpB+
貴世世石でディストンションフィールドを作ると貴世世石のディストンション
フィールドができる 上記 登記
上記 登記

1026 :ご冗談でしょう?名無しさん:2015/05/16(土) 06:38:46.38 ID:7FYVgp1Lc
156 名前:ご冗談でしょう?名無しさん :2015/05/16(土) 06:13:06.53 ID:H1tMIGpB+
貴世世世石でディストンションフィールドを作ると貴世世世石のディストンション
フィールドができる 上記 登記

157 名前:ご冗談でしょう?名無しさん :2015/05/16(土) 06:13:35.66 ID:H1tMIGpB+
貴世世世世石でディストンションフィールドを作ると貴世世世世石のディストンションフィールドができる 上記 登記

158 名前:ご冗談でしょう?名無しさん :2015/05/16(土) 06:14:09.64 ID:H1tMIGpB+
貴世世世世世石でディストンションフィールドを作ると貴世世世世世石のディストンションフィールドができる 上記 登記

159 名前:ご冗談でしょう?名無しさん :2015/05/16(土) 06:23:05.84 ID:H1tMIGpB+
貴世世世世世世石でディストンションフィールドを作ると貴世世世世世世石のディストンションフィールドができる 上記 登記

160 名前:ご冗談でしょう?名無しさん :2015/05/16(土) 06:23:42.15 ID:H1tMIGpB+
貴世世世世世世世石でディストンションフィールドを作ると貴世世世世世世世石のディストンションフィールドができる 上記 登記

161 名前:ご冗談でしょう?名無しさん :2015/05/16(土) 06:29:22.08 ID:7FYVgp1Lc
貴世世世世世世世世石でディストンションフィールドを作ると貴世世世世世世世世石のディストンションフィールドができる 上記 登記

貴世世世世世世世世世石でディストンションフィールドを作ると貴世世世世世世世世世石のディストンションフィールドができる 上記 登記

貴世世世世世世世世世世石でディストンションフィールドを作ると貴世世世世世世世世世世石のディストンションフィールドができる 上記 登記

162 名前:ご冗談でしょう?名無しさん :2015/05/16(土) 06:30:00.70 ID:7FYVgp1Lc
貴世世世世世世世世世世世石でディストンションフィールドを作ると貴世世世世世世世世世世世石のディストンションフィールドができる

1027 :ご冗談でしょう?名無しさん:2015/05/16(土) 07:02:57.84 ID:7FYVgp1Lc
157 名前:ご冗談でしょう?名無しさん :2015/05/16(土) 06:13:35.66 ID:H1tMIGpB+
貴世世世世石でディストンションフィールドを作ると貴世世世世石のディストンションフィールドができる 上記 登記

158 名前:ご冗談でしょう?名無しさん :2015/05/16(土) 06:14:09.64 ID:H1tMIGpB+
貴世世世世世石でディストンションフィールドを作ると貴世世世世世石のディストンションフィールドができる 上記 登記

159 名前:ご冗談でしょう?名無しさん :2015/05/16(土) 06:23:05.84 ID:H1tMIGpB+
貴世世世世世世石でディストンションフィールドを作ると貴世世世世世世石のディストンションフィールドができる 上記 登記

160 名前:ご冗談でしょう?名無しさん :2015/05/16(土) 06:23:42.15 ID:H1tMIGpB+
貴世世世世世世世石でディストンションフィールドを作ると貴世世世世世世世石のディストンションフィールドができる 上記 登記

161 名前:ご冗談でしょう?名無しさん :2015/05/16(土) 06:29:22.08 ID:7FYVgp1Lc
貴世世世世世世世世石でディストンションフィールドを作ると貴世世世世世世世世石のディストンションフィールドができる 上記 登記

貴世世世世世世世世世石でディストンションフィールドを作ると貴世世世世世世世世世石のディストンションフィールドができる 上記 登記

貴世世世世世世世世世世石でディストンションフィールドを作ると貴世世世世世世世世世世石のディストンションフィールドができる 上記 登記

1028 :ご冗談でしょう?名無しさん:2015/05/16(土) 07:04:19.99 ID:7FYVgp1Lc
162 名前:ご冗談でしょう?名無しさん :2015/05/16(土) 06:30:00.70 ID:7FYVgp1Lc
貴世世世世世世世世世世世石でディストンションフィールドを作ると貴世世世世世世世世世世世石のディストンションフィールドができる

1029 :ご冗談でしょう?名無しさん:2015/05/16(土) 07:05:06.29 ID:7FYVgp1Lc
165 :ご冗談でしょう?名無しさん:2015/05/16(土) 06:53:57.64 ID:7FYVgp1Lc[5/19]
貴世世世世世世世世世世世石でディストンションフィールドを作ると貴世世世世世世世世世世世石のディストンションフィールドができる 上記 登記
166 :ご冗談でしょう?名無しさん:2015/05/16(土) 06:54:29.88 ID:7FYVgp1Lc[6/19]
貴世世世世世世世世世世世世石でディストンションフィールドを作ると貴世世世世世世世世世世世世石のディストンションフィールドができる 上記 登記
167 :ご冗談でしょう?名無しさん:2015/05/16(土) 06:55:02.88 ID:7FYVgp1Lc[7/19]
貴世世世世世世世世世世世世世石でディストンションフィールドを作ると貴世世世世世世世世世世世世世石のディストンションフィールドができる 上記 登記
168 :ご冗談でしょう?名無しさん:2015/05/16(土) 06:55:26.85 ID:7FYVgp1Lc[8/19]
貴世世世世世世世世世世世世世世石でディストンションフィールドを作ると貴世世世世世世世世世世世世世世石のディストンションフィールドができる 上記 登記
169 :ご冗談でしょう?名無しさん:2015/05/16(土) 06:55:50.42 ID:7FYVgp1Lc[9/19]
貴世世世世世世世世世世世世世世世石でディストンションフィールドを作ると貴世世世世世世世世世世世世世世世石のディストンションフィールドができる 上記 登記
170 :ご冗談でしょう?名無しさん:2015/05/16(土) 06:56:33.75 ID:7FYVgp1Lc[10/19]
貴世世世世世世世世世世世世世世世世石でディストンションフィールドを作ると貴世世世世世世世世世世世世世世世世石のディストンションフィールドができる 上記 登記
171 :ご冗談でしょう?名無しさん:2015/05/16(土) 06:56:58.80 ID:7FYVgp1Lc[11/19]
貴世世世世世世世世世世世世世世世世世石でディストンションフィールドを作ると貴世世世世世世世世世世世世世世世世世石のディストンションフィールドができる 上記 登記
172 :ご冗談でしょう?名無しさん:2015/05/16(土) 06:57:26.52 ID:7FYVgp1Lc[12/19]
貴世世世世世世世世世世世世世世世世世世石でディストンションフィールドを作ると貴世世世世世世世世世世世世世世世世世世石のディストンションフィールドができる 上記 登記
173 :ご冗談でしょう?名無しさん:2015/05/16(土) 06:57:49.83 ID:7FYVgp1Lc[13/19]

1030 :ご冗談でしょう?名無しさん:2015/05/16(土) 07:05:32.65 ID:7FYVgp1Lc
貴世世世世世世世世世世世世世世世世世世世石でディストンションフィールドを作ると貴世世世世世世世世世世世世世世世世世世世石のディストンションフィールドができる 上記 登記
174 :ご冗談でしょう?名無しさん:2015/05/16(土) 06:58:14.83 ID:7FYVgp1Lc[14/19]
貴世世世世世世世世世世世世世世世世世世世世石でディストンションフィールドを作ると貴世世世世世世世世世世世世世世世世世世世世石のディストンションフィールドができる 上記 登記
175 :ご冗談でしょう?名無しさん:2015/05/16(土) 06:58:50.12 ID:7FYVgp1Lc[15/19]
貴世世世世世世世世世世世世世世世世世世世世世石でディストンションフィールドを作ると貴世世世世世世世世世世世世世世世世世世世世世石のディストンションフィールドができる 上記 登記
176 :ご冗談でしょう?名無しさん:2015/05/16(土) 06:59:12.54 ID:7FYVgp1Lc[16/19]
貴世世世世世世世世世世世世世世世世世世世世世石でディストンションフィールドを作ると貴世世世世世世世世世世世世世世世世世世世世世石のディストンションフィールドができる 上記 登記
177 :ご冗談でしょう?名無しさん:2015/05/16(土) 06:59:37.23 ID:7FYVgp1Lc[17/19]

1031 :ご冗談でしょう?名無しさん:2015/05/16(土) 07:06:00.61 ID:7FYVgp1Lc
貴世世世世世世世世世世世世世世世世世世世世世世石でディストンションフィールドを作ると貴世世世世世世世世世世世世世世世世世世世世世世石のディストンションフィールドができる 上記 登記
178 :ご冗談でしょう?名無しさん:2015/05/16(土) 06:59:59.10 ID:7FYVgp1Lc[18/19]
貴世世世世世世世世世世世世世世世世世世世世世世世石でディストンションフィールドを作ると貴世世世世世世世世世世世世世世世世世世世世世世世石のディストンションフィールドができる 上記 登記
179 :ご冗談でしょう?名無しさん:2015/05/16(土) 07:00:21.49 ID:7FYVgp1Lc[19/19]
貴世世世世世世世世世世世世世世世世世世世世世世世世石でディストンションフィールドを作ると貴世世世世世世世世世世世世世世世世世世世世世世世世石のディストンションフィールドができる 上記 登記

1032 :ご冗談でしょう?名無しさん:2015/05/16(土) 07:16:40.61 ID:7FYVgp1Lc
180 :ご冗談でしょう?名無しさん:2015/05/16(土) 07:15:26.82 ID:7FYVgp1Lc[20/22]
貴世世世世世世世世世世世世世世世世世世世世世世世世世石でディストンションフィールドを作ると貴世世世世世世世世世世世世世世世世世世世世世世世世世石のディストンションフィールドができる 上記 登記
181 :ご冗談でしょう?名無しさん:2015/05/16(土) 07:15:49.40 ID:7FYVgp1Lc[21/22]
貴世世世世世世世世世世世世世世世世世世世世世世世世世世石でディストンションフィールドを作ると貴世世世世世世世世世世世世世世世世世世世世世世世世世世石のディストンションフィールドができる 上記 登記
182 :ご冗談でしょう?名無しさん:2015/05/16(土) 07:16:11.31 ID:7FYVgp1Lc[22/22]
貴世世世世世世世世世世世世世世世世世世世世世世世世世世世石でディストンションフィールドを作ると貴世世世世世世世世世世世世世世世世世世世世世世世世世世世石のディストンションフィールドができる 上記 登記

1033 :ご冗談でしょう?名無しさん:2015/05/16(土) 07:39:35.56 ID:7FYVgp1Lc
貴世世世世世世世世無限超世世世世世世世世世世世世世世世世世世世石でディストンションフィールドを作ると貴世世世世世世無限超世世世世世世世世世世世世世世世世世世世世世石のディストンションフィールドができる 上記 登記

1034 :ご冗談でしょう?名無しさん:2015/05/16(土) 09:05:27.74 ID:uAd6uD4lf
貴世世世世世世世世無限超世世世世世世世世世世無限超世世世無限超世世世世世世石でディストンションフィールドを作ると貴世世世世世世無限超世世世世世世世世世世世世世無限超世世世世無限超世世世世石のディストンションフィールドができる 上記 登記

1035 :ご冗談でしょう?名無しさん:2015/05/16(土) 12:41:39.08 ID:R4Iyylobg
貴世世世世世世世世無限超世世世世世世世世世世無限超世世世無限超世世無限超世世世無限超世石でディストンションフィールドを作ると貴世世世世世世無限超世世世世世世世世世世世世世無限超世世世世無限超世世世無限超世石のディストンションフィールドができる 上記 登記

1036 :ご冗談でしょう?名無しさん:2015/05/16(土) 21:18:43.99 ID:bgrfjQQKz
貴世世世世世世世世無限超世世世世世世世世世世無限超世世世無限超世世世世世無限超世石でディストンションフィールドを作ると貴世世世世世世無限超世世世世世世世世世世世世世無限超世世世世無限超世世世無限超世石のディストンションフィールドができる 上記 登記

1037 :ご冗談でしょう?名無しさん:2015/05/16(土) 21:19:14.89 ID:bgrfjQQKz
貴世世世世世世世世無限超世世世世世世世世世世無限超世世世無限超世世世世世無限超世石でディストンションフィールドを作ると貴世世世世世世無限超世世世世世世世世世世世世世無限超世世世世無限超世世世無限超世石のディストンションフィールドができる 上記 登記

1038 :ご冗談でしょう?名無しさん:2015/05/16(土) 21:26:03.66 ID:bgrfjQQKz
貴世世世無限超世世世世世無限超世世世世世世世世世世無限超世世世無限超世世無限超世世世無限超世石でディストンションフィールドを作ると

1039 :ご冗談でしょう?名無しさん:2015/05/16(土) 21:26:39.50 ID:bgrfjQQKz
貴世世世無限超世世世世世無限超世世世世世世世世世世無限超世世世無限超世世無限超世世世無限超世石でディストンションフィールドを作ると

1040 :ご冗談でしょう?名無しさん:2015/05/16(土) 21:29:59.19 ID:bgrfjQQKz
貴世世世無限超世世世無限超世世世世世世世世世世世世世無限超世世世世無限超世世世無限超世石のディストンションフィールドができる 上記 登記

1041 :ご冗談でしょう?名無しさん:2015/05/16(土) 21:31:16.98 ID:bgrfjQQKz
貴世世世無限超世世世世世無限超世世世世無限超世世世世世世無限超世世世無限超世世無限超世世世無限超世石でディストンションフィールドを作ると

1042 :ご冗談でしょう?名無しさん:2015/05/16(土) 21:33:02.48 ID:bgrfjQQKz
貴世世世無限超世世世無限超世世世世世無限超世世世世世世世世無限超世世世世無限超世世世無限超世石のディストンションフィールドができる 上記 登記

1043 :ご冗談でしょう?名無しさん:2015/05/16(土) 23:07:43.73 ID:zN0KMyqUU
貴世世世無限超世世世世世無限超世世無限超世世無限超世世世世世世無限超世世世無限超世世無限超世世世無限超世石でディストンションフィールドを作ると

1044 :ご冗談でしょう?名無しさん:2015/05/16(土) 23:08:27.36 ID:zN0KMyqUU
貴世世世無限超世世世無限超世世無限超世世世無限超世世世世世世世世無限超世世世世無限超世世世無限超世石のディストンションフィールドができる 上記 登記

1045 :ご冗談でしょう?名無しさん:2015/05/16(土) 23:10:39.31 ID:zN0KMyqUU
貴世世世無限超世世無限超世世世無限超世世無限超世世無限超世世世世世世無限超世世世無限超世世無限超世世世無限超世石でディストンションフィールドを作ると

1046 :ご冗談でしょう?名無しさん:2015/05/16(土) 23:11:38.57 ID:zN0KMyqUU
貴世世世無限超世世世無限超世世無限超世世世無限超世世世無限超世世世世世無限超世世世世無限超世世世無限超世石のディストンションフィールドができる 上記 登記

1047 :ご冗談でしょう?名無しさん:2015/05/16(土) 23:13:58.73 ID:zN0KMyqUU
貴世世世無限超世世無限超世世世無限超世世無限超世世無限超世世世無限超世世世無限超世世世無限超世世無限超世世世無限超世石でディストンションフィールドを作ると

1048 :ご冗談でしょう?名無しさん:2015/05/16(土) 23:14:50.27 ID:zN0KMyqUU
貴世世世無限超世世世無限超世世無限超世世世無限超世世世無限超世世世無限超世世無限超世世世世無限超世世世無限超世石のディストンションフィールドができる 上記 登記

1049 :ご冗談でしょう?名無しさん:2015/05/16(土) 23:18:55.93 ID:zN0KMyqUU
貴世世世無限超世世無限超世世無限超世無限超世世無限超世世無限超世世世無限超世世世無限超世世世無限超世世無限超世世世無限超世石でディストンションフィールドを作ると

1050 :ご冗談でしょう?名無しさん:2015/05/16(土) 23:21:32.18 ID:zN0KMyqUU
貴世世世無限超世世世無限超世無限超世無限超世世無限超世無限超世世世無限超世世世無限超世世無限超世世世世無限超世世世無限超世石のディストンションフィールドができる 上記 登記

1051 :ご冗談でしょう?名無しさん:2015/05/27(水) 18:27:54.48 ID:BtPTqGdqA
貴世世世無限超世世無限超世世無限超世無限超世世無限超世世無限超世世世無限超世世世無限超世世世無限超世世無限超世世世無限超世石無限超世世世無限超世石でディストンションフィールドを作ると

1052 :ご冗談でしょう?名無しさん:2015/05/27(水) 18:29:42.22 ID:BtPTqGdqA
貴世世世無限超世世無限超世世無限超世無限超世世無限超世世無限超世世世無限超世世世無限超世世世無限超世世無限超世世世無限超世石でディストンションフィールドを作ると

1053 :ご冗談でしょう?名無しさん:2015/05/27(水) 18:30:16.83 ID:BtPTqGdqA
貴世世世無限超世世無限超世世無限超世無限超世世無限超世世無限超世世世無限超世世世無限超世世世無限超世世無限超世世世無限超世石でディストンションフィールドを作ると

1054 :ご冗談でしょう?名無しさん:2015/05/27(水) 18:31:20.59 ID:BtPTqGdqA
貴世世世無限超世世世無限超世無限超世無限超世世無限超世無限超世世世無限超世世世無限超世世無限超世世世世無限超世世世無限超世石のディストンションフィールドができる 上記 登記

1055 :ご冗談でしょう?名無しさん:2015/05/27(水) 18:39:42.33 ID:BtPTqGdqA
貴世世世無限超世世無限超世世無限超世無限超世世無限超世世無限超世世世無限超世世世無限超世世世無限超世世無限超世世世無限超世石でディストンションフィールドを作ると

1056 :ご冗談でしょう?名無しさん:2015/05/27(水) 18:40:47.88 ID:BtPTqGdqA
貴世世世無限超世世世無限超世無限超世無限超世世無限超世無限超世世世無限超世世世無限超世世無限超世世世世無限超世世世無限超世石のディストンションフィールドができる 上記 登記

1057 :ご冗談でしょう?名無しさん:2015/05/27(水) 18:45:12.73 ID:BtPTqGdqA
貴世世世無限超世世無限超世世無限超世無限超世世無限超世世無限超世世世無限超世世世無限超世世世無限超世世無限超世世世無限超世石無限超世世世無限超世石でディストンションフィールドを作ると

1058 :ご冗談でしょう?名無しさん:2015/05/27(水) 18:45:39.57 ID:BtPTqGdqA
貴世世世無限超世世世無限超世無限超世無限超世世無限超世無限超世世世無限超世世世無限超世世無限超世世世世無限超世世世無限超世石無限超世世世無限超世石のディストンションフィールドができる 上記 登記

1059 :ご冗談でしょう?名無しさん:2015/05/27(水) 18:57:01.58 ID:BtPTqGdqA
貴世世世無限超世世無限超世世無限超世無限超世世無限超世世無限超世世世無限超世世世無限超世世世無限超世世無限超世世世無限超世石無限超世世世無限超世石無限超世世世無限超世石でディストンションフィールドを作ると

1060 :ご冗談でしょう?名無しさん:2015/05/27(水) 18:57:28.40 ID:BtPTqGdqA
貴世世世無限超世世世無限超世無限超世無限超世世無限超世無限超世世世無限超世世世無限超世世無限超世世世世無限超世世世無限超世石無限超世世世無限超世石無限超世世世無限超世石のディストンションフィールドができる 上記 登記

1061 :ご冗談でしょう?名無しさん:2015/05/27(水) 19:26:29.08 ID:BtPTqGdqA
貴世世世無限超世世無限超世世無限超世無限超世世無限超世世無限超世世世無限超世世世無限超世世世無限超世世無限超世世世無限超世石無限超世世世無限超世石無限超世世世無限超世石無限超世世世無限超世石でディストンションフィールドを作ると

1062 :ご冗談でしょう?名無しさん:2015/05/27(水) 19:27:34.07 ID:BtPTqGdqA
貴世世世無限超世世世無限超世無限超世無限超世世無限超世無限超世世世無限超世世世無限超世世無限超世世世世無限超世世世無限超世石無限超世世世無限超世石無限超世世世無限超世石無限超世世世無限超世石のディストンションフィールドができる 上記 登記

1063 :ご冗談でしょう?名無しさん:2015/05/27(水) 19:45:08.42 ID:BtPTqGdqA
貴世世世無限超世世無限超世世無限超世無限超世世無限超世世無限超世世世無限超世世世無限超世世世無限超世世無限超世世世無限超世石無限超世世世無限超世石無限超世世世無限超世石無限超世世世無限超世石でディストンションフィールドを作ると

1064 :ご冗談でしょう?名無しさん:2015/05/27(水) 19:45:29.55 ID:BtPTqGdqA
貴世世世無限超世世世無限超世無限超世無限超世世無限超世無限超世世世無限超世世世無限超世世無限超世世世世

1065 :ご冗談でしょう?名無しさん:2015/05/27(水) 19:49:21.48 ID:BtPTqGdqA
貴世世世無限超世世世無限超世無限超世無限超世世無限超世無限超世世世無限超世世世無限超世世無限超世世世世無限超世世世無限超世石

1066 :ご冗談でしょう?名無しさん:2015/05/27(水) 19:49:45.12 ID:BtPTqGdqA
無限超世世世無限超世石無限超世世世無限超世石無限超世世世無限超世石のディストンションフィールドができる 上記 登記

1067 :ご冗談でしょう?名無しさん:2015/06/02(火) 20:10:28.12 ID:CWP51FwyL
貴世世世無限超世世無限超世世無限超世無限超世世無限超世世無限超世世世無限超世世世無限超世世世無限超世世無限超世世世無限超世石無限超世世世無限超世石無限超世世世無限超世石無限超世世世無限超世石無限超世世世無限超世石でディストンションフィールドを作ると

1068 :ご冗談でしょう?名無しさん:2015/06/02(火) 20:25:59.42 ID:CWP51FwyL
貴世世世無限超世世世無限超世無限超世無限超世世無限超世無限超世世世無限超世世世無限超世世無限超世世世世

1069 :ご冗談でしょう?名無しさん:2015/06/02(火) 20:26:29.71 ID:CWP51FwyL
無限超世世世無限超世石無限超世世世無限超世石無限超世世世無限超世石無限超世世世無限超世石無限超世世世無限超世石無限超世世世無限超世石無限超世世世無限超世石無限超世世世無限超世石のディストンションフィールドができる 上記 登記

1070 :ご冗談でしょう?名無しさん:2015/06/02(火) 20:26:50.67 ID:CWP51FwyL
貴世世世無限超世世無限超世世無限超世無限超世世無限超世世無限超世世世無限超世世世無限超世世世無限超世世無限超世世世無限超世石無限超

1071 :ご冗談でしょう?名無しさん:2015/06/02(火) 20:27:31.94 ID:CWP51FwyL
世世無限超世石無限超世世世無限超世石無限超世世世無限超世石無限超世世世無超世石無限超世世世無超世石で限ディストンションフィールドを作ると

1072 :ご冗談でしょう?名無しさん:2015/06/02(火) 20:28:00.01 ID:CWP51FwyL
無限超世世世無限超世石無限超世世世無限超世石無限超世世世無限超世石無限超世世世無限超世石無限超世世世無限超世石無限超世世世無限超世石無限超世世世無限超世石無限超世世世無限超世石無限超世世世無限超世石のディストンションフィールドができる 上記 登記

1073 :ご冗談でしょう?名無しさん:2015/06/02(火) 20:28:25.08 ID:CWP51FwyL
貴世世世無限超世世無限超世世無限超世無限超世世無限超世世無限超世世世無限超世世世無限超世世世無限超世世無限超世世世無限超世石無限超

1074 :ご冗談でしょう?名無しさん:2015/06/02(火) 20:28:53.52 ID:CWP51FwyL
世世世無限超世石無限超世世世無限超世石無限超世世世無限超世石無限超世世世無超世石無限超世世世無限超世石でディストンションフィールドを作ると

1075 :ご冗談でしょう?名無しさん:2015/06/02(火) 20:29:18.86 ID:CWP51FwyL
無限超世世世無限超世石無限超世世世無限超世石無限超世世世無限超世石無限超世世世無限超世石無限超世世世無限超世石無限超世世世無限超世石無限超世世世無限超世石無限超世世世無限超世石無限超世世世無限超世石無限超世世世無限超世石の

1076 :ご冗談でしょう?名無しさん:2015/06/02(火) 20:30:22.90 ID:CWP51FwyL
ディストンションフィールドができる 上記 登記

1077 :ご冗談でしょう?名無しさん:2015/07/08(水) 10:18:13.28 ID:Yi9P7OSzW
ウェカメラ、カメラで、インターネット、ネットに、顔、体の情報を読み取り、
インターネット、ネットに取り込み、表示することができる。横浜国立大学にある。上記 登記

1078 :ご冗談でしょう?名無しさん:2015/07/08(水) 10:22:54.71 ID:Yi9P7OSzW
ウェブカメラ、カメラで、インターネット、ネットに、顔、体の情報を読み取り、インターネット、ネットに取り込み、表示することができる。横浜国立大学にある。上記 登記

1079 :ご冗談でしょう?名無しさん:2015/07/08(水) 10:23:32.71 ID:Yi9P7OSzW
ウェブカメラ、カメラで、インターネット、ネットに、顔、体の情報を読み取り、取り込み、インターネット、ネットに取り込み、表示することができる。横浜国立大学にある。上記 登記

1080 :ご冗談でしょう?名無しさん:2015/08/03(月) 21:40:44.22 ID:q7ijjycEl
涙腺崩壊 号泣
http://power-of-nature.blue/cmyhl/

1081 :ご冗談でしょう?名無しさん:2016/02/18(木) 20:18:55.41 ID:1Q9Bb34EC
フェイトのバルデッシュ・ガンバー
ビールと、ポカリスエットと、剣聖神癒癒し回復食と、生命の水を入れて、貴世石を作ったのが、フェイトのバルデッシュ・ガンバー、ポカリスエットと白砂糖を入れてる喰った生命の水に賢者の石とその応用でを化合して、
生命の石をシリコンマットで下に包み、JAVA++Cでプログラムして、光水をそばに置いて、空中浮遊石ができる。登記
marriageの外部ログインのプログラムを使うといい 登記 ??
Rock54: Caution(BBR-MD5:0be15ced7fbdb9fdb4d0ce1929c1b82f)

674 :ご冗談でしょう?名無しさん:2016/02/17(水) 21:30:09.06 ID:???
フェイトのバルデッシュ・ガンバー
ビールと、ポカリスエットと、剣聖神癒癒し回復食と、生命の水を入れて、貴世石を作ったのが、フェイトのバルデッシュ・ガンバー、ポカリスエットと白砂糖を入れてる喰った生命の水に賢者の石とその応用でを化合して、
生命の石をシリコンマットで下に包み、JAVA++Cでプログラムして、光水をそばに置いて、空中浮遊石ができる。登記
marriageの外部ログインのプログラムを使うといい 登記 ??
Rock54: Caution(BBR-MD5:0be15ced7fbdb9fdb4d0ce1929c1b82f)

ビールと、ポカリスエットと、上級剣聖神癒癒し回復食と、生命の水を入れて、貴世石を作ったのが、フェイトのバルデッシュ・ガンバー、ポカリスエットと白砂糖を入れてつくった生命の水に賢者の石とその応用でを化合して、
生命の石をシリコンマットで下に包み、JAVA++Cでプログラムして、光水をそばに置いて、空中浮遊石ができる。登記
marriageの外部ログインのプログラムを使うといい 登記

1082 :ご冗談でしょう?名無しさん:2016/07/26(火) 11:54:17.82 ID:sjZc+rmSO
魔法高校でも習う恒星間の惑星の移動方法、ゲートで、要塞間の空間も移動できる
デルタ新鉄鋼で、扉の枠をつける、扉の枠に貴世石を付ける。扉を付けて貴世石を付ける 恒星間惑星扉 上記 登記

デルタ新鉄鋼で、扉の枠をつける、扉の枠に貴世石を付ける。扉を付けて エネルギーゲートを付ける  恒星間惑星扉 上記 登記

デルタ新鉄鋼で、扉の枠をつける、扉の枠に貴世石を付ける。扉を付けて ゲートを付ける  恒星間惑星扉  上記 登記

1083 :ご冗談でしょう?名無しさん:2016/08/04(木) 11:28:57.88 ID:LTn/02Ze3
ちょっとしたゴミ

1084 :ご冗談でしょう?名無しさん:2016/10/28(金) 13:05:42.93 ID:AQZCD2xbb
フェイトのバルデッシュ・ガンバー 進化系
ビールと、ポカリスエットと、剣聖神癒癒し回復食と、生命の水を入れて、貴世世石を作ったのが、フェイトのバルデッシュ・ガンバー、ポカリスエットと白砂糖を入れてる喰った生命の水に賢者の石とその応用でを化合して、
生命の石をシリコンマットで下に包み、JAVA++Cでプログラムして、光水をそばに置いて、空中浮遊石ができる。登記
marriageの外部ログインのプログラムを使うといい 登記 ??
Rock54: Caution(BBR-MD5:0be15ced7fbdb9fdb4d0ce1929c1b82f)

674 :ご冗談でしょう?名無しさん:2016/02/17(水) 21:30:09.06 ID:???
フェイトのバルデッシュ・ガンバー 進化系
ビールと、ポカリスエットと、剣聖神癒癒し回復食と、生命の水を入れて、貴世世石を作ったのが、フェイトのバルデッシュ・ガンバー、ポカリスエットと白砂糖を入れてる喰った生命の水に賢者の石とその応用でを化合して、
生命の石をシリコンマットで下に包み、JAVA++Cでプログラムして、光水をそばに置いて、空中浮遊石ができる。登記
marriageの外部ログインのプログラムを使うといい 登記 ??
Rock54: Caution(BBR-MD5:0be15ced7fbdb9fdb4d0ce1929c1b82f)

ビールと、ポカリスエットと、上級剣聖神癒癒し回復食と、生命の水を入れて、貴世世石を作ったのが、フェイトのバルデッシュ・ガンバー、ポカリスエットと白砂糖を入れてつくった生命の水に賢者の石とその応用でを化合して、
生命の石をシリコンマットで下に包み、JAVA++Cでプログラムして、光水をそばに置いて、空中浮遊石ができる。登記
marriageの外部ログインのプログラムを使うといい 登記

1085 :ご冗談でしょう?名無しさん:2016/10/28(金) 13:08:24.27 ID:AQZCD2xbb
無から有を作り出す機械で、食料を多くする、その後、一つぶを、小さなものを大きなものにかえる機械で

、大きくする、そして、そのかけらの一つを、または、その一つを、無から有を作り出す機械で、食料を多くする、その繰り返しで

、食料、および、物は、増えていく、食料不足はない、物不足はない、機械不足はない、その他の不足はない 上記 登記

1086 :人や社会が超音波で襲われています:2016/12/18(日) 21:42:32.75 ID:0REXya7Ek
 
 人や社会が超音波で襲われています。
 
 
 超音波の測定器・受信機があれば、誰か、世の中の超音波を聴いてみてください・・・。
 
 誰か、超音波の測定器・受信機を作って聴いてみてください・・・。
 

1087 :ご冗談でしょう?名無しさん:2017/01/21(土) 23:40:26.43 ID:VGUaJNW/X
反重力板を重ね合わせればいい、インテリジェントデバイスで操作すればいい。上記 登記

総レス数 1087
306 KB
掲示板に戻る 全部 前100 次100 最新50
read.cgi ver 2014.07.20.01.SC 2014/07/20 D ★